古代〜近代(19世紀)迄の戦術・兵器・軍装等 3

このエントリーをはてなブックマークに追加
1某研究者 ◆TkxmpUgI
古代〜近代(19世紀)迄の戦術・兵器・軍装に関して
議論或いは情報を提供して貰いたい訳だが
2某研究者 ◆TkxmpUgI :02/08/01 13:20 ID:YkHpn3vc
前スレッド
古代〜近代(19世紀)迄の戦術・兵器・軍装等 2
http://choco.2ch.net/test/read.cgi/army/1016342597/
3名無し三等兵:02/08/01 13:31 ID:j4rtOMXe
4SSBN:02/08/01 17:51 ID:???
新スレ乙カレー
5Lans:02/08/02 11:17 ID:???
旧スレももったいないから、最後まで使いましょうね(w
6Lans:02/08/05 15:47 ID:LsB6NVnb
どうやら前スレが完了したようですな。
(早かったなぁ。最後の方読めなかったよ(w
よって、次回からはここで・・・という事で
「確認AGE」
7海の人●海の砒素:02/08/05 16:00 ID:???
 新スレおつ〜:-)
8おた:02/08/05 16:00 ID:???
だんだんなじみが増えて来たな(笑
9某研究者 ◆TkxmpUgI :02/08/05 17:21 ID:d5P98KLq
http://www.varmouries.com/vdoubl.html
まあこれを見るとプレートアーマーの足の尖った先端部は
簡単に取り外し可能の様にも見えるが
(しかし上の脚部は鎖はおろか綿入れさえ無い様であるが
 脚部は打撃や隙間への攻撃には脆かった訳なのだろうか)
10某研究者 ◆TkxmpUgI :02/08/05 17:28 ID:d5P98KLq
http://www.varmouries.com/vpics/fecht01j.jpg
http://www.varmouries.com/vpics/fecht01i.jpg
まあしかし上の右の騎士は剣の柄で相手を殴ろうとでも言うのだろうか
11Lans:02/08/05 17:29 ID:LsB6NVnb
>9
これって、騎乗用のじゃないですかね?
その為に脚部は騎乗時を考慮して、裏側とか装甲簡略化してそうですよ。

また、下にはいてるズボン側に詰め物がある可能性はどうでしょう?
(ズボンの足先がなんか不自然に見えるんですが・・・)
12某研究者 ◆TkxmpUgI :02/08/05 17:34 ID:d5P98KLq
http://www.varmouries.com/vpics/fecht01d.jpg
http://www.varmouries.com/vpics/fecht01c.jpg
このタイプのプレートアーマーの上体と言うのは
多少は動けると言う事なのだろうか(まあ防御力はやや落ちるのかも知れないが)
13某研究者 ◆TkxmpUgI :02/08/05 17:37 ID:d5P98KLq
>これって、騎乗用のじゃないですかね?
>その為に脚部は騎乗時を考慮して、裏側とか装甲簡略化してそうですよ。

只下馬戦闘の多く成る時代では
これではどうなのかだろうが
(まあ馬上槍試合ではこれでも構わないのだろうが)


>また、下にはいてるズボン側に詰め物がある可能性はどうでしょう?
>(ズボンの足先がなんか不自然に見えるんですが・・・)

まあ靴が尖っているのは鉄靴の形状に合わせた物だろうか
ズボンは綿入れは兎も角鎖は入っていない訳だろうか
14某研究者 ◆TkxmpUgI :02/08/05 17:39 ID:d5P98KLq
http://www.varmouries.com/vpics/fecht01g.jpg
上しかしこれでまともに呼吸出来るのだろうか(苦笑
15AIRSHIP:02/08/05 19:16 ID:zHFnxWB5
>14
充分出来ます。 首の部分の固定部とヘルメットの隙間及びバイザー部からの空気で
充分です。 之は,C.S.Aアヴァロンの実際の物を被った経験からです。 
アヴァロンは、今回のブラック・ホールでも実際に西洋甲冑に拠る剣闘を披露してく
れましたので御覧になった方も多いと思います。
16某研究者 ◆TkxmpUgI :02/08/05 19:36 ID:wMfLfBpe
>充分出来ます。 首の部分の固定部とヘルメットの隙間及びバイザー部からの空気で
>充分です。 

まあそうするとバイザーに多数空けられている穴は呼吸穴と言うよりは覗き窓である訳だろうか
(サレットには穴が少ないのは薔薇戦争時に用いられた兜であり
 ロングボウ対策の為に無用な穴は空けなかったと言う事だろうか
 (まあ薔薇戦争以後はロングボウ兵も壊滅したので覗き窓も増やせた訳だろうか))
17Lans:02/08/05 19:39 ID:LsB6NVnb
>15
いいところに物知りな方が(喜

という事でご質問させてください。
あのようなアーマーの隙間、どのくらいあるのでしょう?
首あたりの余裕とか・・・
(いやぁ、どのくらいの衝撃・・・というかへこみで呼吸困難になるのか、前から知りたかったんで(w
18某研究者 ◆TkxmpUgI :02/08/05 19:40 ID:wMfLfBpe
まあ薔薇戦争のみではなく百年戦争後半の兜にも
穴が少ない物が有ったかとは思うがこれはロングボウ対策だろうか
(まあ馬上試合用の物もランスが飛び込まぬ為に穴の数は少ないと言う事だろうか)
19某研究者 ◆TkxmpUgI :02/08/05 19:45 ID:wMfLfBpe
>あのようなアーマーの隙間、どのくらいあるのでしょう?
>首あたりの余裕とか・・・

まあトーナメント用の物には隙間が多く
首が左右に動かない様に鎧と一体化された物も有る訳だろうが
(矢張りランスの衝撃が如何に大きいかと言う事だろうか)

隙間は物にも拠るだろうがトーナメント用の物が
矢張り一番大きいと見て良い訳だろうか
(まああれだけ有ればランスを受けても呼吸困難には成らないと言う事なのか)
まあトーナメント用は兎も角通常の物は両手剣や両手斧・ランスをまともに受けて無事であるなら
首に槍や刀が当たるのを防ぐ為の肩部のパスガード等は装備されなかったかも知れぬが(笑
20某研究者 ◆TkxmpUgI :02/08/05 19:52 ID:wMfLfBpe
まあしかし首の部分をクロスボウやロングボウが至近距離で貫通しても
致命傷とは成らぬだけの隙間・装甲の厚みは有った訳なのだろうか
21某研究者 ◆TkxmpUgI :02/08/05 20:01 ID:wMfLfBpe
http://www.karlofgermany.com/master04.htm
まあこのタイプでは首の前後に装甲が
追加されている訳だろうか
(まあ故に通常の物は十分で無いのかパスガードを省略する為の
 物なのだろうか)
22某研究者 ◆TkxmpUgI :02/08/05 20:06 ID:wMfLfBpe
http://www.karlofgermany.com/master06.htm
まあこれ等は隙間の無い関節部の構造が宇宙服の参考と迄された様だが
関節部の打撃・貫通に対する防御能力はどの程度な訳だろうか
23某研究者 ◆TkxmpUgI :02/08/05 20:37 ID:wMfLfBpe
http://www.metmuseum.org/collections/view1.asp?dep=4&full=0&item=29%2E156%2E67a
http://www.karlofgermany.com/master19.htm
まあ上のタイプの兜の首の部分が
最も防御力は高い訳だろうか
24AIRSHIP:02/08/05 22:11 ID:zHFnxWB5
実物を見て確認したい方は、毎日曜日に小平でアヴァロンの方たちが、甲冑に身
を固めてスパーリングをやっています。 見に行って下さい。 詳しくは、アヴ
ァロンのサイトで確認して下さい。
25某研究者 ◆TkxmpUgI :02/08/05 22:19 ID:wMfLfBpe
http://members.jcom.home.ne.jp/noyes/
まあアヴァロンのサイトと言うのは矢張り上のサイトの事だろうか
26AIRSHIP:02/08/05 22:19 ID:zHFnxWB5
ごめんなさい、C.S.AではなくS.C.A(The Society of 
Creative SAnacronism)でした。酔っ払って間違えました。
27AIRSHIP:02/08/05 22:32 ID:zHFnxWB5
>17
甲冑の隙間は、胸部から腹部にかけての膨らみを除けば殆ど在りません。 
内張りに張ってあるフェルトや皮のクッシヨンが体にフィットしています。
(タセットは除く)
 はい、このサイトです。
28某研究者 ◆TkxmpUgI :02/08/05 22:41 ID:wMfLfBpe
>甲冑の隙間は、胸部から腹部にかけての膨らみを除けば殆ど在りません。 

まあそれでも1.5kgの片手剣位なら受けても問題無いだろうか
(10kgの両手剣や両手斧では流石にアウトだろうが(苦笑
 しかし3kg程度の片手斧・メイスやバスタードソードではどうなのだろうか)
重装歩兵にハルバードを振り下ろされた場合兜や肩は耐えられるだろうか
(まあパイクの打撃程度なら何とか成るかも知れないが)
29某研究者 ◆TkxmpUgI :02/08/05 22:45 ID:wMfLfBpe
>甲冑の隙間は、胸部から腹部にかけての膨らみを除けば殆ど在りません。 

まあそれでも1.5kgの片手剣位なら受けても問題無いだろうか
(10kgの両手剣や両手斧では流石にアウトだろうが(苦笑
 しかし3kg程度の片手斧・メイスやバスタードソードではどうなのだろうか)

重装歩兵にハルバードを振り下ろされた場合兜や肩は耐えられるだろうか
(まあパイクの打撃程度なら何とか成るかも知れないが)

しかしトーナメントに出る騎士は
フットトーナメントアーマーと通常のトーナメントアーマーを
皆双方用意していた訳なのだろうか
(通常のトーナメントアーマーは首や肩が余り動かぬ物も有り下腹部に隙間が有るので
 徒歩での競技には恐らく使われなかったのではないのか)
30某研究者 ◆TkxmpUgI :02/08/05 22:48 ID:wMfLfBpe
>内張りに張ってあるフェルトや皮のクッシヨンが体にフィットしています。

まあこれは現実問題打撃を止められるだけの物なのだろうか
(筋肉が有る部分はこれが薄く共良いだろうが
 骨が出ている部分は鎧が有ってさえ打撃をまともに受ければアウトではないのか)
31某研究者 ◆TkxmpUgI :02/08/05 22:59 ID:wMfLfBpe
まあプレートアーマーも全ての部分が4.5mm等の厚みは無い訳だろうが
手足等のカーブのきつい部分は余り厚みが無く共防御力は維持出来る訳だろうか
32AIRSHIP:02/08/05 23:05 ID:zHFnxWB5
1つのアーマーに付属部品を付けて、両方の用に当てている例も在ります。
貧乏な騎士や貴族は、何領もの甲冑を買えませんからね。 練習用の斧(
2Kg)を受けた後では、痣が残っていました。 体の捻りに拠ってかなり
ショックを減らす事が出来ます。 動きによって刃先に角度を付ければ、
表面を滑らすことが出来ます。
33某研究者 ◆TkxmpUgI :02/08/05 23:11 ID:wMfLfBpe
まあ溝の彫って有るマクシミリアン鎧等は基本的に通常の鎧に比べて
隙間は大きい訳だろうか


>1つのアーマーに付属部品を付けて、両方の用に当てている例も在ります。

まあこれの実例と言うのを見た事が無いが
スカートと肩(或いは頭部・腕の一部も)を交換すれば
一応徒歩用・騎乗用の両方に変更可能だろうか
その場合実戦用の鎧とトーナメント用の鎧が
同じ物である場合も多い訳だろうか
34某研究者 ◆TkxmpUgI :02/08/05 23:14 ID:wMfLfBpe
>練習用の斧(
>2Kg)を受けた後では、痣が残っていました。

まあ只これはまともに受けたと言う訳では無い訳だろうか
35某研究者 ◆TkxmpUgI :02/08/05 23:18 ID:wMfLfBpe
まあしかし馬鎧の方にも一応綿入れは
有る訳なのだろうか
36名無し三等兵:02/08/06 10:00 ID:???
アヴァロンのサイトは早速お気に入りに登録しました。
鎧と打撃の話が続いている所、横合いから済みませんが、
伺いたいことが有ります。
まず、石畳の上では金属製の靴は滑らないのだろうかということ。
騎乗の動作に鎧が支障にならないかということ。
また、呼吸ができるということは会話も支障なくできるということ。
などです。よろしければ御願いします。
37Lans:02/08/06 11:42 ID:c8UJjQL0
>AIRSHIP
いろいろとご教授ありがとうございます。
小平、時間が取れたら、ぜひ見に行ってみたいものです。
38某研究者 ◆TkxmpUgI :02/08/06 18:10 ID:DY1lKpIQ
>まず、石畳の上では金属製の靴は滑らないのだろうかということ。

まあ確か足の裏に装甲は無いのではなかったか(笑
(映画等では鎖が有る物も有る様だが実物はどうだろうか)


>騎乗の動作に鎧が支障にならないかということ。

まあ腕の動き難いトーナメントアーマー以外は
問題は無いかも知れないが(或いはトーナメントアーマーでも問題無いか)


>また、呼吸ができるということは会話も支障なくできるということ。

まあバイザーを開けた状態で口が開いている物は問題無いだろうが
バイザーを閉じた状態では穴の少ない物はどうだろうか
まあ耳の部分の開いたローマ兵の兜の様に周囲の音が聞き取れるのかは
疑問な訳だが
39某研究者 ◆TkxmpUgI :02/08/06 19:07 ID:DY1lKpIQ
>まあ腕の動き難いトーナメントアーマー以外は
>問題は無いかも知れないが(或いはトーナメントアーマーでも問題無いか)

80kgに達する物も有るトーナメントアーマーは確か吊り上げる機械を用いて
騎乗していた場合も有ったかと思うが実戦用の鎧はどうなのだろうか
(まあ只体力の無い人間は吊り上げていたのかも知れないが)
40AIRSHIP:02/08/06 19:09 ID:sayqrMqg
>36 鉄靴の裏には底は在りません。 靴の上を覆っているだけです。 
騎乗は、御付の従者やクレーンを使って乗り込みます。 階段や柵の
横木も有り。 会話は、耳穴無しの物は聞え難いです。 ローマ帝国
が集団戦のため、耳を出した兜を被った訳が良く解ります。
兜越しの声は、くぐもって聞こえます。
41某研究者 ◆TkxmpUgI :02/08/06 19:14 ID:DY1lKpIQ
まあトーナメントアーマーにも馬鎧がセットと成っいる物も有る様だが
実際のトーナメントでは馬を狙う事は無いので馬鎧は装備されない訳だろうか
(まあ只馬に逸れた攻撃が当たる確率が0でも無い訳なら装備された可能性も有る訳だろうか)
42バキャベッリ:02/08/06 20:22 ID:???
 トーナメント用はあくまで安全第一ですし、野戦用は実用的な物を使った事でしょう。
昔の人が書いた記述にクレーンいないなと書いてあるのは、単にトーナメントしか見た
事が無いので、実戦でもそうだろうと憶測で書いただけの様ですし。なんでも野戦用は
40kg位だそうですから、トーナメント用の半分で済みます。
43名無し三等兵:02/08/08 03:08 ID:???
>>40
回答有り難うございました。近くで公開されることがあったら
是非見たいです。頑張って下さい。
44某研究者 ◆TkxmpUgI :02/08/09 22:46 ID:4l9Y07qk
http://www.geocities.com/Tokyo/Pagoda/8187/Talhoffer.htm
まあこれの一番下の絵では馬上からクロスボウを使用している様だが
中国は兎も角西欧では此れは多用されてはおらぬ訳だろうか
45某研究者 ◆TkxmpUgI :02/08/09 22:55 ID:4l9Y07qk
>まあこれの一番下の絵では馬上からクロスボウを使用している様だが
>中国は兎も角西欧では此れは多用されてはおらぬ訳だろうか

まあショートボウやロングボウ(まあこれは横向きにしか使えないかも知れないが)
の馬上からの使用は有り得た訳なのだろうか


http://www.geocities.com/ulfberth/Oeselgem.htm
まあ女が棒で戦っている写真も有るが
これは実際には行われた訳なのだろうか
46名無し三等兵:02/08/10 00:21 ID:???
>>45
 クロスボウを携行した兵が騎行する図が”スイスの軍隊”に出ていました。
 馬上から射撃したか否かは定かではありませんが。
  ショートボウとロングボウについては、そもそも所持者の体格の問題による
 区分のようなものだと思います。
  日本では技術を磨けば馬上から射撃は出来ていましたが。
 (鎌倉時代の犬追物、笠懸などの訓練は馬に乗って移動する目標を比較的
  近い距離から射撃する訓練だったと思います)
47某研究者 ◆TkxmpUgI :02/08/10 06:36 ID:1NznwDM2
まあランスを構えた下馬重装騎兵相手でも
ハルバードを構えた下馬重装騎兵(ハルバードは歩兵や従者から調達か)なら
ランスは通用せず一方的に撃破可能ではないのか
(まあ或いは完全鎧を着た少数の重装歩兵にハルバードを持たせて
 下馬騎兵を叩く方法も有るだろうが数や地形が無ければ包囲され
 撃破される危険も有る訳だろうか)

まあ矢張りランスを構えた下馬騎兵に乗馬騎兵は突撃出来ぬ訳だろうか
完全鎧を着た下馬騎兵や重装歩兵であればパイク兵に対し
近接戦に持ち込めば撃破可能だろうか
(まあ只盾を構えた重装歩兵も完全鎧が無く共パイク兵に近接戦を挑めるかも知れないが
 ランスを構えた下馬騎兵に近接戦を挑むのは余程数でも無い限り自殺行為だろうか)
48某研究者 ◆TkxmpUgI :02/08/10 06:58 ID:1NznwDM2
完全鎧を来た重装歩兵(ハルバード或いは盾等を装備)は存在する様だが
パイク兵・ランツクネヒトは存在する訳なのだろうか
(完全鎧を来たランツクネヒトの両手剣でランスやパイクを叩き折り
 ハルバードを持った重装歩兵を下馬騎兵・パイク兵に乱入させる等の方向も有る訳だろうか
 完全鎧を装備しハルバードを持った重装歩兵や下馬騎兵を乱入させ
 ランス・パイクを封じた後重装歩兵が突入すると言う手も有る訳だろうが)
完全鎧を装備しハルバードを装備した相手にはハルバードを盾や武器で防ぎつつ
より戦闘能力の高い下馬騎兵を乱入させると言う方向だろうか


>クロスボウを携行した兵が騎行する図が”スイスの軍隊”に出ていました。
> 馬上から射撃したか否かは定かではありませんが。

まあカストレルと同様馬に乗って行軍する弓歩兵も存在する訳で
シャルル七世のランスでは歩兵も全て騎馬で行軍したのではなかったか


>ショートボウとロングボウについては、そもそも所持者の体格の問題による
>区分のようなものだと思います。

まあ馬上からは西欧のロングボウは
(下端が射撃時に馬に当たるので)横向きにしか使えない様だが
(其の場合射程はショートボウより劣ったと言う事は無い訳なのだろうか)
日本の弓の場合は下が短いので馬上でも使用可能で有り
歩兵も屈んで撃てると言うメリットが有った様だが
(しかし屈んで被弾面積でも減らすとでも言うのだろうか
 確か草叢から弓で奇襲する際に有利との事だがどうなのだろうか
 (まあクロスボウ等伏せて撃てる訳だろうしより有利な場合も有るだろうか
 まあ只射撃する直前迄伏せておれば射撃時に発見されたとしても
 余り問題は無い様に思えるが))
49某研究者 ◆TkxmpUgI :02/08/10 20:54 ID:G2QQltZP
http://member.nifty.ne.jp/bintaro/touken/falchion.htm
まあファルシオンは直線の峰を持っていると言うが
http://www.bnf.fr/enluminures/images/jpeg/i4_0008.jpg
上のワットタイラーを攻撃している剣は峰も湾曲しているが
これはファルシオンでは無い訳なのだろうか
(しかしカトラスにしては幅が広いのではないのか)
50某研究者 ◆TkxmpUgI :02/08/10 22:00 ID:q245TIgI
http://www.aemma.org/onlineResources/baumans/contents_body.htm
まあ上を見る限り峰が湾曲している物もファルシオンだろうか

http://www.a-work-of-art.sitehosting.net/page06.htm
まあファルシオンは近接戦用に歩兵が使用したと言う事だろうか
(故に近接戦に小剣では無くファルシオンが使用された例も有ると言う事だろうが
 小剣よりは数は少なかった訳だろうか)
51某研究者 ◆TkxmpUgI :02/08/10 22:21 ID:q245TIgI
http://www.bnf.fr/enluminures/images/jpeg/i4_0008.jpg
まあ上を見ると両手でファルオンらしき物を構えているが
両手用のファルシオンも有ったと言う事なのか
52某研究者 ◆TkxmpUgI :02/08/10 22:40 ID:q245TIgI
http://www.aemma.org/onlineResources/baumans/plate215.htm
まあ上を見ると盾を構えた侭槍を両手で使用している様だが
これは実戦で使用された例は有る訳なのだろうか


53某研究者 ◆TkxmpUgI :02/08/10 23:20 ID:q245TIgI
http://www.aemma.org/onlineResources/baumans/plate218.htm
まあ只此れが上手く使えるかは疑問も有るが

http://www.aemma.org/onlineResources/baumans/plate170.htm
上は盾に剣等を組み合わせて使用していると言う事だろうか

http://www.aemma.org/onlineResources/baumans/plate173.htm
まあこれは足で剣を封じているのか足を剣で取られているのか
何れなのだろうか

http://www.aemma.org/onlineResources/baumans/plate175.htm
まあこの様な行為はトーナメントでは卑怯として
使われなかった可能性は有る訳だろうか

http://www.aemma.org/onlineResources/baumans/plate182.htm
まあ槍と剣を組み合わせる意味は何か有る訳なのだろうか
54名無し三等兵:02/08/11 01:08 ID:8TqOSV5l
某研究者 ◆TkxmpUgI タソは、まあまあ教の教祖ですか?
55名無し三等兵:02/08/11 03:16 ID:???
<クロスボウの伏射
 次発の装填は伏せながらするのは困難なので戦場ではやらなかったと
 思います。
  また、飛翔経路も直線で射手と目標を結べず、弓なりになるので
 こういう点からも伏射は難しいのではないかと思います。

<西洋の格闘術
剣術では柄頭で至近距離の相手に打撃を与える攻撃があるのを見たことが有ります。また、ギリシャではパンクラチオンなどの格闘技があり、双子座の兄弟の
いずれか失念しましたが、一人はレスリングの天才でした。
 
ご紹介のページを見て、中世の戦闘技術はかなり高度だったのではないかと
思いました。
56某研究者 ◆TkxmpUgI :02/08/11 06:43 ID:Tw4wECoC
>ご紹介のページを見て、中世の戦闘技術はかなり高度だったのではないかと
>思いました。

まあ只この種の格闘技術を持つ者は騎士や一部の兵士のみだろうが
騎士に対しては雑兵は3対1でも相手が疲労していない状況では
撃破された可能性も有ると言う事だろうか
(まあ只弓兵は兎も角重装歩兵が全く格闘技術が無いと言う事も
 無い訳だろうが完全鎧が無い者でも騎士に2対1程度なら勝てた訳だろうか)
57某研究者 ◆TkxmpUgI :02/08/12 10:52 ID:stpRqyJ7
http://www.japaneseart.be/doc/page3.mv
まあ兎や龍の前立も有る様だが
http://www.chiba-muse.or.jp/SONAN/kikaku/yoroi/sub0003.htm
矢張り桃形兜は南蛮鎧のコピーと見て良い訳だろうか

58某研究者 ◆TkxmpUgI :02/08/12 11:08 ID:stpRqyJ7
http://www.katchu.com/html/2213.html
まあこの三種類は基本的に南蛮兜のコピーと見て良いのか

http://www.tilde.co.jp/~nao/lists/kabuto.htm
しかし上は全て実物が有ると見て良い訳なのだろうか
59某研究者 ◆TkxmpUgI :02/08/12 11:17 ID:stpRqyJ7
http://www.uhauha-uha.com/gekisen/kabuto.html
まあ上を見る限り全て実在の物の様だが
(しかし何か面に牙が生えている物が妙に多い様に見受けるが(苦笑)
日根野兜も南蛮兜の影響で作られた物の様だが
60某研究者 ◆TkxmpUgI :02/08/13 06:38 ID:OOGdq1Fz
http://www.sengokudaimyo.com/katchu/08.kabuto.html
まあ兜の頂部の凹みは打撃に拠る物かと思ったが
凹み方も兜の形式である様だが(笑

http://www.armourarchive.org/way_of_japanese_warrior/
http://www.budplant.com/prod.itml/icOid/8448
http://www.iwate-np.co.jp/isan/isan60.html
http://www.city.uozu.toyama.jp/kyoiku/kyouiku/syakyou/bunkazai/photo051.htm
http://www.chiba-muse.or.jp/SONAN/kikaku/yoroi/chakinmuinobeP.htm
まあこれを見る限り未だ見た事も無い様な物が相当程度
地方には有る訳だろうか

http://www.shikoku.ne.jp/isono/hyoshi/houmotsu/houmotsu-kan.html
まあこの象型の物はかなり後期の物だろうか
61某研究者 ◆TkxmpUgI :02/08/13 06:57 ID:OOGdq1Fz
http://www.sengokudaimyo.com/katchu/07.makeDo.html
まあ胴も板金が縦方向に分かれている物も有る様だが
鳩胸胴=南蛮胴では無い訳だろうか
62某研究者 ◆TkxmpUgI :02/08/13 09:47 ID:m92O+i6T
http://www.people.cornell.edu/pages/bjm10/hutton/rapnclo.html
まあ服やマントを敵の顔や剣に投げつけてから
剣等で敵を倒すと言う技も有る様だが

http://people.cornell.edu/pages/bjm10/hutton/gsword.html
まあ両手剣の構え方は日本刀と余り差が無い様だが
63某研究者 ◆TkxmpUgI :02/08/13 10:38 ID:m92O+i6T
http://www.aemma.org/onlineResources/thalhoffer/contents_body.htm
まあしかしこの中で実戦で使える様な技は
どれだけ有る訳なのだろうか(笑
64某研究者 ◆TkxmpUgI :02/08/13 10:50 ID:m92O+i6T
http://base.kb.dk/pls/hsk_web/hsk_vis.side?p_hs_loebenr=2&p_sidenr=221&p_illnr=0&p_frem=0&p_tilbage=0&p_navtype=rel&p_lang=eng
まあこれの後ろの方に見た事の無い様な武器や盾が出て来るが
此れ等はトーナメント専用なのだろうか
65某研究者 ◆TkxmpUgI :02/08/13 11:17 ID:m92O+i6T
http://base.kb.dk/pls/hsk_web/hsk_vis.side?p_hs_loebenr=2&p_sidenr=263&p_illnr=0&p_frem=0&p_tilbage=0&p_navtype=rel&p_lang=eng
上の129にクロスボウを馬上で構えている描写も有るが
これは試合のみの事なのだろうか
(ランスを逆手に構えている様な描写も有るがこれは無意味である可能性も
 有る訳だろうか(或いはクロスボウに対抗してランスを投げるとでも言うのだろうか))
66某研究者 ◆TkxmpUgI :02/08/13 11:22 ID:m92O+i6T
http://base.kb.dk/pls/hsk_web/hsk_vis.side?p_hs_loebenr=2&p_sidenr=264&p_illnr=0&p_frem=0&p_tilbage=0&p_navtype=rel&p_lang=eng
しかし上のランスが折れているのはどういう訳なのだろうか
(ランスの先端部が3つに割れているのは確か試合用に
 貫撤力を抑える為ではなかったか)
67某研究者 ◆TkxmpUgI :02/08/13 11:38 ID:m92O+i6T
http://base.kb.dk/pls/hsk_web/hsk_vis.side?p_hs_loebenr=2&p_sidenr=267&p_illnr=0&p_frem=0&p_tilbage=0&p_navtype=rel&p_lang=eng
しかし上の様にプレートアーマーが容易に貫通する共思えないが
(只ハンマーの裏の尖った部分をまともに受けた場合
 プレートが貫通する可能性も有るのではないのか)

http://base.kb.dk/pls/hsk_web/hsk_vis.side?p_hs_loebenr=2&p_sidenr=269&p_illnr=0&p_frem=0&p_tilbage=0&p_navtype=rel&p_lang=eng
しかしこの槍に鉤が付いた様な物も余り見ないが
このハンマーの石突きの部分は妙に尖っており槍同様の物だろうか

http://base.kb.dk/pls/hsk_web/hsk_vis.side?p_hs_loebenr=2&p_sidenr=271&p_illnr=0&p_frem=0&p_tilbage=0&p_navtype=rel&p_lang=eng
まあ上の様に鎧を着て組み合うのは関節の自由度の問題も有り
難しい場合も有るのではないのか

http://base.kb.dk/pls/hsk_web/hsk_vis.side?p_hs_loebenr=2&p_sidenr=272&p_illnr=0&p_frem=0&p_tilbage=0&p_navtype=rel&p_lang=eng
上は互いに隙間を突いていると言う事なのだろうか
(しかし上も百年戦争等の絵と同様物騒な絵が多いが(苦笑)
これで使われている鎧はフットトーナメント専用の物では無いかも知れぬが
腰の関節の自由度は問題では無いのだろうか


http://base.kb.dk/pls/hsk_web/hsk_vis.side?p_hs_loebenr=2&p_sidenr=274&p_illnr=0&p_frem=0&p_tilbage=0&p_navtype=rel&p_lang=eng
まあこのハンマーは石突きの部分に鍵迄付いているが

http://base.kb.dk/pls/hsk_web/hsk_vis.side?p_hs_loebenr=2&p_sidenr=275&p_illnr=0&p_frem=0&p_tilbage=0&p_navtype=rel&p_lang=eng
まあバイザーを開けて口や目に短剣を突き込む様な事も行われたのか
(確か通常はバイザーの下では無くスカートの下を突いていたかと思うが
 脇へは短剣を突き難い訳だろうか)
68某研究者 ◆TkxmpUgI :02/08/13 12:38 ID:m92O+i6T
http://base.kb.dk/pls/hsk_web/hsk_vis.side?p_hs_loebenr=2&p_sidenr=277&p_illnr=0&p_frem=0&p_tilbage=0&p_navtype=rel&p_lang=eng
しかしプレートアーマーの肩はこれ程上に動く訳なのだろうか


http://www.aemma.org/onlineResources/thalhoffer/aboutTalhoffer.htm
まあ妙な構造の鎧が有るがこれは馬上試合用だろうか


http://base.kb.dk/pls/hsk_web/hsk_vis.side?p_hs_loebenr=2&p_sidenr=33&p_illnr=0&p_frem=0&p_tilbage=0&p_navtype=rel&p_lang=eng
まあ左のフレイルの玉は妙に小さいが


http://base.kb.dk/pls/hsk_web/hsk_vis.side?p_hs_loebenr=2&p_sidenr=34&p_illnr=0&p_frem=0&p_tilbage=0&p_navtype=rel&p_lang=eng
これは兵員輸送車の様な物だろうか
(まあ長柄のメイスの様な物も見え良く解らない長柄武器も有るが
 この種の武器解説書には無い様な武器は他にも有る訳だろうか)
69某研究者 ◆TkxmpUgI :02/08/13 12:54 ID:m92O+i6T
http://base.kb.dk/pls/hsk_web/hsk_vis.side?p_hs_loebenr=2&p_sidenr=267&p_illnr=0&p_frem=0&p_tilbage=0&p_navtype=rel&p_lang=eng
まあ或いはこれも脇の隙間を貫いていると見て良い訳だろうか
70某研究者 ◆TkxmpUgI :02/08/13 13:33 ID:m92O+i6T
http://base.kb.dk/pls/hsk_web/hsk_vis.side?p_hs_loebenr=2&p_sidenr=150&p_illnr=0&p_frem=0&p_tilbage=0&p_navtype=rel&p_lang=eng
まあ右は黒人の様にも見えるが
黒人迄トーナメントで戦っていた訳なのだろうか

http://base.kb.dk/pls/hsk_web/hsk_vis.side?p_hs_loebenr=2&p_sidenr=152&p_illnr=0&p_frem=0&p_tilbage=0&p_navtype=rel&p_lang=eng
まあしかし右の人物の顔だけが黒いのはどう言う訳なのだろうか(苦笑


http://base.kb.dk/pls/hsk_web/hsk_vis.side?p_hs_loebenr=2&p_sidenr=157&p_illnr=0&p_frem=0&p_tilbage=0&p_navtype=rel&p_lang=eng
この様に帽子を投げ付けると言う事も当然行われた訳だろうが
トーナメントで許される行為なのだろうか(苦笑


http://base.kb.dk/pls/hsk_web/hsk_vis.side?p_hs_loebenr=2&p_sidenr=158&p_illnr=0&p_frem=0&p_tilbage=0&p_navtype=rel&p_lang=eng
まあしかしこんな間抜けなやられ方をする者が実際存在したのかは疑問も有る訳だろうが(苦笑
71某研究者 ◆TkxmpUgI :02/08/13 13:42 ID:m92O+i6T
http://base.kb.dk/pls/hsk_web/hsk_vis.side?p_hs_loebenr=2&p_sidenr=164&p_illnr=0&p_frem=0&p_tilbage=0&p_navtype=rel&p_lang=eng
まあこれは女が穴の中で動けない男と
戦わされたと言う事なのだろうか
(まあ動けない男になら勝ち得ると見ての物だろうか)
72名無し三等兵:02/08/13 13:52 ID:vuUnA4TR
>>71
それ公認された男女間決闘の一形態よ
73名無し三等兵:02/08/13 13:58 ID:???
>>70
>この様に帽子を投げ付けると言う事も当然行われた訳だろうが
>トーナメントで許される行為なのだろうか(苦笑

よく見れ、某研卿。
やられた奴の胸に短剣が刺さっているですよ。
手に持った帽子とナイフを一緒に投げるという決闘(暗殺)テクの一つ。
スパニッシュあたりでは今でもやるとかやらないとか。
あと、帽子を脱ぐフリをして、そのつばで相手の目を狙うという目つぶし、
フィンガージャブみたいなこともする。
74某研究者 ◆TkxmpUgI :02/08/13 14:00 ID:m92O+i6T
>手に持った帽子とナイフを一緒に投げるという決闘(暗殺)テクの一つ。

まあ決闘では手の内が割れている可能性も有るから使えぬかも知れぬが
暗殺には使える可能性も有る訳だろうか
75某研究者 ◆TkxmpUgI :02/08/13 14:49 ID:m92O+i6T
http://www.eastofindia.com/catalogue1_2_usd.htm]
しかし盾の銃眼から弓を放っている物も有る様だが
これでは狙いは付けられないのではないのか(苦笑
(まあ大体の距離を銃眼から除いて見て
 其の後で放つと言う事だろうか)
76名無し三等兵:02/08/13 15:04 ID:BeG3inVN
上のほうの決闘画像ページ、おもしろいです。ためにナッタ。
某研さん、いつもありがとう。
77某研究者 ◆TkxmpUgI :02/08/13 15:26 ID:m92O+i6T
http://www.eastofindia.com/catalogue_usd.htm
まあ良く見ると足軽には兵糧玉の付いた物迄有る様だが(笑

http://www.eastofindia.com/catalogue2_usd.htm
徒歩武者には兵糧玉は無いのか
或いは行軍時に使用するかも知れぬ馬の側に乗せている訳なのだろうか
78某研究者 ◆TkxmpUgI :02/08/13 15:50 ID:m92O+i6T
まあ兵糧玉を使う場合補給部隊以外の兵士が大量にやられた際でも
食料を敵に奪われる危険も有る訳だろうか
(まあ逆に補給部隊がやられても・或いは補給が遅延しても食料がある程度は保つと言う
 メリットは有る訳だろうが)


http://www.eastofindia.com/ronin.htm
どうやら忠臣蔵のフィギュア迄有る様だが


http://www.eastofindia.com/artillery.htm
しかしこの大砲と言うのは実在した訳なのだろうか


http://www.eastofindia.com/fall%202001.htm
上の中央の見張り台と言うのは実在した訳なのだろうか

しかし足軽には弓等扱えず徒歩武者以上が使ったと言う事は無い訳だろうか
長槍も非力である足軽に使いこなせたかは疑問も有る訳だろうし
足軽の武器は短槍或いは銃+刀程度と言う可能性も有る訳だろうか


http://www.eastofindia.com/Kato%20Kiyomasa%20Ashigaru.htm
矢張り中央の物が車竹束と言う事だろうか
(まあ全てが銃眼を備えている訳でも無い訳だろうか)


http://www.eastofindia.com/Spring%202002.htm
まあ徒歩武者も一部は銃を使用していた可能性も有ると言う事だろうか
79某研究者 ◆TkxmpUgI :02/08/14 07:04 ID:svOhywsw
http://www.armouryshop.co.uk/acatalog/Armoury_Shop_Armoury_Exclusives__90mm___120mm_scales__10.html
まあ実物の騎士と言うのはこれ程派手だった訳なのだろうか
(博物館に置いてある物・或いは絵に出て来る物は割りにシンプルな物が多い様にも見受けるが)


http://users.stargate.net/~ftduq/medieval.html
しかし上にランツクネヒトのパイクマンと言うのが有るが
これは実際存在した訳なのだろうか
まあ矢張り実在したとしてもスイスのパイク兵に勝る物では
無かったと言う事だろうか


http://store1.yimg.com/I/internethobbies_1695_168793005
まあこのタイプの鎧の首は自由度は高いかも知れぬが
余り見ない型だが
80某研究者 ◆TkxmpUgI :02/08/14 07:06 ID:svOhywsw
戦国時代に弓が廃れた理由と言うのは
足軽が銃を扱える様に成った為に
徒歩武者はこれ以上飛び道具は不要として弓では無く槍或いは長槍を扱ったと言う事は無いのだろうか
(騎兵も単独では運用されないので弓を扱う必要は
 無くなったと言う事だろうか)
戦国以前の徒士は長巻・騎兵・徒歩武者は弓と言う事だろうか
(槍の出て来る室町以降は徒士は長巻・短槍或いは長刀・徒歩武者・騎兵は
 槍或いは長槍と弓双方を扱ったと言う可能性も有る訳だろうか)
矢張り所詮雑兵である徒士や足軽に長槍や弓が容易に扱えたとは
思えぬ訳だろうか
(矢張り徒士や足軽に扱えるのは長巻・長刀・短槍・銃程度だろうか)
まあ只徒士や足軽の練度にも差は有り
長槍や弓を使用した足軽や徒士も有るかも知れぬが
何れにせよ徒歩武者の練度を超える事は無い訳だろうか

81某研究者 ◆TkxmpUgI :02/08/14 07:26 ID:svOhywsw
まあ西欧も軽装歩兵と弓兵以外は雑兵では無い訳だろうか
(まあ重装歩兵も完全鎧の無い物は練度は落ちる訳だろうか)
パイクは兎も角ハルバードを扱うのは鎧が軽装な雑兵も含まれる訳なのだろうか
(まあパイク兵も余り完全鎧は無い訳だし完全鎧を装備した歩兵で無く共
 ハルバードは当然扱っていた訳だろうが鎖帷子も無い軽装歩兵が
 ハルバードを使用した訳なのだろうか)
矢張り鎖帷子等を装備した精鋭クロスボウ兵と言うのは
重装歩兵に比べ余り数は多くは無い訳だろうが
(錬度の高い歩兵は矢張り重装歩兵や長弓兵と成る訳だろうか
 (まあ只長弓兵は練度が高いからと言って
  皆鎖帷子を装備している訳では無い訳だろうか))
82某研究者 ◆TkxmpUgI :02/08/14 07:36 ID:svOhywsw
まあ重装備の兵士の鎧を機動力を確保する為に戦場で軽量化する事は
行われた訳なのだろうか
(軽装の雑兵の行軍速度に合わせる場合
 或いは馬がやられた場合等で徒歩で長距離行軍せねば成らぬ場合等では
 どうだろうか)
83某研究者 ◆TkxmpUgI :02/08/14 07:47 ID:svOhywsw
>或いは馬がやられた場合等で徒歩で長距離行軍せねば成らぬ場合等では
>どうだろうか

まあクレッシーで下馬した重装騎兵の鎧を軽装化したと有ったが
これはロングボウの攻撃力の前に逆効果だった可能性も有る訳だろうか
(まあこれは徒歩での行軍距離が長いので疲労軽減の為に軽装化せざる負えなく成った可能性も有る訳だろうか
 只後の下馬戦闘では恐らく軽装化されておらぬだろうが
 ポワティエでは下馬騎兵が敵の騎兵に突撃を受け
 アジャンクールの戦いでは完全鎧を着た侭泥沼を移動せねば成らず疲労で撃破される羽目に成った訳だろうか
 (まあ只鎧を外したとしてもロングボウやハルバード等で撃破だろうが))
84名無し三等兵:02/08/14 07:49 ID:???
どうなんでしょう?
85某研究者 ◆TkxmpUgI :02/08/14 07:53 ID:svOhywsw
>まあこれは徒歩での行軍距離が長いので疲労軽減の為に軽装化せざる負えなく成った可能性も有る訳だろうか

これに関しては敵のなるべく手前で下馬すると言う方向も有るだろうが
敵騎兵や弓兵の存在で敵歩兵に(弓に対して脆弱な)馬が接近するのは困難な場合も有ると言う事だろうか


>ポワティエでは下馬騎兵が敵の騎兵に突撃を受け

まあ此れは下馬騎兵がランスやハルバードを持っており
陣形がある程度維持されておれば英騎兵の突撃も阻止出来た可能性も有る訳だろうか
86某研究者 ◆TkxmpUgI :02/08/14 07:59 ID:svOhywsw
>敵騎兵や弓兵の存在で敵歩兵に(弓に対して脆弱な)馬が接近するのは困難な場合も有ると言う事だろうか

まあ只下馬騎兵は鈍足であるから弓兵を包囲するか地形に追い詰めないと
逃げられる可能性も有るのではないのか
(矢張り弓兵を遠巻きに包囲した後で下馬して突撃すると言う方向も
 有るのではないのか(まあ只包囲の為騎兵を分散すれば敵の重装騎兵に
 各個に撃破される危険も有る訳だろうが)
87某研究者 ◆TkxmpUgI :02/08/14 10:47 ID:TlZEeJIS
http://www.hat.com/Othr3/Pivtorak03.html
まあ上の様にハルバードと楯を同時に装備した兵と言うのは
使われた訳なのだろうか
(まあ格闘時にはハルバードは両手持ちであり
 矢やパイクを避ける際に盾を使用すると言う事かも知れないが)
88某研究者 ◆TkxmpUgI :02/08/14 10:52 ID:TlZEeJIS
まあクロスボウ兵と異なり長弓兵には
パビスは装備されない訳なのだろうか
(まあクロスボウ兵や重装歩兵も全てパビスを装備していると言う訳では
 無いと言う事だろうが)
89某研究者 ◆TkxmpUgI :02/08/14 12:52 ID:TlZEeJIS
http://www.medievalzone.com/SEst.htm
まあ攻城塔が有る場合でも梯子は同時に使われていた訳なのだろうか
(攻城塔の方が石での攻撃を受けぬ分安全か共と思うが
 攻城塔の数が無い場合は敵戦力が一点に集結するので
 上がる事は必ずしも容易では無い訳だろうか
 下や塔から援護射撃をすれば城壁からの射撃や石は防げるとの意見も有るだろうが
 盾を構えて下方向に石を落とされる・或いはクロスボウを撃たれる危険も有る訳だろうか
 まあ或いは真下からも援護が可能だろうか)
90名無し三等兵:02/08/14 13:05 ID:???
ライトを激しく点滅させているのは、敵に照準をつけさせにくくする
ためですよね?
http://www.surefire.com/cgi-bin/main.pl?pgm=co_disp&func=displ&sesent=0,0&strfnbr=6&pgname=SFI_mov
91名無し三等兵:02/08/14 13:14 ID:???
>>1-89 名前:某研究者 ◆TkxmpUgI
君は書いている文章に自信が無いのか?
*か*が*ばっかり多用スルな! 

小学校の作文教室からやり直しなさい。
社会人になったらやっていけないよ。
≪ヒッキ−ならば親兄弟の事も少しは考える事≫
92夏厨ですか:02/08/14 13:21 ID:???
>91

・・・あのな。閣下はな。自然現象なんだ。
でもって某不動産会社の御曹司なんだ。
天に唾してもはじまらん
93某研究者 ◆TkxmpUgI :02/08/14 13:26 ID:TlZEeJIS
http://www.medievalzone.com/SEmas.htm
まあ長弓用の銃眼を装備した車装備の盾も有る様だが

http://www.medievalzone.com/SEbst.htm
まあ焼けた攻城塔から援護射撃を行っている物も有る様だが
この程度の損傷であれば未だ城壁に兵を送れるのではないのか(笑
(攻城塔が古代の物の様に皮張りでは無く板張りなのはバリスタを止める為なのか
 (まあ皮より板の方が安価と言う事も有るのかも知れないが))
94某研究者 ◆TkxmpUgI :02/08/15 07:09 ID:IxZuQvSZ
http://www.histomin.com/linepeg/pgpegren.htm
まあランツクネヒトは一部を除いて鎧は装備されている訳なのだろうか
(只軽装歩兵にしては武装は強力で訓練されていると言う事なのか)
中央に重装のピストル騎兵が見えるが
ピストル騎兵はかなり早期に(16世紀に)登場していたと見て良い訳だろうか
(ランスとピストルが同時に装備されている様に見えるが
 ピストルは2丁程度と18世紀の胸甲兵に比べて少なく
 ピストルを放った後でランスチャージを掛けると言う方向なのか)

http://www.histomin.com/linepeg/pgpegcol.htm
17世紀にも完全鎧を装備したピストル騎兵は残っていたのか
まあピストルは依然2丁程度の様でありランスに頼る分も多い訳だろうか
(或いは完全鎧を装備したピストル騎兵は極少数なのだろうか)
95某研究者 ◆TkxmpUgI :02/08/15 07:17 ID:IxZuQvSZ
まあ只馬には鎧は装備されておらぬ所を見ると(或いは鎧を装備しようと)
マスケット銃を装備する歩兵に対して正面から突撃するのは困難だろうか
(しかし全身鎧を利用して下馬しての歩兵への突撃と言うのは
 有り得た訳なのだろうか
 しかしマスケット銃に対しては鎧の胴体は兎も角
 手足の装甲は保つのだろうか)
96某研究者 ◆TkxmpUgI :02/08/15 07:23 ID:IxZuQvSZ
まあパイクマンアーマーを装備したパイク兵の突撃も有り得た訳だろうが
(しかしハルバード兵は装甲は装備しておらぬ訳なのだろうか)
全身鎧では無いので損害は免れ得ぬ訳だろうか
(全身鎧を装備した歩兵は16世紀後半〜17世紀でも残っていたのだろうか
 矢張り残っていたとしても数は少ない訳だろうか)
97某研究者 ◆TkxmpUgI :02/08/15 09:04 ID:ZCq835Bg
http://www.histomin.com/linebor/borcol/pgborcol.htm
まあ槍や全身鎧の無いピストル騎兵も17世紀中盤には
登場していたと言う事だろうか

http://www.histomin.com/lineand/andren/pgandren.htm
まあ上の盾を持ったパイク兵の武器は
パイクでは無いのではないのか(苦笑

http://www.histomin.com/lineben/bencol/pgbencol.htm
しかし18世紀でもハイランダーはこの様な装備なのか(苦笑

http://www.histomin.com/lineben/benmed/pgbenmed.htm
まあ両手用メイスなら盾の上からでも
かなりの打撃を与えられ敵を転倒させる事も出来たと言うのか
98某研究者 ◆TkxmpUgI :02/08/15 13:18 ID:ZCq835Bg
まあ軽歩兵や弓兵の剣は小剣だが
パイク兵や重装歩兵の剣は皆長剣である訳だろうか
(後は精鋭弓兵も長剣を装備している訳だろうか)
只無論西欧の場合副武器が剣とは限らぬ訳だろうか
(片手用のメイスや斧も有る訳だろうが
 軽歩兵や弓兵は小剣で統一されている訳だろうか
 (只小剣がファルシオン等である場合も有る訳なのか))

しかし小剣や長剣とは別に短剣は装備されている訳なのだろうか
(格闘に持ち込まれた場合小剣は兎も角長剣を装備している場合は
 短剣は無いときついのではないのか)
99某研究者 ◆TkxmpUgI :02/08/15 13:59 ID:ZCq835Bg
http://www.histomin.com/linepos/pgposmedeu.htm
マムルークは槍と弓を同時に装備している訳なのだろうか

http://www.histomin.com/linetmm/pgtmmmed.htm
このサラディンは割合重装備の様だが
(まあアラブ軍にも鎖帷子や鱗鎧位有る訳だろうが)

http://www.histomin.com/linever/vermed/mpve0707.jpg
まあこれは短いハルバードを背負っているロングボウ兵の様だが
長剣はロングボウ兵は精鋭部隊であるから装備している訳だろうか

http://www.histomin.com/linever/vermed/mpve1582.jpg
一応長弓兵には短剣も装備されている様だが

http://www.histomin.com/linewhm/pgwhmren.htm
ランツクネヒトの精鋭部隊(ドッペルソルドネル)の装備は
常に胸甲と長剣と言う訳では無い訳だろうか
(矢張りドッペルソルドネル以外でも胸甲は装備されていた訳だろうか)

http://www.histomin.com/linewhm/pgwhmmed.htm
下のイスラム兵の胸部には妙に短剣が多いが
これは基本的に飾りである訳だろうか
100某研究者 ◆TkxmpUgI :02/08/15 15:27 ID:ZCq835Bg
まあハルバードの長さ(3.5m)を考えれば
完全鎧が無く共両手剣を装備したランツクネトが近接戦に持ち込めるチャンスは有る訳だろうか
(まあ只パイク相手では完全鎧(或いは手甲・足甲+盾或は大型の盾か)が無いとかなり困難だろうか)
101名無し三等兵:02/08/15 21:35 ID:+nF5yHS3
怒濤の快進撃だ。
行く手を阻むものは誰もいない……
つーか、何が言いたいんですか閣下。
102名無し三等兵:02/08/15 21:39 ID:???
ある兵種の主武器と副武器との関係の考察と見ていましたが。
103某研究者 ◆TkxmpUgI :02/08/16 17:15 ID:Bk3QAMpi
http://www.nemo.it/soldiers/sa/sa1.htm
http://www.nemo.it/soldiers/sa/sa35.htm
まあ上の様に盾や鎧・服の損傷等が再現されているフィギュアは
余り無い様だが
104某研究者 ◆TkxmpUgI :02/08/17 06:55 ID:QR73Jh7V
http://www.nemo.it/soldiers/SD/SD2.htm
上のファルシオンを持っているのは矢張り軽歩兵或いは弓兵だろうか

http://www.nemo.it/soldiers/SD/SD5.htm
まあこの様な曲がった様な杭も存在するのだろうか


http://www.nemo.it/soldiers/S90/S90-2.htm
まあこの種の角笛は良く用いられたのだろうか


http://www.homestead.com/realminiatures/ORELFIGURESPAGE.html
まあポーランド騎兵の持つランスの形状は
他の地域には無い物だろうか(日本の旗挿し物の構造にも似ているが)
105某研究者 ◆TkxmpUgI :02/08/17 08:56 ID:QR73Jh7V
http://www.michtoy.com/MTSCnewSite/german_scale_folder/ImagesGermanScale/GS_BLegion/BLMebanolKnight%20Diorama.html
まあこれが例の騎士を吊り上げる機械だろうか

http://www.michtoy.com/MTSCnewSite/german_scale_folder/ImagesGermanScale/GS_BLegion/elephant.html
しかし象の牙と言うのはこれ程長かったろうか
(まあ只このカルタゴの象兵を操っているのはインド人なのか
 そうすると象はアフリカ象では無くインド象である訳だろうか)

http://www.michtoy.com/MTSCnewSite/figures_kits_folder/masterclass/MCFG-01.html
しかしこの様な格好の侍は存在したのか(苦笑


http://www.michtoy.com/MTSCnewSite/scenic_diorama_folder/B_Legion_scenics/BL10006.html
このタイプの攻城塔には外部に装甲は無い様だが
どの道盾も無ければ突撃時に撃たれる訳だろうか
(まあ梯子の方は塔の方に伸びているが高い塔は無視して
 通常の壁面に梯子を掛けた方が良いのかも知れぬが)
106某研究者 ◆TkxmpUgI :02/08/17 10:15 ID:QR73Jh7V
http://www.michtoy.com/MTSCnewSite/black_legion_miniatures/SiegeEngines/siege_tower%20/BLCSE1004.html
この大砲の前が装填時に塞がれるのは矢張り防弾の為だろうか
(しかしこの手の物と言うのはどの程度使われたのだろうか
 良く見ると柵にも銃眼の様な穴が空いているが
 これは矢張り銃眼である訳だろうか)


http://www.michtoy.com/MTSCnewSite/black_legion_miniatures/SiegeEngines/siege_tower%20/BLCSE1003.html
この様なタイプの巨大なバリスタは見た事は無いが
通常の弓型の物より威力は高い訳だろうか


http://www.michtoy.com/MTSCnewSite/german_scale_folder/bartel_modelbau/bartel_Turtle/blb11007.html
http://www.michtoy.com/MTSCnewSite/german_scale_folder/bartel_modelbau/bartel_Turtle/blb11004.html
まあ亀甲隊形も後ろ迄には盾は無い様だが
盾のカーブしている部分の隙間と足が弱点だろうか
(しかし良く見ると陣形の前方を傾斜させて足を守っている写真も有るが
 所詮前面のみの事だろうし移動も困難だろうから
 完全に正面や側面を守りたいならパビスの様な巨大な盾を使用した方が良い訳だろうか)
107おた:02/08/17 10:27 ID:???
>>105
すばらしいですな!
これで、めし食わなければ完璧。(象戦車)
サムライもいいが、騎士釣り上げは、笑える。
すごいシロモンだ。
108某研究者 ◆TkxmpUgI :02/08/17 10:39 ID:QR73Jh7V
>これで、めし食わなければ完璧。(象戦車)

まあ矢張りカルタゴ軍の戦象はインド象である訳なのだろうか
(操作している人間や象の頭部の飾りはインドの物かと思うが
 どうだろうか)


>サムライもいいが、騎士釣り上げは、笑える。

まあトーナメントは兎も角実戦でこれが使われた事は
無い様だが
109某研究者 ◆TkxmpUgI :02/08/17 12:15 ID:QR73Jh7V
http://www.michtoy.com/MTSCnewSite/newmetal_folder/Britains/aKnights/8761.html
まあ実際のトーナメント会場はこれ程狭くは無いのではないのか(笑

http://www.michtoy.com/MTSCnewSite/newmetal_folder/tommyA_folder/TommyImages/AC01.html
この象兵も矢張り操作しているのはアフリカ人では無く
インド人だろうか
110某研究者 ◆TkxmpUgI :02/08/17 13:04 ID:QR73Jh7V
http://www.michtoy.com/MTSCnewSite/scenic_diorama_folder/Hudson_Allen_folder/HA_Images_12_00_folder/9650.html
まあこれを落とすのに一体どの程度の兵が必要だろうか
(まあ大型投石器や砲で粉砕する・或いは兵糧責めと言う方向は無しとしても
 これの内部には兵士は100人程度は居る訳だろうか
 矢張り大型投石器使用や兵糧責めなら300名程度で確実に叩けるだろうか)
111某研究者 ◆TkxmpUgI :02/08/17 13:07 ID:QR73Jh7V
矢張り500名も突入させれば大型投石器等を使わず共落とせる可能性は
有る訳だろうか
112某研究者 ◆TkxmpUgI :02/08/17 13:10 ID:QR73Jh7V
上まあ只これに通常常駐しているのは恐らく100名では無く50名程度だろうか
50名程度なら150名程度の兵士でも包囲は可能であり
300名程度であれば突入しても落とせるだろうか
113某研究者 ◆TkxmpUgI :02/08/17 13:14 ID:QR73Jh7V
まあ只其の様な大兵力が来たら逃げるだろうが
中に馬も無い訳なら敵が騎兵や騎馬で行軍する歩兵なら
逃げる事は余程遠くから敵の接近を把握出来ぬ限り無理だろうか
114名無し三等兵:02/08/17 13:37 ID:???
>>110
こういうのはおそらく見晴らしのよい丘の上に建ってるでしょうな。
周囲に大軍を展開するのが困難な場所と思われます(囲まれたらひとたまりもない)
この塔へ至る道は大きくないと思われる
(道がでかい=物資輸送が楽・場所があるから周囲に柵や堀を作れる)
ので、地道に兵糧攻めがいちばんかと。
115某研究者 ◆TkxmpUgI :02/08/17 13:41 ID:QR73Jh7V
http://www.michtoy.com/MTSCnewSite/scenic_diorama_folder/Hudson_Allen_folder/HA_Images_12_00_folder/9513.html
http://www.michtoy.com/MTSCnewSite/scenic_diorama_folder/Hudson_Allen_folder/HA_Images_12_00_folder/9501.html
http://www.michtoy.com/MTSCnewSite/scenic_diorama_folder/Hudson_Allen_folder/HA_Images_12_00_folder/9502.html
西欧にも草葺屋根の家は結構多い様だが
矢張り都市部では無く村等に多い訳だろうか

http://www.michtoy.com/MTSCnewSite/scenic_diorama_folder/Hudson_Allen_folder/HA_Images_12_00_folder/9543.html
この種の塔には梯子を使うより門から突入する訳だろうか
或いは梯子を使い窓から突入すると言う手も有るだろうが
落石も有るだろうし窓が小さい場合突入は困難だろうか
まあ門を破っている間にも上から石が落ちてくる訳だろうし
これに対しても破城槌でも使わないと突入は困難だろうか
(しかしこの地形で破城槌は使えないだろうし
 周囲に盛り土をしてから使うか或いは攻城塔から破城槌を使うしか
 無いだろうか
 攻城塔を使用して窓から突入と言う手も有るだろうが
 塔上部からの落石で攻城塔の橋が破壊される危険も有る訳だろうか
 (破城槌は兎も角攻城塔等持って来るなら
  大型の投石器で塔毎粉砕すると言う手も有る訳だろうが
  (まあこの地形の場合は塔の下を掘って崩すと言う手は使えないだろうが
   下が土の場合は崩され内部の人間も壊滅する可能性も有る訳だろうか
   矢張り城の様に溝を掘ってトンネルを防ぐと言う手も使えないだろうが))
116某研究者 ◆TkxmpUgI :02/08/17 14:12 ID:QR73Jh7V
>こういうのはおそらく見晴らしのよい丘の上に建ってるでしょうな。
>周囲に大軍を展開するのが困難な場所と思われます(囲まれたらひとたまりもない)
>この塔へ至る道は大きくないと思われる
>(道がでかい=物資輸送が楽・場所があるから周囲に柵や堀を作れる)
>ので、地道に兵糧攻めがいちばんかと。

まあそうすると破城槌や投石器が容易に運べない場所に有ると言う事だろうが
岩場の上に立っておらねばトンネルを作成して倒壊させる・岩場の上に立っていた場合は
兵糧責めと言う事だろうか
(まあ只道を広げられれば破城槌や投石器を使用する方が早いかも知れぬが)
117名無し三等兵:02/08/17 14:18 ID:???
>>115
>この種の塔

火や煙でいぶり出すのも有効かと。
あと、水をどこから補給するかが気になりますな。
118名無し三等兵:02/08/17 14:21 ID:???
>>116
>>岩場の上に立っておらねばトンネルを作成して倒壊させる

軟弱な地盤の上に石の塔は築けないかと思われるが……
119某研究者 ◆TkxmpUgI :02/08/17 15:10 ID:QR73Jh7V
>あと、水をどこから補給するかが気になりますな。

これは中に井戸が有るのか雨水でも貯めている訳なのだろうか


>軟弱な地盤の上に石の塔は築けないかと思われるが……

まあ城壁の下にトンネルを掘って倒壊させると言う技も有るから
城と同レベルの高さの塔ならどうだろうか


>火や煙でいぶり出すのも有効かと。

まあ確かに城より規模が小さいので煙を使えば全員を外に出せるだろうか
120某研究者 ◆TkxmpUgI :02/08/17 15:30 ID:QR73Jh7V
http://www.michtoy.com/MTSCnewSite/newplastic_folder/Zvezda/Zvezdaimages/8014.html
まあこのサイズの投石器では城は崩せないだろうか
(まあ只割に薄い銃眼の部分や屋根の上に落下した場合はどうだろうか)
バリスタは火矢を城の内部に打ち込む為に使われる訳だろうか


http://www.michtoy.com/MTSCnewSite/newplastic_folder/Zvezda/Zvezdaimages/8501.html
まあこの手の木の城は投石器や砲で容易に崩されるのではないのか
(或いは焼き弾やギリシャ火等を受ければ延焼は避け得ないだろうか)
まあ大型の城とて大型の投石器や重砲には耐えられないだろうが
大型の砲や投石器の設置には時間が掛かる訳だろうし容易に延焼するのでは問題と言う事だろうか
121おた:02/08/17 15:36 ID:???
>>108インド象
よくは知らないけど、象って、昔は中東に近いあたりまで居たのかなあ?
森林の分布もまったく違ってるはずだから、象の産地分布も違うのかもしれない。
122某研究者 ◆TkxmpUgI :02/08/17 15:45 ID:QR73Jh7V
まあ戦国時代の兜は徒歩武者は筋兜は少数であり
生産性の高い頭型兜が大半であった可能性は無い訳だろうか
(まあ桃型兜等は構造が単純で有っても騎馬武者のみの使用かも知れないが
 只戦国以前は徒歩武者も筋兜しか無い訳だろうが)
胴も徒歩武者は生産性の高い桶側胴が大半であり糸を用いている物は
少数である訳だろうか
まあ騎馬武者も全てが糸を沢山使用している鎧を使っていたかは
疑問な訳だが
123某研究者 ◆TkxmpUgI :02/08/18 14:10 ID:7FDdXlXf
http://www.michtoy.com/MTSCnewSite/newplastic_folder/Russian%20Plastic/russiancatapult.html
しかしかなり小さなカタパルトも有る様だが
射程距離はどの程度なのだろうか
(これを攻城塔や亀甲車・破城槌に命中させても
 破壊出来ぬ可能性は無い訳だろうか)

http://www.michtoy.com/MTSCnewSite/newplastic_folder/PL_marx/Marx54images/acwmortor.html
まあこれは城壁を崩すのでは無く屋根を狙う砲だろうが
内部を延焼させる為に焼き弾が使われた例は有る訳だろうか


http://www.michtoy.com/MTSCnewSite/scenic_diorama_folder/FormTech_folder/FormtechImages/FormTech_by_Barzso/ftbarzso935.html
まあ中世に樽で壁を作る事は行われた訳なのだろうか
(矢張り柵は兎も角土嚢は無い訳だろうが)
124某研究者 ◆TkxmpUgI :02/08/18 17:05 ID:7FDdXlXf
http://www.michtoy.com/MTSCnewSite/figures_kits_folder/St_pete_folder/StPeteImages2002/AGFrench.html
まあアジャンクールで泥塗れに成った仏騎士がやられている場面や
ポワティエで仏の下馬騎兵が英騎兵に突撃を受けている場面の
フィギュア等は無い訳なのだろうか(笑

http://www.michtoy.com/MTSCnewSite/figures_kits_folder/AmberMiniatures/AmberMini_images/I-(C02).html
まあこれはカタパルトでは無くバリスタではないのか(苦笑
(しかし引かれている筈のバリスタの糸が弛んでいるのは何とか成らないのか(笑)

しかし十字軍がアラブ軍に勝っていた部分は
クロスボウの射程しか無い訳だろうか
(槍騎兵は弓で一方的に撃破される有様であり
 突撃時もクロスボウの援護が無ければ不能だろうか)
十字軍が勝利した例はクロスボウで崩壊したアラブ軍歩兵に歩兵が騎兵と共に突入したと言うのが
正解である訳だろうか


http://www.michtoy.com/MTSCnewSite/figures_kits_folder/St_pete_folder/StPeteImages2002/AG6_2.html
まあ仏軍のテントは何となくアラブ的な雰囲気も有るが
http://www.michtoy.com/MTSCnewSite/figures_kits_folder/St_pete_folder/StPeteImages2002/AG6_1.html
まあしかしここからジャン王が連行されている絵が確か有っただろうか(笑

http://www.michtoy.com/MTSCnewSite/figures_kits_folder/St_pete_folder/StPeteImages2002/AG24.html
斧を持っている弓兵等存在したのだろうか(まあアジャンクールで弓兵が仏の下馬騎兵に対して斧を使用したと言う記述も有っただろうか)

http://www.michtoy.com/MTSCnewSite/figures_kits_folder/St_pete_folder/StPeteImages2002/3190_1.html
まあジャン王はポワティエの際下馬戦闘等したのだろうか
(恐らく後続の歩兵に護衛されていたが其処にも背後から騎兵が突撃していた訳だろうか)
125某研究者 ◆TkxmpUgI :02/08/18 18:31 ID:7FDdXlXf
http://www.michtoy.com/MTSCnewSite/figures_kits_folder/St_pete_folder/StPeteImages2002/3159_3160.html
まあこう言う瞬間を捕らえたフィギュアと言うのは矢張り珍しい訳だろうか
(只上に乗ったとしても何処を攻撃するのかだが
 割に防御され易い頭部を狙うより腰のスカートの裏側を突いた例の方が多いのかも知れぬが)


http://www.michtoy.com/MTSCnewSite/figures_kits_folder/St_pete_folder/StPeteImages2002/3918_3.html
まあ下馬戦闘時もランスはこの様な形で使用された訳なのか
(余りマントを装備している騎士は絵には出て来ないが
 王だからと言って装備しているとは限らぬ訳だろうか
 確か黒太子等は無かった様に見受けるが
 矢張り絵の黒太子とフィギュアの其れでは余りに装飾に差が有る様に思えるが)

http://www.michtoy.com/MTSCnewSite/figures_kits_folder/St_pete_folder/StPeteImages2002/3139_1.html
http://www.michtoy.com/MTSCnewSite/figures_kits_folder/St_pete_folder/StPeteImages2002/3139_2.html
まあこの様な飾りを付けた騎士は実際存在したのか(苦笑
絵では頭部の飾りも殆ど房飾りしか無い物が多いが
(侍の奇妙な変わり兜も出て来たのは戦国以降だろうし
 これ等の影響を受けた可能性は無いのだろうか)

http://www.michtoy.com/MTSCnewSite/figures_kits_folder/St_pete_folder/StPeteImages2002/3157_3161_1.html
まあジャン王は絵では王冠では無く帽子を被っていたかと思うが
戦場に王冠等は持ち込まれていた訳なのだろうか


>十字軍が勝利した例はクロスボウで崩壊したアラブ軍歩兵に歩兵が騎兵と共に突入したと言うのが
>正解である訳だろうか

まあ只アラブ側の弓兵が多ければ近接されて弓を連射された場合も有る訳だろうか
126某研究者 ◆TkxmpUgI :02/08/18 18:33 ID:7FDdXlXf
http://www.michtoy.com/MTSCnewSite/figures_kits_folder/St_pete_folder/StPeteImages2002/3704.html
まあしかしブルゴーニュ軍にも長弓兵は存在したのか
(英の傭兵である可能性は無い訳なのだろうか)

http://www.michtoy.com/MTSCnewSite/figures_kits_folder/St_pete_folder/StPeteImages2002/RG4_1.html
http://www.michtoy.com/MTSCnewSite/figures_kits_folder/St_pete_folder/StPeteImages2002/RG4_3.html
まあこの騎士の場合副武装は短剣では無く小剣ではないのか
(1/35にしては出来は良い様に思えるが価格もかなりの物だろうか(笑)

http://www.michtoy.com/MTSCnewSite/figures_kits_folder/St_pete_folder/StPeteImages2002/3703.html
かなり巨大なパビスだが端に射撃用の溝を付ける必要は無く
中央で良い様に思えるが(これは騎士用の盾のランスを置く溝と位置を間違えたのではないのか(苦笑)

http://www.michtoy.com/MTSCnewSite/figures_kits_folder/St_pete_folder/StPeteImages2002/3702.html
しかしこのパビスは矢は兎も角銃弾は止められるのだろうか(笑
(この様な絵の描かれたパビスは大量に使用されていたのか
 博物館等に有る物は騎士の盾も含め割合派手な物が多い様だが)

http://www.michtoy.com/MTSCnewSite/figures_kits_folder/St_pete_folder/StPeteImages2002/5014.html
しかしコルテスは全身鎧は用いなかったと言う事なのか

http://www.michtoy.com/MTSCnewSite/figures_kits_folder/St_pete_folder/StPeteImages2002/RG5_3.html
まあこの馬鎧の前縁部は馬の脚に当たった場合傷付かないのだろうか

http://www.michtoy.com/MTSCnewSite/figures_kits_folder/St_pete_folder/StPeteImages2002/3727.html
ランツクネヒトは両手剣以外にもモーニングスターを使用していたのか(まあ単なる創作である可能性も有るだろうが)

http://www.michtoy.com/MTSCnewSite/figures_kits_folder/St_pete_folder/StPeteImages2002/3749_4.html
まあこの銃を固定する部分も武器と成り得る訳だろうか
127某研究者 ◆TkxmpUgI :02/08/19 14:46 ID:YniDxg9P
http://www.michtoy.com/MTSCnewSite/figures_kits_folder/St_pete_folder/StPeteImages2002/3751_1.html
まあこの様な巨大なメイスと言うのはどの程度使用されたのだろうか


http://www.michtoy.com/MTSCnewSite/figures_kits_folder/St_pete_folder/StPeteImages2002/3755.html
まあ一見不器用か共見えるランツクネヒトに長弓兵等居るのだろうか(笑


http://www.michtoy.com/MTSCnewSite/figures_kits_folder/St_pete_folder/StPeteImages2002/3194_3.html
これは今のスウェーデンの旗と配色は同じ様だが(笑


http://www.michtoy.com/MTSCnewSite/figures_kits_folder/HM_andrea_folder/90mm_white_metal_folder/s8-f27.html
しかしこれはハルバードにしては短いが
ランスとは別にこれは使われたのだろうか


http://www.michtoy.com/MTSCnewSite/figures_kits_folder/HM_andrea_folder/Vingette-folder/sgs04.html
下手をすると上の人間迄振り落とされないか(苦笑
128某研究者 ◆TkxmpUgI :02/08/19 15:56 ID:YniDxg9P
http://www.michtoy.com/MTSCnewSite/figures_kits_folder/HM_andrea_folder/Armies-CharlesV-folder/s2f3.html
しかし近接戦の頻度が低い弓兵を重装化しても無意味かと思うが
(矢を止めるにはパビスでも持たせれば良いのではないのか)
騎馬して行軍するタイプの精鋭弓兵は基本的に鎧は重装なのだろうか
しかし精鋭弓兵は近接戦の訓練等は受けているのだろうか

http://www.michtoy.com/MTSCnewSite/figures_kits_folder/HM_andrea_folder/Armies-CharlesV-folder/s2f5.html
しかしこれはローマ的な鎧にも見えるが実戦では使用されたのだろうか
(中世に作成されたローマ帝国風の兜や鎧が博物館には有る様だが
 装飾も多く実戦で使われた物では無いと言う事なのか)

http://www.michtoy.com/MTSCnewSite/figures_kits_folder/HM_andrea_folder/Armies-CharlesV-folder/s2f7.html
戦場の合図には太鼓以外も使われたのか

http://www.michtoy.com/MTSCnewSite/figures_kits_folder/HM_andrea_folder/medieval-folder/sms03.html
まあこのサイズの投石器でも城壁を破壊する事は困難なのか
或いはほぼ同じ場所に何発も当てた場合は破壊可能な場合も有るのだろうか
(投石器から建物を延焼させる為の焼き弾の様な物が使用された例は有るのだろうか
 まあ炸裂弾や焼夷弾は有る様だが)

http://www.michtoy.com/MTSCnewSite/figures_kits_folder/HM_andrea_folder/medieval-folder/smf05.html
しかし平安時代は兎も角鎌倉時代の大鎧の手甲は
両手に装備されていたのではないのか(笑
129某研究者 ◆TkxmpUgI :02/08/20 15:39 ID:H6qaAvok
http://www.michtoy.com/MTSCnewSite/figures_kits_folder/Pegaso/90mm_folder/peg90009.html
まあこの騎士の装備している剣は片手剣にしてはやや柄が長いので両片手剣だろうか
(両片手剣にしては長さが短いがこれは腰に装備する為だろうか)

http://www.michtoy.com/MTSCnewSite/figures_kits_folder/Pegaso/90mm_folder/peg90009.html
これは浪人故に剣が木製なのだろうか(苦笑

http://www.michtoy.com/MTSCnewSite/figures_kits_folder/posteMilitaria/pmMS_2.html
この加藤清正のフィギュアは割りに良く出来ているのではないのか

130名無し三等兵:02/08/20 15:46 ID:???
>>129の上段と中段のurl同じだYO。
131某研究者 ◆TkxmpUgI :02/08/20 16:10 ID:H6qaAvok
>>129の上段と中段のurl同じだYO。
失礼まあ中断の物のアドレスは残念乍ら忘れたが
余り重要な物では無いので再度探す事も無いだろうか(苦笑


http://www.michtoy.com/MTSCnewSite/figures_kits_folder/Verlinden_folder/120mm/l1256.html
しかしこの様な形で首を輸送していたら
かなりの重量と成る危険も有る訳だろうし武将クラス以外は鼻や耳で代替された可能性は有る訳だろうし
徒歩の兵士は首等複数持ち歩くのは困難ではないのか
(まあ矢張り敵の武将クラスは鼻や耳で済ます事は無い訳だろうが
 武将を複数徒歩の兵士が複数倒す事も余り考えられぬ訳だろうか
 敵の新手等に襲撃された場合は首を持っている徒歩の兵士は孤立する
 或いは騎乗している兵士に首を渡す訳だろうか)


http://www.michtoy.com/MTSCnewSite/figures_kits_folder/Verlinden_folder/120mm/l1287.html
まあこう言う態勢で組み付いても必ずしも勝てるとは限らぬのではないのか(苦笑


http://www.michtoy.com/MTSCnewSite/figures_kits_folder/Verlinden_folder/120mm/l1299.html
この状況では人間は兎も角馬はアウトだろうか


http://www.michtoy.com/MTSCnewSite/figures_kits_folder/Verlinden_folder/120mm/l1313.html
しかしマント装備の歩兵と言うのは絵では余り見ない訳だが
132某研究者 ◆TkxmpUgI :02/08/20 16:41 ID:H6qaAvok
http://www.michtoy.com/MTSCnewSite/figures_kits_folder/Verlinden_folder/120mm/l1403.html
当世具足に太刀は変である訳だろうがこれは絵の影響と見て良い訳だろうか

http://www.michtoy.com/MTSCnewSite/figures_kits_folder/Verlinden_folder/120mm/l1426.html
この様な構え方は実際有る訳なのだろうか
(矢張り相手を倒して上に乗る迄逆手で短剣は扱わないのではないのか
 或いは相手に組み付いた際に首等を狙う為だろうがこれでは短剣での防御は困難だろうか)

http://www.michtoy.com/MTSCnewSite/figures_kits_folder/Warriors_folder/120mm_folder/wa16031.html
副武装が剣とメイス両方有る様だが
短剣+二種類の副武装が装備される事も多い訳だろうか

http://www.michtoy.com/MTSCnewSite/figures_kits_folder/White-Models-folder/1st-Posting-Folder/90-006.html
しかしこんな後にもマムルークは存在した訳かね

http://www.michtoy.com/MTSCnewSite/figures_kits_folder/White-Models-folder/1st-Posting-Folder/90-015.html
この様な手足の肌が露出している様なランツクネヒトは実際有り得たのか
(しかしプレートアーマを着ているのに手足は全く無防備と言う事だろうか
 スカート部分が左右非対称である鎧も余り見ない訳だが)

133某研究者 ◆TkxmpUgI :02/08/21 17:29 ID:/RpQGzEo
http://www.sentinelminiatures.com/index.html
しかし上のTOPは巴御前の様だが(笑
実際この様な格好だったと言う証明は有るのだろうか(苦笑

http://www.sentinelminiatures.com/Page_11_Sentin.html
上の中央に解説も有るだろうが
今後は鶴姫や甲斐姫・小松姫等は出ないのだろうか(笑
134某研究者:02/08/22 12:05 ID:W9vczgLl
http://www.masaaki.com/index.html
上の巴御前のフィギュアを仕上げたのは
どうやら日本人の様だが(笑
135某研究者:02/08/22 14:05 ID:W9vczgLl
http://www.yomiuri.co.jp/04/20020822i202.htm
まあこれで炭疽菌をどの程度防げるのか
(しかし事前に投与せねば余り無効ではないではないのか)
136某研究者 ◆TkxmpUgI :02/08/22 14:13 ID:W9vczgLl
http://space.tin.it/arte/frmaid/Enea/01090035.jpg
まあこの様な状況と成ったら騎兵の負けだろうか(笑

http://space.tin.it/arte/frmaid/i%20polacchi.htm
しかし此所の騎兵は何故ランスや槍を沢山持っている訳なのだろうか
137某研究者 ◆TkxmpUgI :02/08/23 19:39 ID:hLsOkJl+
http://www.cantabrianet.com/detara/pages/celtic_chariot_eng.htm
まあケルト人も一応戦車を用いていた訳だろうか
(しかし同時期のゲルマン人はどうなのだろうか)

http://www.hz-forum.de/images/cocker/Ashigaru.jpg
まあ徒士が太刀を持っているが
矢張りこれは絵の影響なのだろうか

http://www.hz-forum.de/images/cocker/AshigaruBack.jpg
まあ腹当ての後ろの紐が違う様な気もするし
腹当てに肩の上の装甲等無い訳だろうか

http://www.hz-forum.de/images/chitas/Guerras%20Maoris.jpg
まあこの柵の上が尖っているのは上られない様にする為だろうか
(まあ柵が倒されるか尖っている部分を切られる・或いは美味く上られる可能性も
 有る訳だろうか)

http://www.hz-forum.de/images/ebensperger/Attila%203.jpg
まあこれは確かフン族のアッチラかと思ったが
実際にこの様な骸骨の杯を用いていた訳なのだろうか
(ジンギスカンのフィギュアにも確か人骨が大量に有った訳だし
 アッチラやジンギスカンは欧州に取り野蛮な殺戮者と今も映っている訳なのだろうか)
138名無し三等兵:02/08/23 20:03 ID:???
そういや、某研。
「アッティラ」って映画見た? 都市襲うときにカタパルト使ったり
ローマ軍とインチキチャンバラしてて結構おも(略)
139某研究者 ◆TkxmpUgI :02/08/23 20:17 ID:hLsOkJl+
http://www.hz-forum.de/images/ebensperger/Attila%207_3a.jpg
これを見るとアッチラは中国風の鎧を着ている様にも見えるが
これは矢張りフン族が元居た場所が中国に近い故の物なのだろうか
(まあ只これはフン族の遺品や絵を見て作成された物なのだろうか)

http://www.hz-forum.de/images/Gilles/galbes2.jpg
しかし余りこう言う顔の歴史物フィギュアは見ないが(笑


http://www.hz-forum.de/images/farrugia/Spanish%20Armada%20Officer.jpg
しかし船上で無敵艦隊の兵士はプレートアーマー等着ていたのか(苦笑


http://www.hz-forum.de/images/fuetterer/landsknecht.jpg
このランツクネヒトは一体何をしている訳なのか(苦笑


http://www.hz-forum.de/images/Hubbe/Hu3.jpg
まあバイキングも船上で鎖帷子を着ている様だが
これは着用しても泳げる訳なのだろうか
(只北欧の海に落ちて長時間生き延びられる共思えぬ訳だろうが
 直ぐに船上に戻れればどうかだろうが)
140SSBN:02/08/23 21:36 ID:uN5mKixd
>>139
>>ランツクネヒト
ガチョウ(鶏?)もってぐははーっなフィギュアですし。掠奪中とか推測。
フィギュアが全て歴史的事実に基づいているとも限らないけどネ
141某研究者 ◆TkxmpUgI :02/08/23 22:10 ID:hLsOkJl+
http://www.hz-forum.de/images/Hautz/lae03.jpg
まあ胸毛迄再現されているフィギュア等余り見ないが(苦笑
これの装甲を施してある方の腕はまともに曲がる訳なのだろうか

http://www.hz-forum.de/images/ludewig/Falk1.jpg
まあこれは城外の橋なのだろうか

http://www.hz-forum.de/images/ludewig/Turn1.jpg
このジオラマのサイズはどの程度なのだろうか

http://www.hz-forum.de/images/ludewig/Stadt1a.jpg
しかしこれは妙に立体感が無いが

http://www.hz-forum.de/images/ludewig/Max2.jpg
まあ馬上試合には脛部の鎧やガントレット・馬鎧は
無用かも知れないが

http://www.hz-forum.de/images/rohmer/samu%202.jpg
まあこのタイプの鎧の後方は余り見た事が無いが

http://www.hz-forum.de/images/rohmer/samu%201.jpg
しかし胴丸には手甲が両方有るのではないのか(苦笑
142某研究者 ◆TkxmpUgI :02/08/24 14:00 ID:xul1dWf6
http://www.hz-forum.de/images/Schmaeling/bruno_14.jpg
まあ杭と盾を装備した車両が一体化した様な物は
無い訳だろうか

http://www.hz-forum.de/images/Schmaeling/Il%20Conte%20Meno.jpg
しかしこの後ろのライオンは一体何なのだろうか


http://www.michweb.ch/DC/Accueil/buste/imbu/dcb0005.gif
まあこの手の制服はオスマントルコの影響が大きく
帽子はオスマントルコの其れを横向きとしただけであり
(オスマントルコは前に飾りが有り後ろの部分が垂れている訳だろうか)
胸の装飾も類似している訳だろうか
http://www.michtoy.com/MTSCnewSite/figures_kits_folder/St_pete_folder/StPeteImages2002/3173_1.html
http://www.michtoy.com/MTSCnewSite/figures_kits_folder/St_pete_folder/StPeteImages2002/3173_2.html
矢張りトルコが西欧の服をコピーした訳では無く逆と言う事だろうか

http://www.michtoy.com/MTSCnewSite/figures_kits_folder/St_pete_folder/StPeteImages2002/AG6_3.html
テントの形も西欧の其れに似ているが何れが先に作成した訳なのだろうか

http://www.michtoy.com/MTSCnewSite/figures_kits_folder/St_pete_folder/StPeteImages2002/3220_3221.html
オスマントルコ兵も重装備だが鎧は手足以外はプレートでは無い訳だろうか
143某研究者 ◆TkxmpUgI :02/08/24 14:11 ID:xul1dWf6
http://www.michtoy.com/MTSCnewSite/figures_kits_folder/St_pete_folder/StPeteImages2002/3711.html
http://www.michtoy.com/MTSCnewSite/figures_kits_folder/St_pete_folder/StPeteImages2002/3712.html
まあ矢張り胸の部分の飾りが後の西欧の軍服に似ている訳だろうが

http://www.michtoy.com/MTSCnewSite/figures_kits_folder/St_pete_folder/StPeteImages2002/3732.html
しかし胸の辺りで服が分かれているのも西欧に有った様な気もするが
細部の飾りは兎も角少なく共帽子が縦長と成ったのはトルコの影響だろうか
(トルコ軍が赤主体であるから西欧は識別の為に青や白主体の服と成ったと言う事は
 無い訳なのだろうか)
サーベルはダマスカス刀のコピーと言われるが
矢張りトルコの影響が大きい訳だろうか
144某研究者 ◆TkxmpUgI :02/08/24 14:48 ID:xul1dWf6
まあ18−19世紀の軍服の帽子や胸部の飾り・或いは剣もトルコの影響
(まあ腕の部分の袖の形状もそうではないのか)
肩の部分や兜はローマ帝国の影響と言う事だろうか

後西欧の騎士の頭部のリング状の飾りや
羽飾り等はイスラムの影響では無い訳なのだろうか
145某研究者 ◆TkxmpUgI :02/08/24 23:47 ID:1keenDpq
http://www.sankei.co.jp/news/020824/0824boo017.htm
まあ上杉謙信が女だったとの意見も有る様だが
どうなのだろうか(笑
http://ddb.libnet.kulib.kyoto-u.ac.jp/exhibit/ishin/kanren/doc/big/0710071.html
上を見ると髭は無い様にも見えるが
これを以って謙信が女だとの証明とは成らぬ訳だろうか
146某研究者 ◆TkxmpUgI :02/08/25 20:28 ID:sfsY+2HM
http://www.michweb.ch/DC/Accueil/120mm/im120/dc16109.gif
まああまりこの種のポーズの有る騎士のフィギュアは少ない様だが

http://www.michweb.ch/DC/Accueil/120mm/im120/dc16113.gif
余り巨大では無く合成弓でも無いロングボウの射程が長いのは
木の太さと射手の体力故の物と言う事だろうか

http://www.michweb.ch/DC/Accueil/54mm/im54mm/sc54007.gif
まあこれを見ると十字軍が確かに只の蛮族共見えるが
金の有るアラブ王族等はパレスチナを支援する為に
ハリウッドで十字軍の映画等は作らないのだろうか

http://www.michweb.ch/DC/Accueil/54mm/im54mm/sc54008.gif
まあしかしビザンツ帝国軍の衣装がトルコに似ているのか
トルコの衣装がビザンツ帝国に似ているのか何れなのだろうか

http://www.michweb.ch/DC/Accueil/new/dc16180.jpg
http://www.michtoy.com/MTSCnewSite/figures_kits_folder/St_pete_folder/StPeteImages2002/LK14.html
http://www.michtoy.com/MTSCnewSite/figures_kits_folder/St_pete_folder/StPeteImages2002/3728.html
http://www.michtoy.com/MTSCnewSite/figures_kits_folder/St_pete_folder/StPeteImages2002/3729.html
http://www.michtoy.com/MTSCnewSite/figures_kits_folder/St_pete_folder/StPeteImages2002/3749_1.html
http://www.michtoy.com/MTSCnewSite/figures_kits_folder/St_pete_folder/StPeteImages2002/3749_2.html
http://www.michtoy.com/MTSCnewSite/figures_kits_folder/St_pete_folder/StPeteImages2002/3749_4.html
http://www.michtoy.com/MTSCnewSite/figures_kits_folder/St_pete_folder/StPeteImages2002/3756.html
ランツクネヒトのこれはアラブ的な衣装にも見えるが
矢張り其れを参考とした訳なのだろうか
147某研究者 ◆TkxmpUgI :02/08/25 21:50 ID:sfsY+2HM
http://www.elisena.com/mongols1.htm
まあモンゴル騎兵も槍と弓を同時に装備している物が
フィギュアには多いが実際はどうなのだろうか

http://www.elisena.com/mongols2.htm
モンゴルの重装騎兵の馬の頭部の鎧は
西欧の物に似ているが
人間や馬の鎧は中国には無い物だろうか

http://www.elisena.com/Gok.htm
上は槍にフレイルの様な物が付いているが
これは飾りでは無い訳だろうか


148SSBN:02/08/26 00:41 ID:???
>>147
>人間や馬の鎧は中国には無い物だろうか
ありますよ。(゚Д゚)ウマーの鎧も人の鎧も。少なくとも手元にある学研の本によれば。
>槍と弓
実際装備してるんじゃないかなァ。手元のメンアットアームズでも弓付いてるし。
モンゴル軍は訓練かねて狩猟するからそれよう?それとも戦闘で使うのかな?
149某研究者 ◆TkxmpUgI :02/08/26 01:11 ID:KPpHRnDE
>>人間や馬の鎧は中国には無い物だろうか
>ありますよ。(゚Д゚)ウマーの鎧も人の鎧も。少なくとも手元にある学研の本によれば。

まあ鎧の有無では無くモンゴルに似た形式の馬鎧や人間の鎧は
中国には無い物と言う事なのだが
(モンゴルの物は中国の使用している札鎧では無く日本の戦国時代の鎧に近い構造
 (札では無く横長の板を連結した構造)ではないのか)


>>槍と弓
>実際装備してるんじゃないかなァ。手元のメンアットアームズでも弓付いてるし。
>モンゴル軍は訓練かねて狩猟するからそれよう?それとも戦闘で使うのかな?

まあ矢張り弓をある程度放って敵歩兵や騎兵の隊列を崩してから槍で突撃すると言う方向なのだろうか
(或いは槍騎兵に万一踏み込まれた場合でも槍で対応可能と言う事だろうか)
150某研究者 ◆TkxmpUgI :02/08/26 14:22 ID:REguJesn
http://www.999-s.com/prod/12001.html
まあこの形状の兜は実際存在する訳なのだろうか


http://www.999-s.com/prod/pwm02.html
しかしこんな短い旗と言うのは実際有る訳なのだろうか


http://www.999-s.com/prod/pwm06.html
これは剣が左右逆ではないのか(まあ利き手が逆との意見も有るだろうが)

http://www.999-s.com/prod/5401_p.html
長剣にしては柄が長い様な気もするがどうだろうか


http://www.999-s.com/prod/5403_p.html
これは余り見ない形式の肩だが

http://www.999-s.com/prod/5404_p.html
左足にベルトの様な物が付いている様にも見えるが
此れは何である訳なのだろうか

http://www.999-s.com/prod/5405_p.html
これも余り見ないタイプの肩部だが
151AIRSHIP:02/08/28 23:28 ID:0HRIU3n8
>142 バックのライオンは、ベネチアのシンボルである「翼あるライオン」で、
ベネチアの国旗にも描かれている物です。 レパントの海戦の絵にも描かれてい
ます。 イタリアの海軍旗カントンにも入ってます。
152森 雪:02/08/29 00:04 ID:jWoC8KV6
古代くぅ〜ん
153某研究者 ◆TkxmpUgI :02/08/29 16:09 ID:QKkvkxzs
http://www.ttfxmedia.com/beneito/cgi-bin/cesta.asp?p1=06&p2=02&p3=02&p4=TODO&p5=3
まあこれはイスラム教徒を処刑しようとしているのか
拷問しているのか何れなのだろうか
(まさか髪の毛を短剣で切ろうと言うのでは無い訳だろうが(苦笑)


http://www.ttfxmedia.com/beneito/cgi-bin/cesta.asp?p1=06&p2=02&p3=02&p4=TODO&p5=24
上が敵のハルバードを構えたテルシオ等に突入する為の丸盾兵と言う訳では無い訳だろうか
(しかし手足に装甲が無いと盾が有れ突入し難いのではないのか)


まあ防弾装備が全ての部隊に置ける訳でも無いだろうし
狭い道路や橋の前衛のみに配置されたか
開けた場所での野戦でも大将の存在する部隊の
付近のみの配置だろうか
(或いは敵主力が突入して来ると予想される部分辺りに置かれる訳だろうか)
154某研究者 ◆TkxmpUgI :02/08/30 06:21 ID:ZM8bB3y+
http://www.kinghobby.com/prueba/tienda.asp?empresa=3&accion=2&idioma=2&fam=6
上の騎士の武器が鎖で繋がっているのは
アジャンクールで下馬した騎士が泥濘の中に武器を落として
敵軽装歩兵に撃破された故の物なのだろうか
(まあ行軍中は武器を鞘に入れておけば良かったかとは思うが
 或いは戦闘中に落としたと言う事なのだろうか)
或いは馬上で武器を落とせば拾い難い故の事なのか


http://www.krauthauser.com/Html/deutsch/Katalog/Massstab/Texte/54R9.htm
しかし腰に挿すには妙に長い剣に見えるが
下馬戦闘は考慮されている訳なのだろうか(笑
http://www.krauthauser.com/Html/deutsch/Katalog/Massstab/Texte/54R10.htm
上の物と此れは馬の脚に装甲が有る様にも見えるが
どうだろうか

http://www.krauthauser.com/Html/deutsch/Katalog/Massstab/Texte/54R33.htm
http://www.krauthauser.com/Html/deutsch/Katalog/Massstab/Texte/54R34.htm
http://www.krauthauser.com/Html/deutsch/Katalog/Massstab/Texte/54R30.htm
http://www.krauthauser.com/Html/deutsch/Katalog/Massstab/Texte/54R27.htm
http://www.krauthauser.com/Html/deutsch/Katalog/Massstab/Texte/54R36.htm
http://www.krauthauser.com/Html/deutsch/Katalog/Massstab/Texte/54R26.htm
http://www.krauthauser.com/Html/deutsch/Katalog/Massstab/Texte/54R24.htm
これ等は妙に派手な飾りが付いているが博物館では余り見ない訳だが(笑
155某研究者 ◆TkxmpUgI :02/08/30 06:23 ID:ZM8bB3y+
http://www.krauthauser.com/Html/deutsch/Katalog/Massstab/Texte/54R29.htm
まあこの手の鎧の股関節も一応ある程度は動いたのか(笑
(しかし構造が良く解らぬがランス等を接合部に突き込まれると
 脆い訳ならある種欠陥品ではないのか(苦笑)


http://mobry.dyndns.org:81/~bhorling/photos/honeymoon/london2/PB300677-1-0.shtml
このランスは命中すると柄がばらばらに成る様だが
当然未使用であると言う事だろうか(笑


http://www.krauthauser.com/Html/deutsch/Katalog/Massstab/Texte/54L21.htm
ランツクネヒトも単なるツヴァイハンダーのみでは無く
フランベルジュも用いていた訳だろうか
156某研究者 ◆TkxmpUgI :02/08/30 15:42 ID:w33hEdfD
>上の騎士の武器が鎖で繋がっているのは

まあこの構造では敵のランスが鎖に引っ掛かって剣が吹き飛ぶ危険も有るだろうが
其れで馬が傷付く懸念等も有るのではないのか
157某研究者 ◆TkxmpUgI :02/08/30 18:06 ID:w33hEdfD
http://www.krauthauser.com/Html/deutsch/Katalog/Massstab/Texte/54W11.htm
これは鉄甲装備の戦車だろうか

http://www.krauthauser.com/Html/deutsch/Katalog/Massstab/Texte/54G6.htm
http://www.krauthauser.com/Html/deutsch/Katalog/Massstab/Texte/54G12.htm
しかしこの砲に屋根が装備されているのは
雨天でも射撃を可能とする為だろうか

http://www.krauthauser.co/Html/deutsch/Katalog/Massstab/Texte/54W17.htm
まあこれは割に重装備の戦車だが


http://www.krauthauser.com/Html/deutsch/Katalog/Massstab/Texte/541554.htm
しかしこれの腰部の装甲はプレートなのだろうか
158某研究者 ◆TkxmpUgI :02/08/31 19:53 ID:qTfu/bt3
http://www.pegasomodels.com/gallery_details_en.asp?code=m-042
http://www.pegasomodels.com/gallery_details_en.asp?code=m-090
http://www.pegasomodels.com/gallery_details_en.asp?code=m-126
http://www.pegasomodels.com/gallery_details_en.asp?code=m-142
鎧の金属部分が塗られているのは(金・黒以外は)
余り見ないが実際この様な例は有った訳なのだろうか


http://www.pegasomodels.com/gallery_details_en.asp?code=m-083
アジャンクール辺りにしては鎧の形式がやや古い様な気もするが
首部の鎖等は絵では余り描かれておらぬが実際はどうなのだろうか


http://www.pegasomodels.com/gallery_details_en.asp?code=m-069
これは実際戦ったら何れが強い訳だろうか

http://www.pegasomodels.com/gallery_details_en.asp?code=m-081
まあ竹製の矢と言うのは見た事が無いが
この太刀の柄の長さはやや短いのではないのか(笑

http://www.pegasomodels.com/gallery_details_en.asp?code=m-020
まあ足軽は兎も角徒歩武者の篭手には鎖が使われているのではないのか
159某研究者 ◆TkxmpUgI :02/08/31 20:14 ID:qTfu/bt3
まあ足軽は兎も角騎馬武者や徒歩武者の鎧迄貸し具足共思えぬ訳だが
貸し具足は桶側胴が大半である訳なら
足軽の鎧はTV等の様に畳具足が主体では無く桶側胴が主体である可能性も
有る訳だろうか
160某研究者 ◆TkxmpUgI :02/09/02 07:42 ID:dq1NghSt
http://www.pegasomodels.com/details_en.asp?code=54-037
まあこの両手剣は妙に柄が短いが
両手でまともに扱える訳なのだろうか


http://www.pegasomodels.com/details_en.asp?code=54-073
しかしこれでは片手では武器が重過ぎて突き以外困難だろうか

http://www.pegasomodels.com/details_en.asp?code=54-091
しかし胴部に紋章が有る物=貸し具足と言う訳では無いのだろうか
(矢張り徒歩武者レベルで貸し具足は無いと言う事だろうか)


http://www.pegasomodels.com/gallery_details_en.asp?code=m-015
http://www.pegasomodels.com/details_en.asp?code=54-097
まあこんな兜は実際実在したのか


http://www.pegasomodels.com/details_en.asp?code=54-701
しかし兜の上に乗っているのは何人の頭部なのだろうか
このタイプの兜ではランスが命中すると危ないのではないのか
(イラストの剣は木製の様にも見えるがこれは
 試合用の物だろうか)
161某研究者 ◆TkxmpUgI :02/09/02 07:44 ID:dq1NghSt
>(イラストの剣は木製の様にも見えるがこれは
> 試合用の物だろうか)

まあ只試合が全て木製の武器で行われた訳では
無い訳だろうか
162某研究者 ◆TkxmpUgI :02/09/02 22:11 ID:jV1DaiA1
しかしオスマントルコ兵の胸部の飾りは
中国服(まあこれは女真族の服だろうが)の胸部の飾りにも似ているがこれは関連は無い訳なのだろうか
(フン族の時はこの種の飾りは無い訳だろうから
 これが同じ起源の物であるとすれば矢張りトルコが作ったのではなく女真族が考えた物だろうか)
163名無し三等兵:02/09/03 00:50 ID:0QJSHivw
スピアとランスの違いがいまいち分かりません。
164某研究者 ◆TkxmpUgI :02/09/03 07:10 ID:BQbZ8eTL
>スピアとランスの違いがいまいち分かりません。

まあスピアは歩兵用の槍
(まあ3.5mを超える物はサリッサやパイクとして分類される場合も有るだろうが)
ランスは騎兵用の槍と言う事だろうが
(しかしウイングフッサーのランスは特に長い様に見えるが
 重量は問題無かった訳なのだろうか)
165某研究者 ◆TkxmpUgI :02/09/03 07:15 ID:BQbZ8eTL
http://www.pegasomodels.com/details_en.asp?code=90-905
まあこれが以前話した骸骨の沢山置かれているジンギスカンのフィギュアだが

http://www.pegasomodels.com/details_en.asp?code=90-027
まあ西欧のパビスの形状は日本の大型の盾より複雑であり
重量や生産性は問題かも知れぬが
中央が出っ張っているのは白兵戦時の強度を増す為なのだろうか
(絵には側面を防護する様な形状の物も有ったが
 これは歩兵が散開している時は有効であると言う事か)

http://www.pegasomodels.com/details_en.asp?code=75-008
まあこのハンマーも片手で振り回せる様な代物では
(まあ突きのみなら可能かも知れぬが)無い訳だろうが(苦笑

http://www.pegasomodels.com/details_en.asp?code=90-006
まあ15世紀中盤で胸部や肩部に鎧も無いのは
重装歩兵だろうが
しかし百年戦争等の絵を見ると胸部に装甲が無い様に見える騎兵も
存在する様に見えるが
(ジュポンの下にはプレートが有る様だが
 ジュポンの無い場合で胸部に装甲が無い様に見えるのは
 矢張り映画のジャンヌダルクの装備していた鎧の様に
 胸部に装甲は無い訳なのだろうか
 まあ腹部にしか装甲が無いのでは胸部にランスや弓をまともに受ければ
 アウトである訳だろうが(苦笑)
166名無し三等兵:02/09/04 01:19 ID:mH75r7yC
歩兵用に限るなら、スピアとハルバードの違いもいまいちわかりません。
ジャベリンというものも、長物だよね?パイクというものもあるのか。
167名無し三等兵:02/09/04 01:22 ID:mH75r7yC
あと、重装歩兵はどの国のもすべて「ファランクス」と呼ぶんでしょうか。
ギリシャだけでしょうか。
168>167:02/09/04 01:33 ID:???
お前ぜんぜん自分で調べてないだろ。
169某研究者 ◆TkxmpUgI :02/09/04 06:23 ID:nkEwbGDG
>歩兵用に限るなら、スピアとハルバードの違いもいまいちわかりません。

ハルバードはスピアと異なり斧も装備されている訳だが
矢張り余程短い物でも無い限り片手では突きしか出来ぬのではないのか
ハルバード以外のポールアームの時代毎の変化は下に有る様な物だろうか
http://www.ealdormere.sca.org/images/photo/armour_polearm.gif
(上を見る限り長柄斧のバルディチェからハルバードは変化した物と言う事なのだろうか
 ハルバードも15世紀後半辺りにほぼ最後期の物と同じ形状と成っている様だが
 17世紀に斧の形状が通常とは逆に湾曲したのは何故なのだろうか(まあ鎧が廃れた所為も有るのかも知れぬが))
http://www.ealdormere.sca.org/martial_images.shtml
まあ他の武器や鎧の時代毎の変化も上に有る訳だが


>ジャベリンというものも、長物だよね?

ジャベリンと言うのは投げ槍の事だが
古代のピルムやピラ・暗黒時代のランシアやアンゴとは異なる訳だろうか
(何れにせよ暗黒時代以降はクロスボウの登場等で廃れたと言う事だろうか
 まあ蒙古襲来絵巻では投げ槍が用いられている様だが
 これはローマ兵等の様に盾を貫く為と言う事なのか)



>あと、重装歩兵はどの国のもすべて「ファランクス」と呼ぶんでしょうか。
>ギリシャだけでしょうか。

重装歩兵はギリシャの場合ホプリタイだろうし
ファランクスは兵種では無く陣形(歩兵の密集陣か)の名前だが
170某研究者 ◆TkxmpUgI :02/09/04 06:31 ID:nkEwbGDG
>あと、重装歩兵はどの国のもすべて「ファランクス」と呼ぶんでしょうか。
>ギリシャだけでしょうか。

まあマケドニアの場合はペゼタイロイ・中世の場合はmen-at-armsだろうか


http://www.ealdormere.sca.org/images/photo/armour_helmets.gif
まあしかしサレットは割合短い時期(15世紀後半)で
廃れていると言う事なのか

http://www.ealdormere.sca.org/images/photo/armour_armour2.gif
(しかしマクシミリアン鎧が廃れたのはどういう事なのだろうか
 (これはコストの問題なのか火器に対して溝は逆効果と言う事が判明した故なのかも知れぬが))
171名無し三等兵:02/09/04 06:53 ID:???
えっ「ファランクス」って陣形の名前だと今まで思っていた
172名無し三等兵:02/09/04 08:03 ID:???
173名無し三等兵:02/09/04 21:31 ID:???
 ペゼタイロイはマケドニア・ファランクスとも言いますし、
基本的に重装歩兵の密集陣はファランクスで良いと思いますよ。
ローマのレギオンなどは武装も用法も明らかに違いますから、
初期の3列に並ぶもの以外は、マニプレス戦術として区別した
方が良いでしょうが。
174某研究者 ◆TkxmpUgI :02/09/05 12:49 ID:JkV1bpqS
http://www.romeomodels.com/Immagini%20GIF/Prodotti/54mm/54%20JPG/RM%2054-01.JPG
まあ弓兵も鎖帷子の上に皮甲を装備するのは白兵戦の為(矢張り貫撤に弱い鎖を補強か)と言うより
遠距離からの矢を止める為なのだろうか
(皮甲を装備すればクロスボウの350m・或いはロングボウの300mからの射撃程度なら
 止まる或いは致命傷とは成らない訳なのだろうか)

http://www.romeomodels.com/Immagini%20GIF/Prodotti/54mm/54%20JPG/RM%2054-03.JPG
まあ単純な形状のパビスも有る様だが

http://www.romeomodels.com/Immagini%20GIF/Prodotti/54mm/54%20JPG/RM%2054-06.JPG
まあクロスボウを手で持っているのは馬上からの射撃も考慮していると言う事なのか
(矢張り行軍中にクロスボウを態々手で持つ必要性は無い訳だろうが)
175名無し三等兵:02/09/05 22:33 ID:???
エリン以前には戦闘隊形みたいなものは無かったのかな?
176某研究者 ◆TkxmpUgI :02/09/07 11:04 ID:SMRwSCfg
http://www.verlinden-productions.com/vp_htm_1201_1600/vp1552.htm
まあこの様なフィギュアは矢張り胸像としては珍しいのではないのか(苦笑

http://www.verlinden-productions.com/vp_htm_1201_1600/vp1532.htm
まあこれは赤いプレートアーマーと言う事なのか

http://www.verlinden-productions.com/vp_htm_1201_1600/vp1416.htm
まあこれは腕の曲がり方がおかしいのではないのか(苦笑

http://www.verlinden-productions.com/vp_htm_1201_1600/vp1382.htm
まあこれではロシア兵が余りにも哀れだが(苦笑

http://www.verlinden-productions.com/vp_htm_1201_1600/vp1319.htm
これの象頭部の鎧は鉄製では無い訳なのだろうか
177AIRSHIP:02/09/07 21:39 ID:/8+tHEIm
>176 赤いプレートアーマーと言えるかもしれませんが、赤の皮の下に鉄板を
リベットで固定したスケールメールの一種と言って良い物と思います。 実物は、
アヴァロンの一員が着用しているので、次回のブラック・ホールで見ることができ
ると思います。
178某研究者 ◆TkxmpUgI :02/09/08 00:24 ID:7vL3JByx
>赤いプレートアーマーと言えるかもしれませんが、赤の皮の下に鉄板を
>リベットで固定したスケールメールの一種と言って良い物と思います。

まあ只腹部は兎も角胸部にリベットは無い訳だし
胸部はプレートである可能性は有るのではないのか
179某研究者 ◆TkxmpUgI :02/09/13 13:23 ID:Lw6kTia2
まあ城も掘の幅が広い方が良いのか
(まあ掘りの幅が広ければ跳ね橋は使えず
 橋からの進撃を許す訳だろうか)
或いは堀は狭くても跳ね橋を使った方が良いのだろうか
(或いは堀の幅を広くして途中迄は土の橋とし
 終端部分を跳ね橋とすると言う方向も有る訳だろうが)
180某研究者 ◆TkxmpUgI :02/09/13 13:35 ID:Lw6kTia2
まあ只堀も埋められる或いは水を抜かれる
橋を掛けられる等すれば余り役に立たぬ可能性も有る訳だろうし
梯子を阻止する為に堀を作成するのでは無く壁を高くした方が良いとの意見も有る訳だろうか
(まあ只城壁の付近に土を盛られて其処から梯子を掛けられる危険も
 有る訳だろうか)
まあ壁だけでは砲撃で壁や門が粉砕された場合
其処から簡単に乱入される危険は有る訳だろうか
181某研究者 ◆TkxmpUgI :02/09/13 13:39 ID:Lw6kTia2
>まあ壁だけでは砲撃で壁や門が粉砕された場合
>其処から簡単に乱入される危険は有る訳だろうか

まあ堀が無い場合トンネルを掘られて壁を崩される危険も有るだろうが
一応これは迎撃が可能だろうか
182某研究者 ◆TkxmpUgI :02/09/13 13:58 ID:Lw6kTia2
まあ堀の無い山城は城壁を砲等で破壊されると
脆いと言う事は無いのだろうか
(山城には攻城塔は使い難いだろうが
 城壁付近を埋めて梯子を使う・或いは其の侭盛り土を高くして突入する手は有るだろうか
 確か盛り土の下にトンネルを掘って崩すと言う戦法も有ったろうがこれは迎撃可能だろうか)
183名無し三等兵:02/09/13 14:02 ID:???
盛り土の下にトンネルを彫るだけでなく、爆薬入れて、爆破していた。
これぞ元祖地雷。
防御法は、聴音評定で迎撃トンネルを掘って、逆爆破であちらのトンネルをふさいだそうな。

ネタじゃないぞ。
だって某七生女史の「ロードス島攻防記」にそうあった。歴史的資料としての信頼性には、沈黙しておこう。
184某研究者 ◆TkxmpUgI :02/09/13 14:10 ID:Lw6kTia2
>防御法は、聴音評定で迎撃トンネルを掘って、逆爆破であちらのトンネルをふさいだそうな。

まあ矢張りこれは敵のトンネルに横穴を空けて
トンネルを掘っている敵の背後を爆破しようと言う方向だろうか
只敵も同様の迎撃を考え得るなら城の周囲はトンネルだらけと言う事にも
成り得るだろうか(笑
185名無し三等兵:02/09/13 14:16 ID:???
何を言っておられるか。
当時の技術で、野戦構築されるトンネルですぞ。小爆破で、崩してしまうのです。
大規模に崩される前に。
186某研究者 ◆TkxmpUgI :02/09/13 14:22 ID:Lw6kTia2
まあ只トンネルも板や柱で補強されている類の物では
(まあこれを用いて城壁の下を支えてから
 柱に火を付けて城壁を崩壊させる訳だが)
小爆発で崩れる訳なのだろうか
187某研究者 ◆TkxmpUgI :02/09/13 14:58 ID:Lw6kTia2
兎も角利下げ期待での反発も最早無いのだから
米株は嵌め込みでも無ければ重症と見て良い訳だろうか
188名無し三等兵:02/09/13 15:03 ID:???
昔のトンネルなので、城を建設する地点を海際にしたり、城壁の前面に水堀を
設ければ坑道戦法は難しかったと思います。
 爆薬については、日露とかナポレオンにオスマンあたりの戦法ではないかなと
思います。それ以前は坑道に火を放って城壁の土台を崩すという手であったと
思います。
189某研究者 ◆TkxmpUgI :02/09/13 15:14 ID:Lw6kTia2
>昔のトンネルなので、城を建設する地点を海際にしたり、城壁の前面に水堀を
>設ければ坑道戦法は難しかったと思います。

まあ只海に設置しても海上から攻撃を受ける可能性は有る訳だろうし
堀の水を抜かれてから・或いは堀を埋められてから下を掘られる危険等も有った訳だろうか
190名無し三等兵:02/09/13 23:39 ID:???
そもそも古代の城は、地盤沈下を防ぐ為に強固な岩盤に据える事が多いし、
そうで無くても、地下数メートルは城壁が築いて有る物です。>183の地雷に
付いては、元々は単に坑道を支える木を焼き払って地盤を崩す作戦から発達
した物で、ただの坑道作戦よりも効果が高くなりますが、どちらにせよ大規
模な土木工事と、それを成す為の大兵力を充分に動員出来る国でなくては
実現不可能で、中世のヨーロッパでは先ず不可能な話ですね。

191名無し三等兵:02/09/14 09:48 ID:zyXbwTV8
ロードス島は、オスマントルコに攻められていたのだよ、と言ってみる。
192名無し三等兵:02/09/14 11:10 ID:???
城壁をよじ登ってくる敵にウンコを投げる戦法ってのは効いたんだろうか?
命がけで上ってる奴らはそんなモノ関係ないと思うんだが。
193名無し三等兵:02/09/14 12:01 ID:???
>>192
 しかし、病気になるぞ。化膿するぞ。しかも19世紀以前だぞ。
 強力な攻撃ではないかね?
194ハス ◆yPF68LQs :02/09/14 17:01 ID:???
>>194殿
心理的な面もあるけど、やっぱ感染症が一番の効果だろうなあ。
抗生物質なんてない時代だし、戦争中だから細かい傷なんて身体中にあるから、
結構有効だったはず。
195名無し三等兵:02/09/14 23:02 ID:oklqq1TG
>>192
前田慶次も城攻めの時に上から小便を放尿されていましたね
有効でしょう
196ハス ◆yPF68LQs :02/09/14 23:37 ID:???
うげ…自分にレスしちゃった。恥ず…。
197某研究者 ◆TkxmpUgI :02/09/15 10:21 ID:d8Ey3uu6
まあ16世紀にプレートアーマーのスカートが廃れ装甲が脚と一体化したのは
槍等で跳ね上げられて下の(鎖帷子や綿甲しか無い)部分を突かれる事を恐れての物だろうか
(しかしそれで防弾効果は低下した可能性も有るのではないのか
 或いはベルト等でスカートを跳ね上げられぬ様に固定すれば
 問題なかったのかも知れぬが)
しかしパイクマンアーマーでは暫くスカートが残ったのは軽量化の所為だろうか


グスタフ・アドルフの騎兵はサーベルでは無く槍が主兵装の様だが
槍と同時にピストルも装備していた訳だろうか
(この時点では重騎兵もフルプレートは廃れ脚部の装甲はかなり省かれていた訳だろうか)
クロムウェルの鉄騎兵と言うのはは装甲騎兵の事では無く
信仰を軸とした(鉄の意志を持った)戦闘集団の事を指す訳だろうか
(無論装甲騎兵や装甲歩兵も存在はしただろうが)
198名無し三等兵:02/09/15 10:23 ID:???
このスレ読んでて久しぶりにコサックスやりたくなりますた
某研殿はコサックスおやりになるのでしょうか?
199某研究者 ◆TkxmpUgI :02/09/15 10:31 ID:d8Ey3uu6
まあ戦国時代以降の日本の篭手は
博物館等に残っている物を見る限り板金と綿甲のみで無く
全て鎖帷子で補強されている様だが
矢張りドラマ等で出て来る板金と綿甲しか無い物は間違いである訳だろうか
(しかし戦国時代(室町時代後期)からの登場なのか室町前期からの
 登場かは忘れたが)
満智羅や首の部分の(亀甲金を使った)防具もドラマ等では余り使われていないが
実際の使用頻度はどうだった訳なのだろうか
200某研究者 ◆TkxmpUgI :02/09/15 10:47 ID:d8Ey3uu6
ドラマ等では騎馬武者の面具も装備されておらぬがこれも誤りであり
実際は基本的に全て装備されていた訳なのだろうか
(しかし江戸時代の絵を見ると殆ど騎馬武者も面具は装備されていないが)


>某研殿はコサックスおやりになるのでしょうか?

まあ残念乍ら時間が無くて出来ぬが
所詮ゲームでは戦術の幅が限定されると言う事だろうか
201某研究者 ◆TkxmpUgI :02/09/15 14:14 ID:d8Ey3uu6
http://www.rakuten.co.jp/yoroi/118275/119597/
しかし上の篭手の銀の象嵌は西欧甲冑の模倣なのか
銀色の南蛮胴は現存している物は見当たらないが
(矢張り現存している南蛮胴の大半は黒い訳だろうが)
赤く錆びている物は有る訳でありこれ等は元は銀色だった訳なのだろうか
202ハス ◆yPF68LQs :02/09/15 14:53 ID:???
>某研究者殿
篭手についてですけど、布と板金(もしくは硬く煮た皮)だけでなく、タタミ鎧のように
鎖で補強されてるものが正しいはずです。

面頬や喉輪なども原則として騎馬武者(一部の徒歩武者もしてたんじゃないかな?)は
は装着しています。絵画やTVなどで描写がないのは、顔(表情)が分からなくなるから
演出の為、着けていないのです。

あと、自分の知る限りでは、最近の戦国時代もの(映画、ドラマ含め)で装備や戦い方が
リアルなものは皆無ですよ。タタミ鎧が鎖で出来てなかったり、武者ばっかりで
一番数が多かったであろう、中間や小者クラスの戦闘員が出てこないし。
203某研究者 ◆TkxmpUgI :02/09/15 15:15 ID:d8Ey3uu6
>篭手についてですけど、布と板金(もしくは硬く煮た皮)だけでなく、タタミ鎧のように
>鎖で補強されてるものが正しいはずです。

まあ外側或いはほぼ全体が
鎖で覆われている物が戦国時代では大半と言う事だろうか


>面頬や喉輪なども原則として騎馬武者(一部の徒歩武者もしてたんじゃないかな?)は
>は装着しています。絵画やTVなどで描写がないのは、顔(表情)が分からなくなるから
>演出の為、着けていないのです。

まあ只1つ位出来る限りリアリティに徹した物が作れない訳なのだろうか(苦笑


>タタミ鎧が鎖で出来てなかったり、武者ばっかりで0
>一番数が多かったであろう、中間や小者クラスの戦闘員が出てこないし。

まあ畳具足も確かに博物館の物は板金の間が鎖で接合されている様だが(苦笑
足軽の鎧に関しては桶側胴と畳具足が有る様だが
何れが多く用いられた訳なのだろうか
204ハス ◆yPF68LQs :02/09/15 15:44 ID:???
>某研究者殿
タタミ鎧ですけど、実際は足軽等の下級兵士の装備ではなく、武者や武将も装備
してたようです。下級兵士は主に桶側胴だったのでしょう。
タタミ鎧と桶側胴では明らかに桶側胴のほうが手間がかからずに造れますから。
205某研究者 ◆TkxmpUgI :02/09/15 15:50 ID:d8Ey3uu6
>タタミ鎧ですけど、実際は足軽等の下級兵士の装備ではなく、武者や武将も装備
>してたようです。下級兵士は主に桶側胴だったのでしょう。

そうするとTVや映画の足軽の装備は誤りであると言う事だろうか


>タタミ鎧と桶側胴では明らかに桶側胴のほうが手間がかからずに造れますから

まあ鉄の量は桶側胴の方が多い訳だろうが
畳具足は鎖帷子等の手間が掛かる故にコストは桶側胴の方が安い訳だろうか
206某研究者 ◆TkxmpUgI :02/09/15 16:46 ID:d8Ey3uu6
http://www.chiba-muse.or.jp/SONAN/kikaku/yoroi/chaitotatamiP.htm
まあ武将用の畳具足と言うのは上の様な物だろうか


http://www.costumemuseum.jp/collection/j_isho/ie2.html
上に有る畳具足は武将用或いは騎馬武者・徒歩武者用なのだろうか


http://www.city.maizuru.kyoto.jp/kouho/020316/06.html
上の畳具足は民家に有ったと言う事なら
足軽用なのだろうか


http://museum.umic.ueda.nagano.jp/hakubutsukan/story/matsudaira/cont_matsu/doc_matsu/113.html#1
まあこれはかなり後期の畳具足である訳だろうが
207ハス ◆yPF68LQs :02/09/15 17:41 ID:???
>某研究者殿
>まあ鉄の量は桶側胴の方が多い訳だろうが
足軽以下の戦闘員が着けてた桶側胴は、鉄はあまり多く使われて無かったようですよ。
それでも畳鎧よりは多いでしょうけど。ちなみにオレが見たお貸し具足の本物は、
薄い鉄(1MMぐらい)に革と和紙を重ねて造ってありました。
革と和紙、草木布のみで造った桶側胴もざらにあったようですし。

あと、TVや映画などの足軽の装備の話ですけど、あれはなんか綺麗すぎるんですよ。
常備軍として金を掛けて作られていった戦国後期ならともかく(それでも整然としすぎ)
それ以前は格好に統一感はあまりなかったと思うんです。篭手、脛当て含めた具足一式
もってた奴もいれば、胴のみ、腹巻きのみ、篭手のみ、とかの人間もいたそうですから。

あと、腰に手拭いや小刀(俗に守刀とかいわれるようなの)、ナタ、打ち飼い袋(雑納です)
などを思い思いに装備してたとか。
208ハス ◆yPF68LQs :02/09/15 17:57 ID:???
>206
やっぱり、畳鎧といえど、豪華な物から色々ですね。上から2番目のものは良く
分かりませんが、3番目のは確かに足軽(下級武士)のもののようです。
後期の畳鎧は鎖がなくて、布と板金(もしくは革)だけで作ったものも多かった
と聞いた覚えがあります。

あ、ところで、中間、小者は足軽じゃあないですよ。上の書き込みで誤解させたかも
しれませんけど。足軽は一応、武者と同じように“侍”ですから。中間や小者はさらに
その下に仕える、雑用係みたいなものです。それでも戦闘時には足軽と同じくらいの
装備で戦ってたようです。
序列を簡単に書くと、 武者→従者(足軽)→雑用係(中間、小者、下人) です。
209某研究者 ◆TkxmpUgI :02/09/15 18:14 ID:qinuzcDv
>それでも畳鎧よりは多いでしょうけど。ちなみにオレが見たお貸し具足の本物は、
>薄い鉄(1MMぐらい)に革と和紙を重ねて造ってありました。
>革と和紙、草木布のみで造った桶側胴もざらにあったようですし。

まあこんな物では銃弾や弓を止められるのは
下手をすると300m以遠と言う事だろうか(苦笑
(まあ只同時期の西欧の銃兵や弓兵は殆ど皮鎧のみだろうし
 それよりはましである訳だろうが
 槍兵は西欧のパイクマンアーマーの方が鎧の厚みは有る訳だろうか
 只徒歩武者や騎馬武者の鎧は厚さ1mmと言う事は無いかも知れぬが
 (まあ以前調べた貸し具足(桶側胴)の厚みは1.4mm程度だったが))
210某研究者 ◆TkxmpUgI :02/09/16 11:38 ID:rahBfvNR
>薄い鉄(1MMぐらい)に革と和紙を重ねて造ってありました。

まあこれは足軽は綿入れ等は着ないので
衝撃吸収効果も狙っての物だろうか


>只徒歩武者や騎馬武者の鎧は厚さ1mmと言う事は無いかも知れぬが
>(まあ以前調べた貸し具足(桶側胴)の厚みは1.4mm程度だったが))

騎馬武者の鎧には鉄板を2枚重ねた物も有る様だが
これは1.4mm程度の物を二枚重ねたのか或いはもっと厚い物を
2枚重ねた訳なのだろうか
211名無し三等兵:02/09/19 08:57 ID:???
あれ
久々に来てみたら粘着AA消えてるね。
さすが冒険。
粘着も粘着しきれないってか。
212某研究者 ◆TkxmpUgI :02/09/21 18:40 ID:nEWEVvFo
>足軽は一応、武者と同じように“侍”ですから。中間や小者はさらに
>その下に仕える、雑用係みたいなものです。それでも戦闘時には足軽と同じくらいの
>装備で戦ってたようです。

まあそうすると傭兵は兎も角徴用された兵士は足軽では無く
中間や小者なのだろうか(まあ彼等は銃は兎も角刀や槍・弓は扱えるのだろうか)
しかし足軽は侍で有っても兜も被れないと言う事なのだろうか
(中間や小者等は陣笠さえ無いと言う事は無いのだろうか)
213ハス ◆yPF68LQs :02/09/21 21:31 ID:???
>>某研究者 殿

>まあこれは足軽は綿入れ等は着ないので
>衝撃吸収効果も狙っての物だろうか
特にそういった意味はないと思います。鉄より比較的安価にある程度の防御力を
える為だと考えられます。和紙といっても、何枚も重ねて圧縮するように造る
らしいので、革と和紙だけでもある程度の防御力はあったようです。(斬撃くらい
は余裕でしょう)

騎馬武者の鎧についてはちょっとわかりません。でも造るんだったら、1ミリ程度の
物を重ねた方が楽でしょうね。

>徴用された兵士は足軽では無く中間や小者なのだろうか
ここらへんの話はちょっと複雑でして、浅学なオレには説明しづらいのですが、挑戦
してみますね。
まず、戦国後期の常備軍的になってきた頃以前、まだ在地の武士団や山賊等が
傭兵として戦力の中心だった頃を話の前提とします。

この頃は、武士、従者、中間・小者が一つの戦闘単位になって戦っていました。
で、この中で武士―従者、武士―中間・小者の契約(主従)関係があります。

武将が戦争の為に戦力を集めるとき、在地の武士団や山賊等を傭兵として雇う場合、
前述の戦闘単位ごと雇うことが出来ますが、村から徴用した場合、武将直属の配下
に編入することになるでしょう。結果、足軽として従者にされることも、雑用係と
して荷駄隊等になることも、両方あったと思います。



214ハス ◆yPF68LQs :02/09/21 21:48 ID:???
で、そういった傭兵や村から徴用されたもの達の装備ですが、基本的には自前であったよう
です。ですから、装備の良い者、悪い者、色々です。
足軽なのに結構立派な兜だったり、陣笠だったり、無かったり。中間・小者はおっしゃる
通り、陣笠が精一杯でしょうね。兜は身分的に無理だと思います。
あと、武器の扱いですが、戦国時代の村(もしくは荘園)は、独自の戦力を持っていました。
傭兵、山賊のような武装集団や、領地問題で対立する他の村の侵略行為に対抗する為です。

銃が流通してからは、銃を装備してる村も当然あったでしょうし、弓や槍などの武器の扱い
には全然困らなかったでしょうね。
215某研究者 ◆TkxmpUgI :02/09/22 15:37 ID:fHTr9yBc
>で、そういった傭兵や村から徴用されたもの達の装備ですが、基本的には自前であったよう
>です。ですから、装備の良い者、悪い者、色々です。

まあそうすると貸し具足と言うのは侍である足軽以上にしか
支給されないと言う事なのだろうか
216某研究者 ◆TkxmpUgI :02/09/22 15:49 ID:fHTr9yBc
まあ雑兵の敵味方の識別と言うのは個別の兵の旗指し物が無く紋章が鎧に入っていない時代では
旗の有る部隊単位でだろうか(まあ只旗の全く無い相手の敵味方識別はどうするのだろうか)
217某研究者 ◆TkxmpUgI :02/09/22 15:55 ID:fHTr9yBc
まあ旗のみでは無く盾にも紋章が有る場合も有るだろうが
戦国以前に盾に紋章は有ったのだろうか
218某研究者 ◆TkxmpUgI :02/09/22 15:59 ID:fHTr9yBc
西欧の雑兵も個別の兵に旗指し物や紋章が付いていない場合も多いので
識別は旗の有る部隊単位でだろうが(まあ使用言語で敵味方を識別するのも可能だろうが傭兵ではどうだろうか)
パビスや小型の盾等に紋章の有る場合も有るだろうが紋章の無い物も多い訳だろうか
219名無し三等兵:02/09/23 03:24 ID:???
某研氏、或いはここのスレッドに興味を持たれる人たちが
ゲームを好むかは知らぬが、城攻・城塞都市建築をテーマとした
シミュレーションゲームがあるので紹介しておく
ゲーム故かなりデフォメルされているが、なかなか良くできている
ように思う

・STRONG HOLD
・CRUSADER(STRONG HOLDの続編)

製造メーカーOHP
http://www.stronghold-game.com/
220名無し三等兵:02/09/25 02:43 ID:???
誰か日本語ないし英文サイトでフリドリッヒ大王の軍隊について
まとめたサイトご存知ないですかね?

7年戦争とかロスバッハの戦いとか、あるいは諸兵科の装備や制服について
調べたいです
221某研究者 ◆TkxmpUgI :02/09/25 02:57 ID:e2MYYZws
http://plaza4.mbn.or.jp/~tactic/18.html
まあ上のサイトの七年戦争の部分は未だ未完成の様だが
今はFOMC街で単に起きているだけであり
脳内は仮眠状態であるから昼以降として貰いたい訳だが(苦笑
222名無し三等兵:02/09/25 03:19 ID:???
>220
洋書ならオスプレイのmen at arms シリーズかな。
たしかこれは翻訳版が出てないはず
TTP://www.ospreypublishing.com/title_detail.php/title=Q1515~ser=MAA~per=12

それにしても某研氏から回答レスをもらえるとは・・・うらやま
223名無し三等兵:02/09/25 03:45 ID:???
>>222
某研猊下はけっこうレスをしてくれるような気が。
そういや上の方にあった中世の決闘の絵を集めてたホムペが無くなってるような。
224某研究者:02/09/25 19:08 ID:XrnJ4l6f
まあフリードリヒも前半は規律重視・後半は火力を重視したと言う方向性で
良い訳だろうか(まあ新たな陣形も何か用いていたかと思ったが
これは後で確認したいと思うが)
225220:02/09/26 02:28 ID:???
猊下はじめ皆様の即レスいただき感謝に耐えませぬ。ありがとうございます

>>221
猊下ご教示の「戦術の世界史」および「古代戦の戦術」
ttp://ha6.seikyou.ne.jp/home/Bf109G/sub11ksenzyutu.htm
は臣も愛読しております。
やはりフリドリヒ大王の有名な斜行戦術および制服や編成について
ネットではなかなかみつからないというのが残念ですね。
きのう質問直後、frederick king of the greatで探しまくったのですが
英文サイトではなかなかないようです。prussiaとかドイツ語よめれば
ありそうなのですが、なかなか・・・
226220:02/09/26 02:36 ID:???
>>222
オスプレイのフレデリック大王シリーズの総論本ですね。
以下に1 cavalry 2 infantry(邦訳あり)3specialist troops
があって、このシリーズ猛烈に欲しいのですねえ

amazonのE-BOOKsで中古本漁ってみましょうか。
上手くすれば輸送料こみで1冊10$で船便(50日待ち)
定価15$+運賃5$でamazon在庫、航空便(1week)ですが
すると西山書店で2500円の在庫買ったほうが早いかなあ・・・
227某研究者:02/09/26 06:42 ID:JWy6tAs/
http://markun.cs.shinshu-u.ac.jp/~ryam/jinkei/jinkei8.htm
まあ上がフリードリヒの陣形の特徴だが
最小単位が通常より分割された斜線陣を使用していると言う事だろうか
228某研究者:02/09/26 06:55 ID:JWy6tAs/
まあ只イエナ−アウエルシュタットの戦いでプロイセンの斜線陣は
仏の縦隊突撃に敗れている訳だが(苦笑
(まあ要するに仏軍=レギオンでありプロイセン=斜線陣だろうか)
229某研究者:02/09/26 07:20 ID:JWy6tAs/
http://markun.cs.shinshu-u.ac.jp/~ryam/jinkei/jinkei9.htm
ナポレオンの旅団では2つの縦隊が防御時は旅団側面を防護し
攻撃時は敵部隊の結節点に向けて突入すると言う事だろうか
230某研究者:02/09/26 08:05 ID:JWy6tAs/
まあ同数の兵力であれば大砲の数の差と言うのは
プロイセン軍と仏軍では有った訳なのだろうか
(無いとすれば同数の兵力で破れた場合は火力では無く散兵と縦隊突撃で
 プロイセン軍は壊滅したと言う事なのか)
231SSBN:02/09/27 12:03 ID:???
フリードリッヒの斜線陣って戦術機動で砲弾飛び交うなか敵歩兵横隊の横を付くための機動じゃなかったでしょうか。
あと厳格に訓練した正規歩兵で素早く陣形転換とかでしたか。
>>230
その時期のフランスの砲っていい物で射程等で諸外国に勝っていたとどこかでみたような記憶が。
232東郷ターン:02/09/27 13:37 ID:uaR9M6YM
単縦進ですすむロシア艦隊の先頭を、日本艦隊がTの字戦法で横隊を構成し
片舷射撃で圧倒的な火力比(10:1)で撃沈するようなもの?

>フリドリヒ大王の斜行戦術(Oblique order)の目的
233名無し三等兵:02/09/27 15:44 ID:???
>>232
(メモ帳か、なんかに移すなり、等幅フォントで見ること
                /
↓――――――――――――――/
敵の正面          /
             /
            /
           /
というような感じで、敵の片翼に攻勢を集中させていく感じ
234某研究者 ◆TkxmpUgI :02/09/27 21:30 ID:tjp6xQQs
まあそうすると仏兵も斜線陣に対して縦隊突撃をした訳だろうが
敵の結節点への突撃と言うのは敵部隊の隙間へなのか
或いは指揮官の存在する場所に突撃すると言う事なのだろうか

斜線陣でも仏陣形の横を突くのは困難だった
或いは側面も仏陣形は上手く防護したと言う事なのだろうか
(仏の散兵で足止めされている間に陣形を変更され縦隊突撃を受けた可能性も有るだろうか)
235某研究者 ◆TkxmpUgI :02/09/27 21:35 ID:tjp6xQQs
>というような感じで、敵の片翼に攻勢を集中させていく感じ

まあ只重装歩兵斜線陣の様に自軍の片翼にも攻撃が集中する懸念が有るだろうが
砲兵等の集中配備で対応する訳だろうか
(まあ敵も同様の斜線陣を用いて来ればアウトだろうか)

236某研究者 ◆TkxmpUgI :02/09/27 21:39 ID:tjp6xQQs
>(まあ敵も同様の斜線陣を用いて来ればアウトだろうか)

まあ斜線陣の場合殆ど一箇所のみへの突撃なので
敵も其処への砲兵の集中配備等で対応し易いが
仏兵の場合は複数箇所を縦隊で攻撃し得るので
敵の砲兵の穴を突き得ると言う事だろうか
237某研究者 ◆TkxmpUgI :02/09/27 22:04 ID:tjp6xQQs
斜線陣や横隊とナポレオン軍の様な少数精鋭部隊は
同じコストであれば何れが強いのだろうか
まあ或いは同じコストのテルシオとマウリッツ型何れが強いのかと言う事だが
238某研究者 ◆TkxmpUgI :02/09/27 22:53 ID:tjp6xQQs
まあ横隊や斜線陣よりレギオンの方が
地形を利用出来ると言う事だろうが
(まあ地形を利用したレギオンの陽動でナポレオンに各個撃破された例も多いと言う事だろうか)
平地での物量戦に持ち込まれれば同等コストの横隊や斜線陣に対して
確実に勝てると言える訳だろうか
239某研究者 ◆TkxmpUgI :02/09/27 23:12 ID:tjp6xQQs
http://www6.tok2.com/home/temjin/dearnapoleon.html
ナポレオンも散兵を用いたのは古代の戦術の摸倣ではなく
米独立戦争時の米軍の狙撃兵を参考にした訳だろうか
上を見る限り矢張り地形の利用を主に考えてレギオンの様な陣形を
使用したと言う事なのか
(仏人にはプロイセンの様な規律は真似出来なかったと言う記述も有る様だが
 これは独仏のサッカー等にも通じる事だろうか(苦笑)
240某研究者 ◆TkxmpUgI :02/09/27 23:39 ID:tjp6xQQs
まあ騎兵に対抗させる為に敵に密集方陣を組ませ
其処に砲を撃ち込んで大被害を与えると言う戦術も
有り得た訳だろうか
(まあ銃剣装備の兵のみではなくハルバード兵やパイク兵が存在すれば中空の方陣でも
 敵騎兵を阻止出来ただろうか)
241某研究者 ◆TkxmpUgI :02/09/27 23:45 ID:tjp6xQQs
まあ上は自軍の騎兵が利用可能である事が必要だろうが
敵の騎兵より自軍の騎兵の数或いは能力が若干上回っていれば
ある程度騎兵を歩兵に突撃させる事は可能だっただろうか
(敵歩兵も陣形を直ぐに変更出来ぬ訳なら
 自軍の騎兵の存在を匂わせるだけで密集隊形にさせ得ただろうか)

まあしかし仏軍の散兵が騎兵に各個撃破された例は有るのだろうか
(まあ騎兵が接近した場合は散兵も後退して後方の歩兵と方陣を組んだだろうか)


242某研究者 ◆TkxmpUgI :02/09/27 23:48 ID:tjp6xQQs
まあしかし指揮官の居る周辺は騎兵の奇襲攻撃に備えて
常に密集方陣と言う事は無いのだろうか
243名無し三等兵:02/09/28 00:40 ID:???
>>229
そのHPレギオンが第二次ポエニ戦争で敗れて衰退したとか言ってるぞ。
244某研究者 ◆TkxmpUgI :02/09/28 01:16 ID:emKUZksH
http://www.asahi-net.or.jp/~uq9h-mzgc/keitai1.html
米軍ではなくオーストリア軍が散兵戦を古代以降では一番先に使用した様だが
米軍はこれを模倣したのではなく独自の考案である訳だろうか

>フランス革命戦争においても散兵戦術はしばしば採用された。祖国を守る熱意溢れた兵隊たちは、むりに隊形に縛り付けなくても脱走することはない。
>むしろ、彼らの積極性を戦場で活用した方がいいとの判断から、フランス軍は積極的に散兵戦術を採用するようになった。
>他国にも散開して戦う兵隊はいた。だが、フランス軍はどの列強よりも大規模な散兵を投入できた。君主の軍隊でなく国民の軍隊だったからこそ可能な戦法だった。

まあ仏或いは米国並みに散兵を使用する為に各国は民主化を加速したと言う事も
有るのかも知れないが


>白兵戦が中心なら射撃に慣れていない兵たちでもかなり戦える。

まあ只これは局所的にも味方の物量が多ければと言う条件付きだろうが(苦笑
故に雑兵を縦隊とし精鋭部隊を横隊或いは散兵として使用したと言う事だろうか
245某研究者 ◆TkxmpUgI :02/09/28 01:29 ID:emKUZksH
>深さ3列から4列程度で、横に細長く伸びたこの隊形は、17世紀のグスタフ=アドルフのころから広く使われるようになり、18世紀に欧州ではほぼ完成を見た。

まあ密集方陣を組まなく共3列から4列程度の隊形を維持できておれば
騎兵突撃は困難だったと言う事だろうか
(まあ只銃撃や砲撃で隊列が薄く成れば(ハルバード兵やパイク兵の1列でも存在しない限り)
 其処へ騎兵突撃を受け得ただろうか)
246某研究者 ◆TkxmpUgI :02/09/28 01:36 ID:emKUZksH
>(まあ只銃撃や砲撃で隊列が薄く成れば(ハルバード兵やパイク兵の1列でも存在しない限り)
> 其処へ騎兵突撃を受け得ただろうか)

矢張り穴が空いた部分は隊列が素早く横に間隔を詰めて
埋めていた訳だろうか(まあこの移動で部隊の弾薬装填がやや遅れる可能性も有るだろうが)

まあ騎兵は兎も角歩兵の縦隊突撃には歩兵が3列・4列有ろうと
突破されていた訳だろうか


>故に雑兵を縦隊とし精鋭部隊を横隊或いは散兵として使用したと言う事だろうか

まあしかし雑兵である縦隊が敵横隊や騎兵に横撃された場合は
やはり銃撃能力や隊列を埋める能力が低いので
脆かった訳だろうか
247某研究者 ◆TkxmpUgI :02/09/28 01:43 ID:emKUZksH
訓練を受けていない縦隊の機動力も横方向には低い訳だろうし
矢張り其処を攻撃されると脆い訳だろうか
248某研究者 ◆TkxmpUgI :02/09/28 01:56 ID:emKUZksH
まあしかし白兵戦主体の雑兵の縦隊も一応銃は持っていた訳だろうか

http://www.asahi-net.or.jp/~uq9h-mzgc/reality.html
散兵もエリート部隊と雑兵が2種類存在し
全員が散兵と成る事も有った様だが
249某研究者 ◆TkxmpUgI :02/09/28 02:14 ID:emKUZksH
>そのHPレギオンが第二次ポエニ戦争で敗れて衰退したとか言ってるぞ。

まあしかしハンニバルを倒したスキピオはレギオンでは無い陣形を
用いていたのだろうか
(まあしかしハンニバルの陣形と言うのはレギオンとは異なっていた訳なのだろうか)
250通行人:02/09/28 02:16 ID:???
>248
猊下、あいかわらずお元気でなによりです。ご無沙汰しております
とおもって覗いたら・・・

お。リンク先はR/DさんのLa patrie en danger(祖国は危機にあり!)ですな。
このひと例のNapoleonic!日本語サイトで、元傭兵の柘植さんがかいた
一連のナポレオンもの仮想戦記に「なんとか合理的に辻褄合わせをする」いう
冷徹かつ爆笑ものの共同企画を執筆してますねえ。

アマルガムとかdemi-brigadeとか近代の兵制ファンには避けてとおれぬサイトですね
251はじめてきました:02/09/28 02:24 ID:lbFvPJ+T
初めてここにきたものです。
お聞きしたいのですが、日本にも手で持って使う盾はあるのでしょうか?
252某研究者 ◆TkxmpUgI :02/09/28 02:41 ID:emKUZksH
>お聞きしたいのですが、日本にも手で持って使う盾はあるのでしょうか


まあ小型の手盾と言うのも有る様だが置いて使うタイプの盾も
一応手に持って移動は可能だろうが


まあ中世の軍隊は縦6列や10列の陣形を用いてるが
3列辺りが主体の近世の軍隊に比べ
砲撃に対して脆いと言う事は無いのだろうか
(まあ槍兵のみ6列も並べる意味は果たして有るのかだろうが
 陣形が無用に横に長く伸びるのを避けたと言う事だろうか)
253某研究者 ◆TkxmpUgI :02/09/28 02:52 ID:emKUZksH
http://markun.cs.shinshu-u.ac.jp/~ryam/jinkei/jinkei7.htm
グスタフ・アドルフの軍にはホイールロック銃部隊が多いが
コストの割に発射速度は速いのだろうか
(まあしかし仮に倍のコストとしても発射速度は倍には成らないだろうが)
254某研究者 ◆TkxmpUgI :02/09/28 02:59 ID:emKUZksH
高価なピストルを持たない竜騎兵の大量配備と言うのも
意味は有ったかも知れぬが
(まあ戦列歩兵が乗馬して・或いは馬車等で移動する事も
 多かった訳だろうか)
まあ所詮雑兵である竜騎兵等より軽装槍騎兵に重装騎兵を阻止させる為にピストルを持たせた方が
良かったかも知れぬが(まあ只軽装槍騎兵にもピストルは有ったのだったか)
255某研究者 ◆TkxmpUgI :02/09/28 07:15 ID:K4Gkxkpj
>散兵もエリート部隊と雑兵が2種類存在し
>全員が散兵と成る事も有った様だが

まあ騎兵にも騎兵銃を使用する散兵である猟騎兵と言うのが存在した様だが
胸甲兵や他のピストルを持つ騎兵も散兵と成った訳なのだろうか
騎銃兵(カラビニエール)は胸甲を装備したと言うが
胸甲兵も後にカービン銃を装備している訳だろうし
両者の違いは基本的に地位や白兵戦能力の差と言う事なのだろうか
256某研究者 ◆TkxmpUgI :02/09/28 07:29 ID:K4Gkxkpj
>まあ騎兵にも騎兵銃を使用する散兵である猟騎兵と言うのが存在した様だが
>胸甲兵や他のピストルを持つ騎兵も散兵と成った訳なのだろうか

騎兵は歩兵に比べ仏以外の国も散兵と成り易いだろうが
ピストルしか持たない騎兵が歩兵に対し散兵戦を行っても
有効打を与えられただろうか(まあカービン銃ならどうかだろうが)
まあしかし槍騎兵はピストルは兎も角カービン銃は持たなかったのだろうか

騎銃兵で胸甲兵や軽装槍騎兵をアウトレンジすると言う方向も有るだろうが
敵の騎銃兵が随伴していた場合は無理だろうし
後期の胸甲騎兵にはカービン銃が装備されている訳だろうか
(まあしかしカービン銃と言うのは移動中は装填出来ぬ訳なら
 装填中に踏み込まれてやられる可能性も有る訳だろうか
 (まあ胸甲兵もピストルを複数持つなら騎銃兵に撃たれた後でも
  追撃してピストルを放てるだろうか
  まあ騎銃兵が逃げ続ければ別だろうが
  騎銃兵は自衛用にピストル等は持たないのだろうか))
257某研究者 ◆TkxmpUgI :02/09/28 07:34 ID:K4Gkxkpj
>まあ騎兵にも騎兵銃を使用する散兵である猟騎兵と言うのが存在した様だが
>胸甲兵や他のピストルを持つ騎兵も散兵と成った訳なのだろうか

まあ当然騎銃兵も散兵と成った訳だろうが
猟騎兵の方が射撃の腕前は上である訳だろうか
(地上の精鋭散兵同様ライフル式の銃を装備している可能性も有るだろうか
 矢張り騎兵銃や精鋭部隊の持つ銃は基本的にホイールロック式なのだろうか)
258某研究者 ◆TkxmpUgI :02/09/28 07:59 ID:K4Gkxkpj
http://w3.shinkigensha.co.jp/book_naiyo/4-88317-279-1p.html
まあ騎兵銃が登場したのは16世紀中盤からの様だが
槍騎兵に迄装備されていたのだろうか


>猟騎兵の方が射撃の腕前は上である訳だろうか
>(地上の精鋭散兵同様ライフル式の銃を装備している可能性も有るだろうか

まあ火打石の衝撃で精度が悪く成るフリントロック式を
精度を要求する為のライフル式の銃に用いる訳も無いと言う事だろうか(苦笑
259某研究者 ◆TkxmpUgI :02/09/28 08:24 ID:K4Gkxkpj
http://www.vmnf.civilization.ca/vmnf/expos/champlain/guer4_en.html
矢張りホイールロックのカービン銃も有る様だが
マスケット銃に比べ銃身はかなり短いのではないのか
(まあ只これより長い物も有るかも知れないが)
カービン銃に銃剣を装備してもサーベルと長さは大差無いので
矢張り行われなかった訳だろうか(苦笑
260某研究者 ◆TkxmpUgI :02/09/28 09:47 ID:K4Gkxkpj
http://www.bohebohe.com/other_halfarmor.htm
まあ上の鎧の厚みは5cmでは無く
5mmの間違いだろうが(苦笑
(まあ5mmでも近距離では銃弾には貫かれると思うが
 どうだろうか)

http://www.btinternet.com/~thporter.antiques/Photolink82.htm
まあこれを見る限り兜の厚みは5mm等はとても無いだろうが
弓や銃弾は相当程度遠距離で無ければ貫かれるだろうか
261某研究者 ◆TkxmpUgI :02/09/28 09:59 ID:K4Gkxkpj
http://www.btinternet.com/~thporter.antiques/Photolink82.htm
>まあこれを見る限り兜の厚みは5mm等はとても無いだろうが
>弓や銃弾は相当程度遠距離で無ければ貫かれるだろうか

しかし良く見ると厚さは枠の部分以外は1mm程度も無い様に見えるが
どうだろうか(苦笑
隙間を作ってでやり過ごすとしても弓は兎も角銃弾相手では
アウトだろうか(苦笑

http://www.btinternet.com/~thporter.antiques/Photolink35.htm
日本の兜に比べ中がやや雑に見えるが(苦笑
西欧の兜に内張りは無いのだろうか
まあしかしバイザーの内側にも枠が有るのは
バイザーを開放した状態で白兵戦をする事も考えていたと言う事だろうか
262某研究者 ◆TkxmpUgI :02/09/28 10:27 ID:K4Gkxkpj
http://www.btinternet.com/~thporter.antiques/Photolink25.htm
まあ上の厚みも5mm等は無い様に思えるが
良く見るとスカート部分はベルトでは無く鉄製の蝶番で固定されている様だが
スカート中央部の隙間も良く見るとスカートが重なっており
直立時は隙間が無い構造と言う事だろうか
263某研究者 ◆TkxmpUgI :02/09/29 22:48 ID:nFJE7TkY
まあしかし3列の歩兵にも槍騎兵なら(相手にハルバードやパイクが無ければ)
突撃可能である訳だろうか
(まあ只銃撃を封じないと突撃は無理だろうが
 散開しての突撃ならどうだろうか
 矢張り弾薬が尽きないと現実的には無理だろうか)
まあ1列の歩兵にも槍の無い重騎兵では突撃し難いと言う事は有るだろうし
槍騎兵の方が確実に仕留められる訳だろうか


仏の槍騎兵はプロイセンやロシア・ポーランドの其れより弱いと言うが
プロイセンやロシアの槍騎兵も仏重騎兵のピストルやカラビニエールに撃退されていたと言う事は
無いのだろうか
(まあ只双方のカラビニエールやピストルの銃弾が消耗した後は
 騎兵の突撃戦と成り得るだろうし其の場合仏の騎兵は
 プロイセンやロシアの騎兵に同数では(槍騎兵の能力の差で)やられていたと言う事は
 無いのだろうか)
264某研究者 ◆TkxmpUgI :02/09/30 01:10 ID:uv4iX9Aa
まあグスタフ・アドルフは騎兵からカービン銃を捨てさせ
サーベルに持ち替えさせたと言う事だろうが
其れ程騎銃兵と言うのは使えない訳なのだろうか
(騎兵は兎も角歩兵に対しては騎銃兵は役に立たないと言う事だろうか)
グスタフ・アドルフも騎銃兵を全廃した訳では無いだろうが
サーベルでは無く槍を持たせると言う発想は無かったのだろうか
(槍兵ではピストルが使えないと言う事かも知れぬが
 槍とピストル双方を携帯している騎兵のフィギュアと言うのも有った訳だが)
265某研究者 ◆TkxmpUgI :02/09/30 01:16 ID:uv4iX9Aa
まあ矢張り騎兵銃より射程の長い竜騎兵を下馬させて
カラビニエールをアウトレンジすると言う方向も有る訳だろうが
(矢張りカラビニエール等より竜騎兵の方が安価であり
 数を揃えられる訳だろうか)
266某研究者 ◆TkxmpUgI :02/09/30 01:34 ID:uv4iX9Aa
竜騎兵の使うドラグーンマスケットは確か歩兵のマスケットより
短かったと思うが
乗馬して射撃するのでも無ければ短くする意味は無く
下手をすると歩兵のマスケットに対抗不能と成るのではないのか
(まあマスケット銃はかなり近接してから発射するのであれば射程は無意味と言う意見も有るだろうが
 そうであれば安価で装填速度の速い(非ホイールロックの)ピストル(これは多銃身でも良いだろうが)と
 短槍でも装備すれば良いとの意見も有るだろうが)

まあ矢張りカービン銃よりはドラグーンマスケットの方が
銃身は長い訳だろうか
267某研究者 ◆TkxmpUgI :02/09/30 08:04 ID:JZfRjKas
http://www.sanynet.ne.jp/~imperial/Napoleon/yougo02.html#ri
竜騎兵はどうやら馬上からの銃撃能力が有る様だが
(後期には下馬戦闘はしなかった様だが)
速射能力はドラグーンマスケットはピストルやカービンには劣る訳だろうか
まあ通常のマスケット銃を持たせて下馬戦闘し竜騎兵をアウトレンジする言う方向も有る訳だろうが
其れ程の射程の差は無いと言う事だろうか


http://www.sanynet.ne.jp/~imperial/Napoleon/yougo00.html#ki
カラビニエは単なる騎銃兵と言うよりは
エリート騎兵の事を指す訳だろうか
(この時代の騎銃兵は竜騎兵の事だろうか)
268某研究者 ◆TkxmpUgI :02/09/30 08:14 ID:JZfRjKas
http://www.sanynet.ne.jp/~imperial/Fred/fred-h.html
まあ以前質問されたフリードリヒ大王に関するサイトだが
上の完成を待つと言う手も有るだろうか
269某研究者 ◆TkxmpUgI :02/09/30 08:21 ID:JZfRjKas
http://www.sanynet.ne.jp/~imperial/Napoleon/weapon2.html#r
まあどうやらライフル銃はホイールロックでは無い様だが
火打石の反動は問題では無いのだろうか
(しかし装填時間が非ライフル銃の2倍で命中率が1.5倍では
 正直役に立たぬ武器だろうか)
270某研究者 ◆TkxmpUgI :02/09/30 08:32 ID:JZfRjKas
槍騎兵にはどうせ銃剣では歯が立たぬ訳だろうし
槍歩兵+短く装填速度の速い小銃を装備した歩兵の方が
良いとの意見も有るだろうか
271名無し三等兵:02/09/30 09:39 ID:???
>>234-240あたり(?)
銃砲の普及後の隊形については、兵のモラルの問題と不可分かと思われます。

プロイセン軍の横隊戦術は、多少は敵対する軍隊よりもモラルは上とは言え、
基本的に傭兵・徴募兵が主体の低モラルの軍隊の中で、火力を極大化し、
できるだけ、一翼にそれを集中させて…という発想があるかと思われます。

一方、ナポレオンの軍隊は、近代的な意味での徴兵主体であり、かつ、
当初は革命防衛、ついでナショナリズムというイデオロギー的な
熱狂・幻想に支えられた、異様にモラルの高い軍隊です。

それがさらに、ナポレオンのような優れた将帥が統率し、さらには、
そこそこのレベルの下位司令官たちに指揮されるわけですから、
崩れにくい。それゆえに、損害は出やすくても衝撃力の高い、
縦隊による銃剣突撃を可能であり、かつ機動力の高い運用が可能であった、
と、おれなんかは考えます。
そのあたりが、アウエルシュタットにおける、ダヴーの軍団の大勝の一因であるかと思われます。

また、恐らく、政治的熱狂に近い心理状態の軍隊が戦場のイニシアチブを握り、
かつ、銃砲が支配的であったという点で、独立戦争時の米軍の戦術は、
ある程度、ナポレオン時代の戦術にも影響を与えているだろうな、
とも考えます。加えて言うならば、ラファイエットが率いるフランス軍は、
独立戦争に義勇兵(だっけか? 正規軍だったかも知れないが…)を送り込んでいるわけで、
そうした具体の戦闘例も見聞する機会も多かったろうと思われます。
272某研究者 ◆TkxmpUgI :02/09/30 10:45 ID:BMyfDEmk
まあ歩兵は槍騎兵を銃撃で阻止するか(砲撃に対する損害覚悟で)
隊列の厚みを増して止めるしか無い訳だろうか
273某研究者 ◆TkxmpUgI :02/09/30 11:12 ID:BMyfDEmk
>独立戦争時の米軍の戦術は、
>ある程度、ナポレオン時代の戦術にも影響を与えているだろうな、

まあ散兵は兎も角縦隊突撃は
米軍は用いておらぬ訳だろうか
274名無し三等兵:02/09/30 11:27 ID:???
>>273
縦隊突撃は、当時の米軍の人的資源の薄さを考えると、
やりたくてもできないでしょうねえ。

なお、散兵を基本とするようになるのが本格化するのは、
南北戦争後半からだとも思いますし、その意味では、
仏軍への影響は、部分的に留まりますかね。
275某研究者 ◆TkxmpUgI :02/09/30 12:10 ID:BMyfDEmk
まあしかし仏の槍騎兵がもう少し強力なら
ワーテルローで勝てぬにしてももう少し善戦できた可能性は
無いのだろうか(笑
276名無し三等兵:02/09/30 12:51 ID:???
>>275
まあ、槍騎兵に限らず、ロシア遠征以来の消耗による騎兵の数的不足と質的劣化は、
1815年では相当にきつかったでしょうなぁ。
まして、会戦中、ネイの判断ミスでそいつを、さらに損耗しちゃってるわけで…。
(映画『ワーテルロー』スレの方が相応しい話題かとも思えますがw)
277某研究者 ◆TkxmpUgI :02/10/01 10:40 ID:gGXrQ4bU
まあ横隊に対しては砲は威力不足なので
散弾を用いていたと言う事は無いのだろうか
(まあ遠距離から砲を横隊に何発も放てばダメージは与えられるだろうが
 効率はどうだろうか
 まあしかし砲で縦隊が一気に殲滅された様な例は
 有るのだろうか)
278某研究者 ◆TkxmpUgI :02/10/01 10:45 ID:gGXrQ4bU
まあ横隊に対して榴弾を放っても
矢張り爆発する前に通り抜けて仕舞い殆ど通常弾と同等の効果しか得られない訳だろうか
(まあ只縦隊に放った場合は外れても至近距離で炸裂すれば効果は有るだろうか)
279名無し三等兵:02/10/01 13:47 ID:BiLum42V
榴弾と近接信管の起源(ぶどう弾あたりまで)検証の必要ありますな。
というかキボーン
280名無し三等兵:02/10/01 22:29 ID:vUx6QEMQ
>275
どんな騎兵でも、そこそこ規律の行き届いた歩兵には勝てませんヨ。
>278
どんなリュウダンかわかんないけど、良く言われてるのを横隊組んだ
部隊に撃つととんでもない事がおこるのです。
281某研究者 ◆TkxmpUgI :02/10/01 23:21 ID:tzj46WYf
しかし同等コストの大砲と銃兵では
何れの攻撃能力が上だろうか
(まあ大砲は銃をアウトレンジ可能だろうが
 歩兵10人等に比べて攻撃力は
 敵を遠距離から一方的に砲撃出来ねばどうかだろうが
 歩兵のみでは敵に近接する前に砲撃でダメージを受け得るだろうが
 歩兵の人数が多ければそれでも敵に近接した後は攻撃力は上とは成らぬのだろうか)
282AIRSHIP:02/10/04 00:20 ID:OGWAdV9w
当時の銃には、まともな照準器は付いておらず、敵の目の中の白が見えた時に、発
砲したのは有名な話です。(戦列歩兵において。) 照準とは、敵戦列に向かって
凡その方向に向ける事であり、一斉発射に拠る公算射撃的なものであった事です。
砲には、曲りなりにも照準器が付いていて、グレープショットを歩兵銃より早く装
填できた事も事実です。(グスタフ・アドルフが戦列に軽砲を配備した理由。)
一発の玉の重量の上でも、射程は長く効果は大きかった様です。
ただ、散開した敵にはあまり効果は無かった様です。 
283某研究者 ◆TkxmpUgI :02/10/05 10:42 ID:vBsqUWPU
まあ只軽量砲を防御時は兎も角突撃時に使用できるのかと言う事だろうし
防御時は兎も角突撃時は歩兵を増やした方が良いとの意見も有る訳だろうが
284某研究者 ◆TkxmpUgI :02/10/06 07:45 ID:+GJMg4lp
まあしかし弓が主兵装である時代に
近接戦用の日本刀があれだけ高度な物と成ったのも疑問では有る訳だろうが
(或いは雑兵は長巻が主兵装であるから其処からの技術移転なのだろうか)
285名無し三等兵:02/10/06 12:40 ID:gHLvFisK
日本史板にこんなスレが・・・

http://academy.2ch.net/test/read.cgi/history/1033130728/l50
本当は平和だった戦国時代
286名無し三等兵:02/10/06 16:05 ID:???
自作の武器作ったことある奴の数→
http://sports.2ch.net/test/read.cgi/budou/1014126743/

754からの自作の鎧や模擬戦の話題がおもしろい
287某研究者 ◆TkxmpUgI :02/10/07 09:38 ID:+F5Xo2rE
まあ長巻の登場が8世紀・蕨手刀の登場が7世紀であるから
日本刀の様な刀が出て来たのは長巻の影響では無い訳だろうか
しかし割りに短い刀である蕨手刀が
両手用と成り長大化したのは何時頃である訳なのだろうか
(これは長巻の影響で長大化し両片手用と成ったと言う事は無いのだろうか)

歩兵も長巻の前は矛或いは剣が主武器で有った訳だろうか
しかし弓が騎兵の主兵装と成ったのは何時頃からなのだろうか
(まあ弓が騎兵の主兵装と成る前に日本刀の原型の様な刀が
 作成された可能性と言うのも有る訳だろうか)
288某研究者 ◆TkxmpUgI :02/10/07 09:52 ID:+F5Xo2rE
>しかし弓が騎兵の主兵装と成ったのは何時頃からなのだろうか

まあ弓を止める為の大鎧が登場するよりは恐らく前だろうが
矢張り其処から100年等は前では無い訳だろうか
289えすえすびーえぬ:02/10/11 00:04 ID:???
日本はどうかしりませんが、西洋というか中東というかでは騎兵が兵科になったときから弓が主兵装ですヨ
その前の戦車の主へいそうも弓でしたし。
それに対抗するために装甲化が始まったんでしたっけか?
290某研究者 ◆NITkxmpUgI :02/10/11 18:24 ID:ZH9KaLgc
>日本はどうかしりませんが、西洋というか中東というかでは騎兵が兵科になったときから弓が主兵装ですヨ

まあヌミディア騎兵やギリシャ・ローマの騎兵も投げ槍騎兵であるし
戦車は兎も角騎馬弓兵を用いていたのはモンゴルや中国の方が
先かも知れぬが
291某研究者 ◆NITkxmpUgI :02/10/13 12:40 ID:3TH20pBu
http://www2.justnet.ne.jp/~teppounosato/kaji/hinawa2.htm
しかし上の20連発の火縄銃と言うのは
一人で運用可能なのだろうか(苦笑
(歩兵は兎も角騎兵は1発銃を放ったら基本的に突撃するのであれば
 同時に多数の弾丸を放つ銃を持った方が良い訳だろうか
 まあコストの問題も有るだろうが騎兵に比べたら
 連発銃等安価だろうか)
292某研究者 ◆NITkxmpUgI :02/10/13 18:33 ID:3TH20pBu
http://syoryuu.hp.infoseek.co.jp/sengoku/yoroi5.html
http://syoryuu.hp.infoseek.co.jp/sengoku/yoroi2.html
まあこの様な前立てが有るとは驚いたが

http://syoryuu.hp.infoseek.co.jp/sengoku/yoroi1.html
しかし赤い面頬等他に見た事は無いが
(まあ大概は黒い物な訳だろうが)
293某研究者 ◆NITkxmpUgI :02/10/15 23:34 ID:2uuuur06
まあ竹束も内部に粘土と石を詰めて
至近距離からの銃撃も止められる様に成っていたと言うが
大阪夏の陣で鉄の盾が使われたのは竹束の効果が無かった故であり
粘土と石を詰めてさえ銃には貫かれていたのだろうか
(まあしかしその鉄の盾も貫かれていた様だが(苦笑
 これは至近距離でのみだろうか)
294名無し三等兵:02/10/15 23:39 ID:???
>>293
鉄砲玉よけで変わりダネですと、牛や熊の生皮を立てかけ、
そこに水をぶっかけて重くしたものとかもあったらしいです。
ポイントは、スダレみたいにして下は固定しないこと、とか。
弾が、皮に当たって動くうちに運動エネルギーを使い果たして
皮を貫く前にポトリ、ということらしいです。
295某研究者 ◆NITkxmpUgI :02/10/15 23:47 ID:2uuuur06
まあ上は要するにイスラエル戦車のスカートの様な
物である訳だろうか(笑
只矢張り至近距離からの銃撃には対応しておらぬ訳だろうが
296某研究者 ◆NITkxmpUgI :02/10/17 18:36 ID:mfzOO9HF
まあ戦国時代の火縄銃も明の倭寇経由で東南亜細亜で使われていた
銃を入手したとの意見も有る訳だろうが
南蛮胴は流石にヨーロッパから買った物だろうか
297某研究者 ◆NITkxmpUgI :02/10/24 00:04 ID:9xFiv43+
http://www.sevenstarstrading.com/article/2hand/ming3.html
まあこれは明が日本から輸入或いは模倣した日本刀の
扱い方だろうか(矢張り使い方は日本の其れとは異なるのだろうか)

http://www.sevenstarstrading.com/article/2hand/qing.html
これも日本刀の様な武器だろうし明のみ成らず清も日本刀の様な武器を
使用していたと言う事だろうか(まあしかし朝鮮に関してはどうなのだろうか)
298某研究者 ◆NITkxmpUgI :02/10/24 00:19 ID:9xFiv43+
http://www.sevenstarstrading.com/article/08art02.html
まあベトナムでも日本人傭兵等の影響か
日本刀の様な刀が使われていた様だが
299某研究者 ◆NITkxmpUgI :02/10/24 00:51 ID:9xFiv43+
http://www.artnet.com/Magazine/features/stern/stern11-17-4.asp
まあこれは割に装飾の少ない部分も有るが
実戦で使われた物では無い訳だろうか
300名無し三等兵:02/10/24 00:57 ID:???
>>297
明が平和ボケしてたのは有名な話。
朝鮮も同様。
ちなみに倭寇(殆どは中国人や朝鮮人)で日本刀の実用性が評価され、
日本刀は清の近衛兵の標準装備になっていた。
301某研究者 ◆NITkxmpUgI :02/10/24 01:10 ID:9xFiv43+
>日本刀の実用性が評価され、
>日本刀は清の近衛兵の標準装備になっていた。

まあ鎧の有効性が火器で薄れていく中では
切れ味の良い刀の有効性は増していたと言う事だろうか
(しかしコピー品も日本刀同様に硬軟二重構造なのだろうか)
302300:02/10/24 01:50 ID:???
>>301
日明貿易ググってみればわかるけど、日本の輸出品に「刀」がある。
中国や朝鮮で日本刀を作っていたかどうかは知らない。

明に関して言えば、鎧は日本と大差無い。
集中運用された鉄砲との戦闘は全く経験が無く、
秀吉の朝鮮出兵で、明の重装騎馬隊が日本の鉄砲に大打撃を受けている。

 余談だが、島津が朝鮮出兵で滅茶苦茶強かったので
 朝鮮人から「石曼子(シーマンズ)」と呼ばれて恐れられますた。

中国の剣というと、青龍刀のような叩き切る大刀と、突くための直線の剣しかない。
もっと言えば、中国の歩兵の武器は槍か弓。
剣はあくまで個人レベルでの特殊兵装。
清の時代ならサーベル導入も考えられるが、
サーベルは片手保持なので敵に大打撃を与えられない。
日本刀は軽量ながらも両手保持「も」でき(力を入れられる)、
小型ながら突く・切る両方ができる実用的な点を
皇帝の身辺警護用で評価されたんではないかと思う。
303某研究者 ◆NITkxmpUgI :02/10/24 07:13 ID:DIuPm0w3
>中国や朝鮮で日本刀を作っていたかどうかは知らない。

まあ形の似た刀は有る様だが
これの内部は硬軟二重構造と成っているのだろうか


>明に関して言えば、鎧は日本と大差無い。

まあ鉄板が一体化されている日本の鎧より
中国の札鎧や鱗鎧の方が動き易いが打撃には脆いと言う事だろうか



>集中運用された鉄砲との戦闘は全く経験が無く、
>秀吉の朝鮮出兵で、明の重装騎馬隊が日本の鉄砲に大打撃を受けている。

まあ重騎兵の装甲も銃には無力だったと言う事だろうし
騎兵を阻止する為の長槍兵の存在も有ったろうか


>もっと言えば、中国の歩兵の武器は槍か弓。

槍は短槍であるなら騎兵や日本の長槍兵を阻止し難いと言う事は
有る訳だろうか
304某研究者 ◆NITkxmpUgI :02/10/24 07:14 ID:DIuPm0w3
>サーベルは片手保持なので敵に大打撃を与えられない。

まあ只馬上では日本刀も片手で運用だろうが
大太刀の様に馬上でも両手で用いられる場合も有る訳だろうか
日本刀はサーベルより重量が軽い(サーベル1.5kg・日本刀1kg)が
切れ味が良く装甲の無い相手には威力が有る事
重量の割に硬軟二重構造で強度が高い点・長さも恐らく変わらない点が
評価された可能性も有るだろうか
(サーベルと同じ1.5kgの重量の日本刀を作れば
 矢張りサーベルよりかなり長い物が出来るだろうか)
305某研究者 ◆NITkxmpUgI :02/10/24 07:35 ID:DIuPm0w3
>集中運用された鉄砲との戦闘は全く経験が無く、

まあ彼等のクロスボウや弓は盾(或いはハンドキャノンもか)で止められても
銃は無理であったと言う事だろうが
明軍の騎兵は兎も角歩兵の装甲は殆ど綿甲と盾しか無く
歩人甲等は少数である訳だろうか
まあしかし明軍の大砲の炸裂弾や散弾には日本もやられていた可能性は
無いのだろうか
(まあ日本側の大砲にも炸裂弾は兎も角散弾は有るだろうし
 (まあ大砲の絶対数の差は銃で補っていた訳だろうか)
 散弾は兎も角炸裂弾は横隊を取っておれば余り被害は無い訳だろうか
 散弾も竹束等なら止められたと言う事だろうか)
306砲術隊:02/10/24 09:01 ID:7OKbyOYh
10月27日に行われる高島平まつりのイベントとして、高島第七小学校の校庭
で「西洋流火砲鉄砲隊」による演武が行われます。
「西洋流火砲鉄砲隊」は幕末に長崎の役人・高島秋帆が徳丸原(現・東京都
板橋区高島平) で行った砲術調練の再現をめざしてこの9月に結成されました。
当日は、不可能と思われていた火薬の使用許可が下りたため、本物の火縄銃
の発砲をします。(空砲だけど弾が入って無いだけで実射と同じ。)15名で
合計100発くらい発砲するのですごい迫力です。 時間は午後2時過ぎから。
秋帆が使用したのはゲベール銃ですが、雷管式の為個人所有・使用が不可能
なので火縄銃で代用します。また、行う演武も今回はオランダ式ではありま
せんが今後、文献を研究し順次改めていくのであまりつっこまないで下さい。
最寄り駅は、都営三田線高島平駅下車南口出て5分(高島平図書館隣)

307某研究者 ◆NITkxmpUgI :02/10/24 09:14 ID:NogUrpE3
>まあ彼等のクロスボウや弓は盾(或いはハンドキャノンもか)で止められても

まあハンドキャノンの銃身の長さはかなり短い訳だろうし
威力・射程は火縄銃に相当程度劣っていた訳だろうか


西欧の短槍兵には手甲や足甲・盾が装備されている物も多いが
これであれば矛槍兵は兎も角長槍兵には対抗可能だろうか
(まあ完全鎧が装備されている矛槍兵・短槍兵・長槍兵も少数派は存在するだろうが)
騎兵を阻止するなら短槍兵でも一応何とか成るだろうか
308某研究者 ◆NITkxmpUgI :02/10/25 06:27 ID:QOsUG+1l
しかし十字軍に相当数存在したと言われる女騎士の中で
アルスラーン戦記のエトワールの様に
裏切った者は居た訳なのだろうか

イスラム側も女騎士の様な者が存在したなら
彼等同士の戦いと言うのは有ったのだろうか

まあしかしエトワールの様に恐らくろくに風呂にも入っていない様な女等
アラブ側では野蛮人として扱われる以外無かったかも知れぬが(苦笑
(まあ騎士は一応水泳の訓練を受けていると言うから
 他の者より多少はましかも知れぬが(苦笑)
裏切った後はまあ入っていたのかも知れぬが(笑

しかしギリシャやローマ時代・或いはマケドニア・暗黒時代や其れ以後
或いはバイキングには女騎士の様な者は居たのだろうか
309某研究者 ◆NITkxmpUgI :02/10/25 06:38 ID:QOsUG+1l
>イスラム側も女騎士の様な者が存在したなら
>彼等同士の戦いと言うのは有ったのだろうか

まあしかし他の場所でも女騎士対女騎士の戦いと言うのは
有った訳なのだろうか
310某研究者 ◆NITkxmpUgI :02/10/30 13:06 ID:T3XhY/2O
まあ西欧には馬上からハンドキャノンを使用している絵も有るが
これはピストルやカービン銃の様に多用はされなかったと言う事だろうか
(クロスボウを馬上で構えている様な絵も有るが
 これも馬上からの射撃は多用されていない訳だろうか)
311某研究者 ◆NITkxmpUgI :02/10/31 07:46 ID:rCVIh2uh
まあ十字軍の弓の射程が30mしか無く
アラブ軍の弓は150mの射程が有ったと言う記述も有るが
十字軍のクロスボウの方がアラブ軍の短弓より
射程は長いのではないのか
(まあしかし30mと言うのは短弓としても余りに短いのではないのか(苦笑)
312海の人●海の砒素:02/10/31 07:52 ID:???
>311
 クロスボウの場合は狙撃(目標を狙って当てる)、アラブ軍の弓(って、例の馬上弓ですか?)
の場合は数うちゃあたる(大勢で、文字通り矢を「降らせる」)からじゃないでしょかね、その射程
の違いってのは。
313某研究者 ◆NITkxmpUgI :02/10/31 09:25 ID:Wq5KOHCj
>クロスボウの場合は狙撃(目標を狙って当てる)、

まあこれはクロスボウの数が無いから狙撃せざる負えぬのだろうか
しかしクロスボウが万単位で存在しているのに
30m迄接近しないと放たないと言う事は
矢張り敵に完全鎧等は無い訳だろうから無いと言う事だろうか(笑
314名無し三等兵:02/11/02 12:44 ID:???
きのう、始皇帝暗殺をやってた。
戈を振り回す戦車戦や、攻城戦に萌え
315えすえすびーえぬ:02/11/04 22:46 ID:???
十字軍(第何次か忘れた)の重装騎兵がアラブ兵蹴散らしたのってどんな感じで展開したんでしょうかな。
弓を装甲で弾きつつ敵陣突破→壊乱?
316某研究者 ◆NITkxmpUgI :02/11/05 23:07 ID:yIkeoK5/
>弓を装甲で弾きつつ敵陣突破→壊乱?

まあ矢が尽きた所に突入したのか
歩兵の長射程のクロスボウの射撃で戦意を喪失した所への
突撃だろうか
(まあイスラム側も弓騎兵や弓歩兵ばかりではないし
 弓の攻撃で陣形が崩れない事に痺れを切らして
 無理に突入して来た槍歩兵や槍騎兵に対しての突撃
 (無論これはクロスボウで突入時に混乱したかも知れぬが)かも知れないが)
317某研究者 ◆NITkxmpUgI :02/11/05 23:13 ID:yIkeoK5/
>弓の攻撃で陣形が崩れない事に痺れを切らして

まあリチャードも騎兵を歩兵の後ろに置いていたと言うから
完全にクロスボウの長い射程のみでアラブ軍を止められ無く共
弓歩兵や歩兵のパビスでアラブ軍の弓が無力化されていた可能性は
有るだろうか
(アラブの槍騎兵はクロスボウの前に接近出来ず
 (まあこの時代に歩兵の槍襖は恐らく無いだろうし
  槍歩兵さえおらぬのかも知れぬが)
 アラブの槍歩兵は十字軍と異なりパビスの様な盾が無ければ(まあ小型の盾は有るのかも知れないが)
 十字軍側のクロスボウを防ぐ手段は無かったと言う事だろうか)
リチャードはパビスから身を乗り出した時にクロスボウで狙撃され命を落としたと言うから
十字軍の時代にも恐らくパビスは用いられていたと言う事だろうか
318某研究者 ◆NITkxmpUgI :02/11/05 23:15 ID:yIkeoK5/
まあアラブ軍側は十字軍の盾を奪って使用する等して
柔軟な部分も有る様だが
十字軍側はサラセン軍の武器等を奪って
使用する事は有ったのだろうか

まあアイユーブ朝騎兵の鎖の上に鱗鎧を装備した鎧の方が
十字軍の鎖のみの鎧より強力かも知れぬが
アイユーブ朝軍の鎖帷子は十字軍と異なり全身には施されておらぬ訳だろうか

ウマイヤ朝の騎兵も鎖の上に鱗鎧を装備しており
同時代のビザンツ帝国の騎兵よりも重装備だったのではないのか

まあオスマントルコの兵士も割に重装備であり
イスラム軍も鎖帷子を着ていない様な兵士は割に少数だった可能性は
有る訳だろうか

まあイスラム軍も兜を被った兵士も多く
映画の様に皆ターバンを巻いて皮鎧や服しか着ていないと言う様な事は
無いと言う事だろうか

まあ矢張り白人のイスラム教徒は野蛮人であると言うイメージが
映画に出て来る彼等から兜や鎖帷子・鱗鎧を取り去っている訳だろうか

(まあイスラムとは関係無いがササン朝ペルシャの騎兵も
 同時期の西欧の騎兵等とは比べ物と成らぬ位重装備である訳だろうか)
319某研究者 ◆NITkxmpUgI :02/11/05 23:19 ID:yIkeoK5/
まあ只アラブ軍側にもパビスの様な大型の盾が有ったのかも知れないが
どうだろうか
320某研究者 ◆NITkxmpUgI :02/11/05 23:21 ID:yIkeoK5/
まあローマや西欧よりもビザンツ帝国や中国の方が
馬鎧は用いていたのではないのか
ローマのプレートアーマーは兎も角中国の明光鎧の方が
同時期の西欧の鎖帷子等よりは強力だったのではないのか
(しかしローマ兵の鎧は皆腕部が無防備だが
 同時代のパルティア等の方が腕は防護されていたのではないのか
 ビザンツ帝国の兵士やバイキングも割に腕は無防備の様だが)
321某研究者 ◆NITkxmpUgI :02/11/05 23:23 ID:yIkeoK5/
まあしかし日本の鎧の篭手等の部分に鎖が使われたのは何時頃からなのだろか
(鎌倉時代の物には矢張り無いだろうが使われたのは
 西欧の甲冑の影響では無いと言う事なのだろうか)
腿の部分を防護する佩楯の装備や袖の小型化は西欧の甲冑の影響では
無いのだろうか

まあ割に防護されている部分が多いとされる当世具足も
鎧の種類に拠らず上腕部は鎖程度で余り防護されていないから
ここに矢や槍が命中すると脆いのではないのか
(まあ只後ろからの矢は幌で止まる場合も有るだろうが)
322某研究者:02/11/05 23:26 ID:yIkeoK5/
まあアラブ軍のクロスボウの装備率は矢張り十字軍よりは低いかも知れぬが
アラブ軍の弓の装備率と言うのは十字軍よりそれ程
高い訳なのだろうか
323某研究者:02/11/05 23:40 ID:yIkeoK5/
>ローマのプレートアーマーは兎も角中国の明光鎧の方が
>同時期の西欧の鎖帷子等よりは強力だったのではないのか

まあほぼ全身を被う鱗鎧である宋の歩人甲も
十字軍の鎖帷子よりは強力かも知れないが
324某研究者:02/11/06 00:03 ID:+G2r4thF
まあ弓に対してはプレートアーマーや鱗鎧が強く
鎖帷子や掛甲は隙間や穴の部分が脆いと言う事だろうか
(まあ只槍に対しては鱗鎧は付き込まれると脆いだろうか
 しかし掛甲と比べてどうだろうか)
325某研究者 ◆NITkxmpUgI :02/11/06 18:09 ID:1zbg6/rF
http://www22.tok2.com/home/lifazhu/t3.html
まあ上を見るとアイユーブ朝軍の槍歩兵はパイクの様な槍と
大型の盾・投げ槍迄も装備している様であり
西欧の歩兵より強力だった訳だろうか
まあ只盾の枚数が無ければクロスボウでアウトレンジされる危険は
有ったと言う事だろうか
326名無し三等兵:02/11/06 18:20 ID:???
327名無し三等兵:02/11/06 18:24 ID:???
その武器の両翼には、

風が坐し、その刃には太陽と火が燃えさかる。

その体は青白く輝き、

その重さは2つの山からなるもののようであった。

これは天地のあらゆる元素より成るものであった。

自ら炎のような火を発して進み、

轟々と鳴り響き、金鉄よりも堅く、

この世界のあらゆるものをたちまち破壊する力を持っている。

(註・そしてこの武器が実際に使われたとき)

その恐ろしい光り輝く強大な槍が放たれたとき、

30万もの軍勢が、一瞬のうちに滅び去った。


『ラーマーヤナ』よりラーマ王が決戦で、使用する決戦兵器の形態と使用した結果について 抜粋

古過ぎますか?
328名無し三等兵:02/11/06 18:25 ID:???

   /⌒⌒⌒⌒⌒ ))     
  (   从   ノ.ノ       
   ( ./  ̄ ̄ ̄ ̄\     
   |::::::  ヽ     丶.      
   |::::.____、_  _,__)      
  (∂: ̄ ̄| ?? =| ?? |     
   (  (   ̄ )・・( ̄ i     
   ヘ\   ... )ー( .._丿    ねぇ、お米ちょうだいよ。
 /  \ヽ _二__ノ|\     何でも話すから。
/⌒ヽ.   \__/\/\_|     この目が嘘ついてると思う?
|   ヽ       \o \
|    |          \o \
329某研究者 ◆NITkxmpUgI :02/11/06 20:58 ID:20PDXZk/
まあアイユーブ朝軍の弓騎兵と十字軍の重騎兵の戦いは
正直勝負に成らぬ(無論弓騎兵を重騎兵が包囲するか川にでも追い詰めない限り前者が一方的に勝つ)だろうが
アイユーブ朝軍と十字軍の重騎兵同士が戦っても
馬の扱いの差等で前者が勝った可能性は高い訳だろうか

歩兵はクロスボウと弓の射程の差を生かせれば十字軍側が
勝てた可能性も有る訳だろうか
330某研究者 ◆NITkxmpUgI :02/11/06 22:08 ID:20PDXZk/
まあマムルーク騎兵の騎射能力が高いのは
モンゴル人が存在する所為だろうか
(まあトルコ人の騎射能力も高かったのかも知れないが
 ギリシャやスラブ人等はどうだったのだろうか)
331某研究者 ◆NITkxmpUgI :02/11/07 15:58 ID:1XJa6xNW
http://crusades.boisestate.edu/Islam/13.htm
まあ上を見るとサラディンがプレートアーマーの様な物を
着ている様にも見えるがこれは誤りであり
十字軍の時代には存在しない物だろうか
(まあトルコの軍楽隊等を見るとプレートアーマーの様な胴鎧を
 着ている者も存在する様だがこれは欧州の胸甲兵の模倣だろうか)
332某研究者 ◆NITkxmpUgI :02/11/08 00:32 ID:sDpLKSbt
まあ武田信玄が上洛しておれば
信長や秀吉等只の愚将とて扱われた可能性も
有るのではないのか(苦笑
まあ信玄の考案した140kmを二時間で伝わったという狼煙のネットワークの様な物は
信長等も用いていたのだろうか
まあしかし日本の日の丸と言うのも元は確か
武田の旗ではなかったか
家康は武田の政治や軍事手法を真似ていたと言うが
これは決定的な物だったのだろうか
333某研究者 ◆NITkxmpUgI :02/11/08 00:34 ID:sDpLKSbt
http://www8.plala.or.jp/shingen/shingen/shoyuhin.html
まあ上の信玄の軍配に水晶が埋め込まれているのは
何か意味が有るのだろうか
信玄の兜は割に地味だがこれのみを用いていた訳では
無いと言う事なのか
334某研究者 ◆NITkxmpUgI :02/11/08 01:06 ID:sDpLKSbt
まあ武田のライバルは所詮謙信で有って
信長や秀吉・家康等は眼中に無かったと言う事は
無かったのだろうか(笑
335某研究者 ◆NITkxmpUgI :02/11/08 12:13 ID:21U170JQ
まあ後は武田軍の鉄砲装備率に関してだが
長篠の信長軍40000の鉄砲が3000
武田軍15000の鉄砲が1500であるから鉄砲の装備率は
武田軍の方が上ではないのか
(まあ武田は織田軍と異なり竹束や塹壕が使用出来なかったと言う問題は
 有るだろうが)
336某研究者 ◆NITkxmpUgI :02/11/08 12:36 ID:21U170JQ
>長篠の信長軍40000の鉄砲が3000

まあこれは1000丁程度との説も有るから
鉄砲の装備率だけではなく総数においても
長篠の武田軍の方が上だった可能性は有るだろうか
337某研究者 ◆NITkxmpUgI :02/11/08 14:46 ID:21U170JQ
まあそうすると鉄砲を使用する兵は全軍の1割にも満たないと言う事だろうが
弓兵は其れより更に少ないのだろうか
(まあ両方共雨天には使用し難いだろうしこれを主力とするのは
 城の防衛用でも無い限り無理である訳だろうか)
338某研究者 ◆NITkxmpUgI :02/11/08 14:50 ID:21U170JQ
まあナポレオン等の戦列歩兵の火打石式の銃も
雨天では使用出来ない訳なのだろうか
(まあそうすると雨天では槍騎兵に
 戦列歩兵は全く勝てないだろうが)
339某研究者 ◆NITkxmpUgI :02/11/08 14:56 ID:21U170JQ
まあ近世の攻城戦では攻城塔は使用されたのだろうか
(基本的に大砲での城壁破壊或いは
 兵糧攻めである訳なのだろうか)


>まあそうすると鉄砲を使用する兵は全軍の1割にも満たないと言う事だろうが
>弓兵は其れより更に少ないのだろうか

故に実質的には槍歩兵や槍騎兵の力や物量が
野戦では勝敗を分けたのかも知れぬが
武田軍の槍が信長軍の槍より短かったと言う事は
無いのだろうか
340某研究者 ◆NITkxmpUgI :02/11/08 15:08 ID:21U170JQ
まあそうするとスイスのパイク兵が重要であると言う事かも知れぬが
下馬した重騎兵にパイクがどの程度通用するかと言う事も
有るだろうか
(まあハルバードであれば効果は有るだろうか)
341名無し三等兵:02/11/08 15:14 ID:x9x5bl6Y
帝京学園の冲永嘉計・元理事長を脱税容疑で逮捕
http://headlines.yahoo.co.jp/hl?a=20021106-00000211-yom-soci
帝京大学(東京都板橋区)を舞台にした多額の寄付金問題で、東京地検特捜部は6日、系列の学校法人「帝京学園」元理事長・冲永嘉計容疑者(58)(品川区)を、受験生の親から受け取った入学をめぐる口利き料を隠した所得税法違反(脱税)の疑いで逮捕した。
同容疑者は、同大の冲永壮一前総長(今年9月に辞任)の実弟。特捜部は今後、入学の口利きを巡る不透明な資金の流れの解明を進める。

【社会】脱税で帝京学園元会長を逮捕
http://news2.2ch.net/test/read.cgi/newsplus/1036572579/

帝京の犯罪行為一覧
○単位を取れなかった腹いせに理工学部の学生が教員をリンチ
○いじめで校舎から飛び降り自殺
○暴走族、麻薬、食堂放火
○集団レイプ
○薬害エイズ
○守衛にスパイクシューズのまま跳び蹴り
○暴力団を雇って組合員を銃撃
〇脱税
〇宗教法人乗っ取り

帝京大学の過去の犯罪特集・1
http://choco.2ch.net/test/read.cgi/news/1015445145/
342某研究者 ◆NITkxmpUgI :02/11/08 15:22 ID:21U170JQ
http://216.239.33.100/search?q=cache:_fZB6NeXjmkC:www.jttk.zaq.ne.jp/babsu400/yari.html+%E6%AD%A6%E7%94%B0%E8%BB%8D%E3%80%80%E6%A7%8D%E3%80%80%E9%95%B7%E3%81%95&hl=ja&ie=UTF-8&inlang=ja
まあ武田軍の槍には木槌が付いていたと言うが
長さは織田軍の槍と比べてどうだったのだろうか
(長さが同じなら武田軍の槍兵の方が上と言う可能性は無いのだろうか
 騎兵の槍には木槌は付いていたのだろうか)

しかし日本の騎兵が良く言われる様に下馬戦闘主体であれば
槍が歩兵より短い必要は果たして有るのかと言う事だろうが

島津の騎銃兵は雨天ではどうかだろうし
雨天では槍を持って戦っていたと言う事は無いのだろうか
(まあ他にも城の防衛時等は銃を使えた方が有利だろうが
 島津騎兵の槍の能力と言うのは他の騎兵より低いと言う事は
 無いのだろうか)
343某研究者 ◆NITkxmpUgI :02/11/08 15:26 ID:21U170JQ
まあ長篠も柵のみではなく塹壕の様な物で
敵歩兵や騎兵を阻止していたと言うから
銃兵の数が割りに少なく共長時間銃撃を加える事で
敵の損害を増せたと言う点は有るだろうが
これは果たして決定的な物と成り得た訳なのだろうか
武田軍も竹束の盾等を使えば遠距離からの銃撃は防げただろうが
近距離からの銃撃も恐らく止められる車竹束等の重い物は泥濘化した状態では運べなかった
或いは運べても壕を越せなかったと言う可能性は有るだろうか
344某研究者 ◆NITkxmpUgI :02/11/08 15:36 ID:21U170JQ
http://museum.umic.ueda.nagano.jp/hakubutsukan/story/matsudaira/cont_matsu/doc_matsu/014.html
まあ武田軍の槍は5m有る物も有る様だが
信長軍の歩兵の槍は皆6.5m等有ったのだろうか
まあ上の物には木槌は付いておらぬ様だから
矢張り武田軍の槍全てに木槌は装備されていなかった訳だろうか
345某研究者 ◆NITkxmpUgI :02/11/08 22:09 ID:3oiGfa/x
http://www8.plala.or.jp/shingen/shinp1.html
兵士が移動する為の棒道等は
確か謙信は使用していなかったかと思うが
信長等が後で作成したと言う道路も信玄の模倣と言う意見も有るがどうだろうか
346名無し三等兵:02/11/13 00:12 ID:???
     
347某研究者 ◆NITkxmpUgI :02/11/16 06:36 ID:6u2GWUWX
http://plaza4.mbn.or.jp/~tactic/ainj.htm
まあマムルークがモンゴル軍に対抗する為にキリスト教徒の領地から
補給を受けたとの記述も有るが
これは十字軍側もモンゴル軍は危険な連中として見ていた所為だろうか
348名無し三等兵:02/11/17 19:05 ID:???
>余談だが、島津が朝鮮出兵で滅茶苦茶強かったので
>朝鮮人から「石曼子(シーマンズ)」と呼ばれて恐れられますた

これは最後の戦いで名を上げたから、そう言われるようになったわけで
それ以前は城に押し寄せた明の小規模の軍勢を大鉄砲で撃退
する程度で、ほとんど交戦の機会はありません。

もしかしてソースは司馬氏の小説ですか?
349某研究者 ◆NITkxmpUgI :02/11/17 19:13 ID:hpAOQMTK
>まあ武田軍の槍は5m有る物も有る様だが
>信長軍の歩兵の槍は皆6.5m等有ったのだろうか

まあ槍の長さは統一されていたと言う意見も有る様だが
武田軍の歩兵の槍は全て5m・信長軍は全て6.5mと言う訳では
無いと言う事なのだろうか
350名無し三等兵:02/11/17 23:39 ID:3DkvxHvW

超強力漢方バイアグラ「威哥王」、「蟻力神」をどこよりも安い特価でご提供!
ガンやアトピーを治す免疫力を強化する話題のアガリクスもなんと1キロ10000円!!!
緊急避妊薬(アフターピル)や低用量ピル「マーベロン」、抗鬱剤プロザックも特価!
そしてあの101の最新版101Gやダーバオであきらめていた髪もフサフサ!

http://www3.to/ultimateviagra
http://ultimateshop.lastsale.com
http://urlto.net/ultimateshop/
(上記3つの内繋がり易いものからいらしてください)
351名無し三等兵:02/11/18 06:02 ID:???
中国漢方恐るべし。毒で我々をあぼーんする手か
352某研究者 ◆NITkxmpUgI :02/11/23 08:17 ID:lZR+YsKm
まあ鎌倉時代は騎射戦主体であるなら巴は映画等の様に
薙刀では無く現実には弓を武器としていた訳だろうか
353名無し三等兵:02/11/23 12:11 ID:???
そのころの(承久/何某時代、と呼称するらしいが〜忘れちゃった)の、いくさの研究は、いろいろやられていて何冊か読んだことがります。
弓が最上級の武器で、長柄や刀は、徒歩の武器だったそうな。
一説によれば、源平合戦のころから、地元民が工兵やら警備やらで大量動員されるようになり、以前の土豪武士たちの騎乗弓射・組討みたいな戦術から離れていったとも、いいますが。
手元にないんで、書名を出せない。スマソ
354某研究者 ◆NITkxmpUgI :02/11/26 13:22 ID:uytAQzx/
まあしかし合成弓でさえ無いイングランドのロングボウが
合成弓である日本の弓より射程が長いと言うのは
引き手の体力の問題だろうし
日本の弓にも八人張り等の強力な弓は有る訳だろうし
これはイングランドのロングボウの一番強い物・或いはモンゴル等の強化弓と
威力は変わらないかも知れないが
355名無し三等兵:02/11/26 13:24 ID:???
>>354
ロングボウとかは、下賎の民の使う、支援兵器的扱いではなかったでしょうか。
「兵器と戦術の世界史」つー本で、鎌倉武士を「準装甲弓騎兵」と、絶妙な表現をしておりました。
自ら槍もち突撃するタイプの重装甲騎兵でなく、弓射の威力による〜という評価でしょうね。
356某研究者 ◆NITkxmpUgI :02/11/26 15:24 ID:uytAQzx/
サーベルの原型のスイスのシュヴァイツァーサーベルは
バスタードソードの一種の様だがこれは曲刀なのだろうか
曲刀であれば16世紀の登場であるから
これは両片手用でもあるなら日本刀を参考に作られた可能性も有るかも知れないが

サーベルがアラブのシミターを参考としたなら先に片手用の物が
出て来る筈であるし両手用の物が先に出てきたと言う事は
当時輸入されたかも知れぬ日本刀が参考とされた可能性は有るのではないのか
(まあ只直刀型のサーベルも有るのでシュバイツァーサーベルが
 必ずしも曲刀である訳では無いだろうか)
357名無し三等兵:02/11/26 16:09 ID:???
シミターは、鎧をあまり着用しない中東の武器ですな。
両手用の武器は、鎧を想定していると思われるので、形だけの模倣でしょう。
つーか、日本刀を模倣した可能性は、ほとんどないと思われますが。
358某研究者 ◆NITkxmpUgI :02/11/26 16:25 ID:uytAQzx/
>両手用の武器は、鎧を想定していると思われるので、形だけの模倣でしょう。

日本刀では無く両手用のファルシオンの様な物から
変化した可能性も有るのだろうか
359名無し三等兵:02/11/27 11:13 ID:???
可能性はつねにあります。
戦場の貴族は、自己顕示欲高いですから、カスタム武器くらいは持つでしょう(と、いうより、武器という特殊な製品は、文化的な伝承とともにあるはずですから、そう極端に進歩しなかったのでしょう。現代までは)。
360ネタ提供:02/11/28 07:24 ID:???
各国の陣形を語ってくれ。
騎兵隊の楔型陣形とか意味在るの?

日本の野戦での陣地とか愉快な感じ。
旗に使う布を迷路みたいに張り巡らして無かったっけ?
361名無し三等兵:02/11/28 11:43 ID:???
過去の戦場は、ほとんど統制が効かなかったから、陣形で戦闘開始のタイミングをとっていた、という説もある。
斜形陣なんて、それだな。
兵科に意味が出てくると、兵科を集中して、指図とともに適切なタイミングで戦闘を開始するようにもなる。
と、やはり陣形と順序とのかかわりがあろう、と指摘できる。

火力展開に関心がもたれるのは、銃砲が戦術に取り込まれてからだろう。
アザンクールが火力陣形を意図していたかどうかは、よくわからない(漏れは違うと思う)
362名無し三等兵:02/11/28 16:01 ID:???
>>361
火砲が出現する前の戦争だよ。
大砲までは許すけど(偉そう

金床とハンマーみたいな戦術は理解出来るんだけど
鏃型の陣形の意味が分からない。

アニメネタで悪いが、ベルセルクとかでも敵の包囲網から突破するのに
鏃型の陣形で突破したりしてた。
363保守:02/11/30 18:01 ID:???
 
364某研究者 ◆NITkxmpUgI :02/11/30 18:24 ID:oGqutvcu
陣形に関しては魚鱗による中央突破分断か
鶴翼で側面から後方に回り込み包囲殲滅と言う方向も有るだろうが
魚鱗で中央から後方に回り込む事も一応可能だろうか
(鶴翼が中央から魚鱗等の突入で二つに分断された場合の指揮系統は問題無いのだろうか)
敵の同数の横隊に対しては側面からは回り難い訳なら
魚鱗で攻撃した方が良い訳だろうか
365某研究者 ◆NITkxmpUgI :02/11/30 19:29 ID:oGqutvcu
まあナポレオンの散兵では砲兵と合わせても騎兵突撃を阻止し難く
騎兵が突入してくる様なら縦隊を横に並べるか
或いは後方の横隊も前進させて騎兵突撃を防いだと言う事は無いのだろうか
366某研究者 ◆NITkxmpUgI :02/11/30 19:43 ID:oGqutvcu
一人張りの弓が250m飛ぶなら
5人張りは1.2km飛ぶと言う様な事は無いのだろうか(苦笑
しかし8人張りは一体何m飛ぶのだろうか

8人張り等下手なバリスタより高威力かも知れぬが
一人は兎も角二人の人間を完全に貫く様な力は有るだろうか

まあしかしバリスタの弓の部分を取り外して
其れを怪力を持つ人間が直接引いて弓として使う様な事は
有り得たのだろうか
367某研究者 ◆NITkxmpUgI :02/11/30 20:03 ID:oGqutvcu
ナポレオンの陣形の中で狙撃を行う散兵の銃は精度の高い火縄銃の方が
良かったと言う事は無いのだろうか
(まあ火縄ライフル銃迄必要だったかは
 ライフル銃の装填速度を考える限り疑問かも知れぬが)
368某研究者 ◆NITkxmpUgI :02/11/30 20:08 ID:oGqutvcu
まあ只火縄銃では銃剣を使い難いと言う事も
有るのかも知れないがこれは致命的共言えないだろうか
369名無し三等兵:02/12/03 14:28 ID:???
>>365
いや、突撃は一度きりだから、戦闘全体が突撃のための前戯みたいな側面は、ある。
ナポレオンの「攻撃させるための陽動攻撃」なんて、それだと思ってる。
ギリシャの文献で、戦争を調べると、あのファランクスは、互いに丘の上で気勢を上げた後、なだれを打って陣形を乱して、互いに突撃して言ったらしい(いつの時代までそれが続いているのかは、漏れは知らんが)。
歩兵も、突撃したらそれで統制が保てなくなるのだと思う。

鏃型の陣形も、確固とした意思と目的を持った指揮官に追従して行くと、自然にああなる、という説もある(「軍事の地平線」あたりの紛争では、そうやってごく一部の軍事能力者が、決定的な能力を発揮したのでは、と漏れは妄想している。考古学的な資料はないけどね〜)
370某研究者 ◆NITkxmpUgI :02/12/03 18:07 ID:2eTSh7xx
>ギリシャの文献で、戦争を調べると、あのファランクスは、互いに丘の上で気勢を上げた後、なだれを打って陣形を乱して、互いに突撃して言ったらしい

まあそうするとローマのレギオンと乱戦に成っても勝てたのかと言う事だが
レギオンの機動力で包囲されてから乱戦に持ち込まれては
勝機は無かったと言う事だろうか
371某研究者 ◆NITkxmpUgI :02/12/03 18:09 ID:2eTSh7xx
17世紀のプレートアーマーの腕の関節部は
旧式の隙間の有る物である場合も有る様だが
これは何故なのだろうか
(可動範囲は旧式の方が広いのだろうか)
372某研究者 ◆NITkxmpUgI :02/12/03 21:24 ID:ugd5iRcp
http://academy.2ch.net/test/read.cgi/whis/1011864885/l50
まあナポレオン関連は上でもやっているから
興味の有る者は参照して貰いたい訳だが
373某研究者 ◆NITkxmpUgI :02/12/04 10:06 ID:vUVba8gD
まあしかしカラビニエールアーマーと言うのは3クオーターアーマーとは異なるのか
3クォーターよりやや軽装にも見えたが騎銃兵とサーベル騎兵の格の差に拠る物なのだろうか
374劣等従者:02/12/04 11:29 ID:???
>>370
ローマはおろか、マケドニアにも勝てなかったと思われ。

その文献はキーガンの「戦略の歴史」だったかもしれない(失念している。スマソ
突撃に失敗すると、逃げるしかなかったそうな。
大量の戦死者と、軍事傾倒をなんとなく納得する話ではある。
375某研究者 ◆NITkxmpUgI :02/12/04 12:03 ID:vUVba8gD
>ローマはおろか、マケドニアにも勝てなかったと思われ。

まあしかしマケドニアの槍兵も最後には陣形を乱して突撃していたのだろうか
(チャージ後は槍は役に立たなく成るから
 彼等は盾と剣も携帯していたのだろうか)
376劣等従者:02/12/04 12:07 ID:???
してたと思う。
ただ、突撃するまでの前戯が、アレクサンダーの場合すごく上手。
だから、戦闘力ではさらに差がついただろう。

マンアットアームズシリーズ(オスプリー 邦訳が大日本絵画から出ていたと思う)
では、盾と小剣を持っていたように思う。
377某研究者 ◆NITkxmpUgI :02/12/04 12:24 ID:vUVba8gD
>マンアットアームズシリーズ(オスプリー 邦訳が大日本絵画から出ていたと思う)
>では、盾と小剣を持っていたように思う。

確かに長槍を持っているのに盾を持っているのはおかしいと思ったが
これはピルムに対応する為では無く最後の突撃の為の
物である訳だろうか
まあ矢張りローマの様にもっと軽装な兵で敵を包囲してから突入した方が
良いのかも知れないが
(しかし後期のローマ兵はプレートや鎖を着ているが
 これは機動力が低下して敵歩兵に包囲を許す可能性は
 無いのだろうか)
378劣等従者:02/12/04 12:30 ID:???
後期ローマ兵は、ガリア人みたいに興奮して裸で暴れるヤシを相手に、近接戦闘をするからでは?
部族単位で襲撃戦闘してくるわけだから、ハンニバルを相手にしたような正規戦ではなかったのでしょう。
379某研究者 ◆NITkxmpUgI :02/12/04 12:33 ID:vUVba8gD
上まあ要するに機動力で敵を包囲してから突撃する事等
頭に無い連中には
軽装兵より重装兵の方が有効だったと言う事だろうか(苦笑
ダキア人の両手剣に対応する為に完全鎧の様な物を着ている
ローマ兵も居ただろうがダキア人の戦術も
ガリア人等と同類である訳だろうか
380劣等従者:02/12/04 12:36 ID:???
ダキア人は、東欧の騎馬系民族では?
だとしたら、山岳や地形を利用した、襲撃こそがやり口では?
トラヤヌスの長城なんかも、侵入そのものを警戒しているようだし。
381某研究者 ◆NITkxmpUgI :02/12/04 13:26 ID:vUVba8gD
まあ自軍を包囲しようとする軽装の敵相手でも
両翼を軽歩兵として包囲の可能性の低い中央の歩兵は
重装とすると言う方向も有るだろうが
戦場では何が起こるか解らぬ部分も有るだろうか
382劣等従者:02/12/04 13:30 ID:???
包囲といっても、相手が正規戦闘に乗ってこなければ、「歴史上のベトナム」みたく小規模戦闘の累積になるはずです。
だとしたら、機動力を優先した機動部隊と、長城を守る部隊に分かれて、別々の装備をするのでは?
唐代の長城警備も、職業軍人指揮する機動部隊(あるいは友好騎馬民族)と、屯田兵の長城警備部隊に分かれていたらしいし。
383某研究者 ◆NITkxmpUgI :02/12/04 13:32 ID:vUVba8gD
敵軽歩兵が上手く動いて味方重歩兵と軽歩兵を分断して来る
可能性は有る訳だろうし其の場合は
重装歩兵が包囲されるか
或いは無理に重装歩兵を守ろうとして軽歩兵が包囲される
或いは分断され各個撃破される可能性等も有るだろうか
384劣等従者:02/12/04 13:33 ID:???
敵の捕捉なしに、重装備歩兵は出せません。
したがって、軽装の歩兵や騎兵による、サーチアンドデストロイが延々行われた可能性のほうが、おそらく高いでしょう。
385某研究者 ◆NITkxmpUgI :02/12/04 13:39 ID:vUVba8gD
城を守る場合は重装歩兵の方が
敵が壁から侵入する時間を稼げる以上機動力は無く共良いのかも知れないが
(まあこれは攻撃側も同様だろうが
 重装備の兵では梯子を上る時間が掛かる危険は有るだろうか)
大量の攻城塔の中に兵士を隠されれば
何処から主力が来るかは不明な場合も有るかも知れぬが
狭い通路等を利用して敵を足止めすれば
重装部隊が来る時間も稼げるだろうか
386劣等従者:02/12/04 13:42 ID:???
>>385
いえ、補給に制限のある襲撃側は、応援の気配があれば退かざるをえないでしょう。
だからこそ、機動部隊に価値があるのです。
そして、騎馬民族の多くが、襲撃と離脱を繰り返した理由でもあるんじゃないかな、と妄想
387某研究者 ◆NITkxmpUgI :02/12/04 13:46 ID:vUVba8gD
>ダキア人は、東欧の騎馬系民族では?

まあダキア人の両手剣ファルクスを防ぐ為に
ほぼ全身に装甲を施した重装歩兵が
使われた様だが
388劣等従者:02/12/04 13:47 ID:???
そうですか。
よー知らんところなので、すみません。
389某研究者 ◆NITkxmpUgI :02/12/04 14:02 ID:vUVba8gD
http://www.pakupaku.com/game/jisyoalt_buki.htm
まあファルクスと言うのはどちらかと言えば両手剣と言うより
トラキアのロンパイア同様薙刀の様な物だろうか
390劣等従者:02/12/04 14:04 ID:???
リンクどうも。

どうやら、馬上刀の系列のようですね。
やはり、山野で襲撃に使われたように思うのですが。
391某研究者 ◆NITkxmpUgI :02/12/04 15:39 ID:L2Q4lFd9
>どうやら、馬上刀の系列のようですね。

まあしかし鐙の無い状態で馬上からこれを両手で振るえるのだろうか
392某研究者 ◆NITkxmpUgI :02/12/04 19:04 ID:L2Q4lFd9
まあ17世紀の騎兵の白兵戦用武装はサーベルだろうが
3クオーターアーマーを着た騎兵をサーベルで倒せたのだろうか
(矢張り馬を狙うか鎧の無い足を狙うと言う方向なのだろうか
 サーベルで殴って動きを鈍くしてから隙間を狙うと言う方向も有るだろうが
 これでは倒すのに時間が掛かるのではないのか)
393劣等従者:02/12/05 11:46 ID:???
>>392
無理でしょう。やっぱ、混乱の中で圧死とか(それは冗談にしても)、取り囲んでぼこぼこ(薩摩ジゲン流式)とかなら、討ち取れるのでは?
実際、日本の合戦で首を取ろうと思ったら、組み討ちをやらねばだめだ、ということになってたし(だから、一番槍などが次善の策として行われたのでは、と妄想)
394某研究者 ◆NITkxmpUgI :02/12/05 12:17 ID:C5dWoNcK
>混乱の中で圧死とか(それは冗談にしても)、

グスタフアドルフが確か其の様な死に方をしたかと思うが
どうだったろうか

まあしかしサーベルの他にメイスの様な物は
17世紀の騎兵は携帯していなかったのだろうか
(17世紀迄モーニングスターが使われたと言う情報も有るが
 これは騎兵に装備されていたのだろうか)
395名無し三等兵:02/12/05 12:19 ID:???
>>391
あぶみの使用無しで剣を振るえたかどうかは、現代人としてはもはやなんとも…

>>394
アザンクールのフランス王だったかもしれません。
396名無し三等兵:02/12/06 10:14 ID:bt6eg6DL
投石器
長弓、石弓
ライフル
ミサイル 

戦場では射程距離の長い武器を持っていた方が有利なんだよな。
アトは命中精度か。
397劣等従者:02/12/06 11:22 ID:???
あとは、発射速度ね。火力とは発射量の二乗だ、と定義する人もいるくらいだから(めっちゃ近代的な発想だけど、ま、わしらも現代人だし)
398名無し三等兵:02/12/06 22:02 ID:xY74A43/
これらのスレにもご意見お願いします。

「歴史上の打撃武器〜槌は無慈悲な武器の覇王」
http://academy.2ch.net/test/read.cgi/whis/1038658668/l50

「『斧マニア』募集」
http://hobby.2ch.net/test/read.cgi/hobby/998299272/l50
399名無し三等兵:02/12/06 23:00 ID:???
一番強いのは兵器を考え作り運用する人間の知恵だね。
400名無し三等兵:02/12/07 00:15 ID:7V5nJBJY
よんひゃく
401名無し三等兵:02/12/13 21:55 ID:RzN3CfZ8
age
402某研究者 ◆NITkxmpUgI :02/12/22 03:30 ID:nJ0Fz65G
まあしかしサーベルが軽量な物でもロングソードより重い
1.7kgも有るのはどういう事なのだろうか
(バックソードやパラッシュは日本刀並みに軽量だが
 騎士はこれだけを持たずサーベルも同時に携帯していた様だが)
レイピアは3クオーターアーマーやハーフーアーマーが廃れた後は
鎧の隙間を貫く必要が無くなったので使用されなく成った訳だろうか
403これからが楽しくなる:02/12/23 19:02 ID:Dw6CVuj7
>>402
某研氏はナベショーの『ドイツ参謀本部』をどのように解釈しておられますか
いま、ナポレオンVSプロイセンのお歴々の部分を読み終わったのですが
404某研究者 ◆NITkxmpUgI :02/12/25 07:33 ID:CsPsIodP
まあ残念乍らドイツ参謀本部は読んでないが
この手のHPに比べて内容はどうなのだろうか

しかし室町時代には脚の後部迄防護する臆病板や佩楯も装備された様だが
これを装備して山を上る事は可能だったのだろうか
(矢張りこれを装備しての山城攻略や山での行動は
 困難だった訳だろうか)
405劣等従者:02/12/25 11:30 ID:???
無理でしょうねえ。
日本の山岳は、傾斜がきついですから。
上ったあとに戦闘準備をすればいいし、第一、森の中の道になりますし。

そういえば、昔、平家物語で木曾義仲の山岳戦を呼んだ記憶が…
406某研究者 ◆NITkxmpUgI :02/12/25 11:40 ID:H1LT4DLG
>無理でしょうねえ。
>日本の山岳は、傾斜がきついですから。

臆病板や足の裏を防護する宝幡佩楯
http://www.hi-net.zaq.ne.jp/osaru/kattyuu.htm#muromatiyoroi
(上参照)
を装備しては矢張り山は上れないのかも知れぬが
40kgの大鎧を装備した場合でも関節の動きは拘束されないだろうが
山を下馬して上るのは矢張りきついだろうか
407某研究者 ◆NITkxmpUgI :02/12/25 11:46 ID:H1LT4DLG
>山を下馬して上るのは矢張りきついだろうか

まあこれが一人では困難なら従者に押して貰う様な事は
無いのだろうか(苦笑
408劣等従者:02/12/25 11:54 ID:???
その辺の、従者の機微は、戦国にかかれた「雑兵物語」にありましたけど、流し読みしたのであまり覚えてないです(検索したら、出てくるかも。文庫で見つけたのでリーズナブルだった)。
もっとも、フル武装で戦場まで移動したかどうかも、良く知らないですけど(すみません

そういえば、室町末期、臆病板をつけたとしても、後ろからふくらはぎを狙った…などというマメ知識を「クイズおもしろゼミナール」なる番組で見た記憶が…(ごくあやふやにありますが…
409某研究者 ◆NITkxmpUgI :02/12/25 11:57 ID:H1LT4DLG
まあ関節の動きを拘束されないタイプのプレートアーマーなら
山も上れただろうが
実戦で使われた物の大半の関節の可動範囲は制限されており
重装歩兵の腕のプレートの部分等も動きは制限されている訳なら
西欧の軍隊は弓兵と軽装歩兵以外は急な山には上れず
山城は正門以外からは攻略不能だった可能性も有るだろうか
(まあ只プレートを捨てるか関節部を改良すれば
 上れただろうか
 或いは梯子を多数並べれば
 長大な急傾斜も上れたかも知れぬが攻城戦は兎も角野戦でこれを行うのは
 きつい場合も有るだろうか)
410某研究者 ◆NITkxmpUgI :02/12/25 12:02 ID:H1LT4DLG
>そういえば、室町末期、臆病板をつけたとしても、後ろからふくらはぎを狙った…などというマメ知識を「クイズおもしろゼミナール」なる番組で見た記憶が…(ごくあやふやにありますが…

まあ臆病板も後方からの斬撃や槍の攻撃に備えたと言うよりは
十文字槍等の攻撃に対応した物或いは飛び道具に対応した物かも知れぬが
(後方の死角からは完全鎧でも着ないと矢張り一撃でやられ得るだろうが)
411劣等従者:02/12/25 12:02 ID:???
実際、胴丸の出現は、徒歩兵の増加によるというのが、日本での定説のようです。
外国の鎧発展史はまったく無知なので知らんのですが。

とはいえ、攻城戦において、フルアーマーの甲冑兵がはしごを上った可能性は小さいと思います。
あれは、一種の突撃工兵と考えるべきでしょうし。

もっとも、下馬した装甲兵による近接戦闘による防御(つまり迎え撃つ場合ですね)では、装甲兵の戦闘力は十分に高かったようです。
とはいえ、総崩れになったり、あるいは相手の勢いが馬鹿みたいに強かったりしたら、圧死というやつも実際に多発してますし…
412某研究者 ◆NITkxmpUgI :02/12/25 12:05 ID:H1LT4DLG
>十文字槍等の攻撃に対応した物或いは飛び道具に対応した物かも知れぬが

長刀の先端部が両刃の物も有る訳だろうし
これで脚を引っ掛ける攻撃に対応した可能性も有るだろうか
413劣等従者:02/12/25 12:11 ID:???
そういう武器は、さらに発展して、たとえば袖がらみ(戦国の武器じゃないですけど、同様の対騎兵兵器があったと思います)のような、ものへ発展していったでしょう。
収束手榴弾で対戦車肉薄戦闘をしていた歩兵に、のちに吸着地雷が配布されるような感じで。
騎兵を引きずり落とすことは、対騎兵戦闘の本筋です。
騎乗のまま、弱点をつくようなことは、相手が油断したり、陣形を乱したときにしか通用しませんし。
414某研究者 ◆NITkxmpUgI :02/12/25 12:13 ID:H1LT4DLG
>とはいえ、攻城戦において、フルアーマーの甲冑兵がはしごを上った可能性は小さいと思います。

まあ関節の動きを制限されているプレートアーマーでも
山の急傾斜は兎も角梯子は多分上れるかとは思うが
矢張り梯子を上るのはフルプレート等は着れぬ雑兵の仕事である訳だろうか(苦笑
(まあ只重装騎兵も攻城戦を只見ていた訳では無いだろうし
 門から・或いは城壁をカタパルト等で崩した場所からは突入した場合も有る訳だろうか
 しかしトンネルからの突入は雑兵のやる事として無い訳だろうか(苦笑
 攻城塔からの城内突入と言うのは重装騎兵は行った訳なのだろうか
 或いは城壁上での戦闘には城内の重装騎兵は加わったのだろうか)
415劣等従者:02/12/25 12:17 ID:???
塩野七生のロードス島攻防記によれば、城壁に重装の甲冑兵を配置して、準備砲撃(青銅の攻城砲だそうな)のあとで突撃してくるトルコ兵を迎え撃っておりました。
ま、彼ら聖ヨハネ騎士団は宗教騎士団ですから、宗教的正義と、彼らの貴族的プライドに支えられた戦いでもあったようですけど。

個人的には、重装騎兵は城攻めを見てただけでは、と思ってますけど。
416某研究者 ◆NITkxmpUgI :02/12/25 12:20 ID:H1LT4DLG
>騎兵を引きずり落とすことは、対騎兵戦闘の本筋です。

まあこれは騎兵を狙って落とすよりも
馬を潰して落とした方が早い場合も多い訳だろうか


>騎乗のまま、弱点をつくようなことは、相手が油断したり、陣形を乱したときにしか通用しませんし。

まあ騎兵の槍が槍歩兵に届く前に歩兵の長槍が馬に届く訳だろうか
(只西欧の片手槍と盾を持つ槍兵では騎兵のランスが先に
 歩兵に届く危険も有るだろうし
 盾を構えていてもランスで貫かれる或いは衝撃で倒される危険は有るだろうか)
417劣等従者:02/12/25 12:24 ID:???
将を射んとすればまず馬を射よ、って奴ですな。
騎兵を脱落させると、騎兵の持っていた突撃衝力が失われますから、それでミッションキル的な意味が果たせるかもしれません。
従者が集まってきて、守ろうとしたり、あるいは騎兵部隊がナウシカの3巻みたいに損害無視で突撃してきたら、大変ですけど。

敵も方形陣などで抵抗してくるはずですから、側面へ回り込んだり、あるいは敵陣形を引っ張って崩すような機動をして、突撃側を有利にしてから、満を持して突撃したいのでは、と思いますが。
418某研究者 ◆NITkxmpUgI :02/12/25 12:26 ID:H1LT4DLG
>城壁に重装の甲冑兵を配置して、準備砲撃(青銅の攻城砲だそうな)のあとで突撃してくるトルコ兵を迎え撃っておりました。

まあトルコ兵も重装の兵が多いし(歩兵は西欧の兵より平均的には重装備かも知れぬが)
斧やメイスも携帯していたのでかなり苦戦を強いられていたのではないのか
419名無し三等兵:02/12/25 12:28 ID:???
軽装備だったようです。損害比はかなり開いていたような(失念失礼)。
10倍どころか、100倍近くの兵力を集めてきて、かつ砲撃しても「講和による退去」で決着がついてます。
ロードスは。
420某研究者 ◆NITkxmpUgI :02/12/25 12:36 ID:H1LT4DLG
>あるいは騎兵部隊がナウシカの3巻みたいに損害無視で突撃してきたら、大変ですけど。

まあ歩兵の段列にも限りは有るだろうし
強行突入され陣形の背後や中空の方陣内部に突入されれば
きつい場合も有るだろうか


>敵も方形陣などで抵抗してくるはずですから、側面へ回り込んだり、あるいは敵陣形を引っ張って崩すような機動をして、突撃側を有利にしてから、満を持して突撃したいのでは、と思いますが。

まあ方陣を組んでいるのが雑兵では
全方向から騎兵や歩兵に同時に突入されると
下手をすると逃げ場が無くなると言う事で
恐慌を起こす可能性と言うのも否定出来ないだろうか
(矢張り騎兵や歩兵が同時に全方向から突入して来るのを見ただけで
 恐慌に陥る危険も有るだろうか
 まあ敵を直接見ず共背後や側面の山影等から同時に敵の音がしただけで
 恐慌に陥る危険も場合に拠っては有るだろうか)
421劣等従者:02/12/25 12:39 ID:???
全方向からは、突っ込まないでしょう。アレキサンダーの例に習わずとも、一翼側から崩せばよいわけで。
伝令の手段が限られている当時、方陣の側面に騎兵や、軽歩兵が置かれるのは、軍事的に重要だったのでしょう。
だからこそ、騎兵・重装歩兵比率みたいなものが、盛んに論じられたのだと思ってますけど。
422某研究者 ◆NITkxmpUgI :02/12/25 12:40 ID:H1LT4DLG
>軽装備だったようです。損害比はかなり開いていたような(失念失礼)。
>10倍どころか、100倍近くの兵力を集めてきて、かつ砲撃しても「講和による退去」で決着がついてます。

まあ軽装備でもメイスや斧を持っていれば
数で攻められた場合プレートアーマー装備時の疲労の問題も有るだろうし
きつい部分は有る訳だろうか
423劣等従者:02/12/25 12:44 ID:???
塩野女史の歴史的正確性には、疑問がついてますけど、彼女の描写によれば、聖ヨハネ騎士団の疲労と、消耗も大きかったようです。
なにしろ、接近戦だけでなく、城の補修もしなければならない、地中海の暑い日差しの下で、というわけで。
ダニガンの出している自然損耗率から推定すれば、限られた軍事要員で守る防御戦としては、限界まで戦ったのでしょう。
424某研究者 ◆NITkxmpUgI :02/12/25 12:47 ID:H1LT4DLG
>全方向からは、突っ込まないでしょう。アレキサンダーの例に習わずとも、一翼側から崩せばよいわけで。

まあ歩兵が歩兵を包囲した例は有るだろうが
騎兵が全方向から同時に歩兵に突入したと言う例は
有るのだろうか
(下手をすると恐慌を起こし方陣の内側に皆逃げて圧死と言う可能性も
 有るだろうか
 弓兵に包囲された圧死した例が確か有っただろうか
 少数の精鋭部隊に大部隊が包囲された場合でも
 歩兵がじりじりと方陣の内側に後退していき
 逃げ場が無くなって圧死と言う可能性も有るだろうか
 まあ大部隊の外周部に精鋭兵を置いておけばこの様な状況に成る可能性は
 低いだろうか)
425名無し三等兵:02/12/25 12:52 ID:???
アサンクールは、弓兵で阻止、反撃、圧死だったと思います。
ハンニバルはガリア人を陣形の真中に置いたようです。たしかカンナエでも前進させたガリア兵が圧迫されるのを読んで、それで殲滅の罠をつくっていたんじゃなかったかしらん?
そういえば、ローマがガリアと戦ったときにも、しばしば戦いに夢中になるガリア人を包囲、戦意喪失、虐殺的殲滅などということがあったそうな。
426某研究者 ◆NITkxmpUgI :02/12/25 12:58 ID:H1LT4DLG
>カンナエでも前進させたガリア兵が圧迫されるのを読んで、それで殲滅の罠をつくっていたんじゃなかったかしらん?

まあ矢張り戦術的後退を演技として見抜かれぬ為に
敢えて雑兵であるガリア人を中央部に置いたと言う事なのだろうか
(まあ雑魚であるガリア人が後退しても誰も演技とは思わないだろうし
 雑兵にもこの様な利用価値が有ると言う事だろうか(苦笑
 中央部が弱いと言うのも両翼を強化したと敵に解釈されるなら
 不自然では無いと思わせる事も可能だっただろうか(笑)
427名無し三等兵:02/12/25 13:00 ID:???
戦場で、敵の戦意と錬度を見抜くのは、多分難しいのでしょう。
「さあやるぞ!」「がんがれー!」とたきつけて歩けば、戦意万全に見えますし。
それに、ガリア人は蛮人と見られてましたから。
428名無し三等兵:02/12/25 13:02 ID:gr89PBli
古代から近代とは、幅が広すぎる。
429名無し三等兵:02/12/30 18:39 ID:9bqRSfhZ
すいません。
ヴォーバン式以降の西洋の城郭が、
どう発達したかご存知の方はいらっしゃいませんか?
430名無し三等兵:02/12/31 11:01 ID:???
セヴァスティヤン・ル・プレストル・ド・ヴォーバン Sebastian Le Prestre de Vauban) 1633-1707

フランスはルイ十四世治世下で、ル・テリエの後を引き継ぎ、各種城塞と攻城に勤しみ、多大な貢献を果たしたヴォーバン元帥。技術将校として、この地
位に登りつめた人物は始めての者だったという。

1633年 サンレジェにて元貴族の貧家に生まれる。10歳にして両親を失った彼は、小さいときから軍人を志していたという。首尾よく軍隊入りを果たした彼
は、数学や製図、築城に勝れた才能を示し、それがやがて認められて有数の築城家になった。
1651年 コンデ公の軍に士官候補生として入隊。
1653年 フランス国王の軍隊に入隊。シェヴァリエ・ド・クレルヴィルの下で軍事技術を学ぶ。
1655年 王室侍従技術官に任命される。マレシャル・ラ・フェルテ連隊の歩兵中隊大尉。
1659〜67年 主に王国内の要塞の修理、改善に従事。
1667年 陸軍省の技術的業務を司る総監に就任。
1673年 マーストリヒトにて初めて「平行壕」を用いる。
1684年 ルクセンブルク攻囲戦にて「騎兵式胸壁」を初めて用いる
新要塞の構築 33回
旧城砦の修築 300余
攻囲を指揮すること53回
この功績を認められて、
1703年 元帥の称号が与えられた。これを機に引退。
ルーヴァアの進めた軍制改革により、その昇進を獲得したとも言える。ルーヴァア元帥は父のル・テリエの改革の流れを引継ぎ、その死の1691年までに、
フランス軍を無秩序な群集から、近代陸軍にまで仕立て上げていた。
431名無し三等兵:02/12/31 11:02 ID:???
1703年 「余暇」出版。
1705年 「攻囲論」出版。

城塞の構築に、敵の要塞の粉砕にフランス全土を歩いたヴォーバンの目には、貴族ら特権階級の優遇と、庶民の苦しみ喘ぎとが全く矛盾することに気づ
いたのである。
「王国十分の一税(案)」の提言。
国民の大多数が貧民と化しているとの結論に達したヴォーバンは、不公平で非効率で複雑な租税制度の改革こそ焦眉の急であるとの結論に達した。
1688年 執筆開始
1699年 執筆完。財務総監に提出するも実行不可能。
1704年 再度提言。が受け入れられず。
「王国十分の一税」秘密出版に踏み切る。友人の間に密かに配布。まもなく宮廷やパリで大きな反響を呼び起こした。
1707年2月 著書の没収が命ぜられる。
1707年3月には、著者の起訴が決定される。ヴォーバンは病床にあったが、この結論の報告が病状を悪化させた。
同年3月30日息を引き取った。
「良心に反しては何事もなさなかった」との一言を残してだ。

こういう理数的なタイプで且つ憂国の士が存在しないのも現代日本の抱える問題の一つなのかもしれない。日曜朝の文系的アバウト論議には正直辟易
する。且つこういう専制君主時代に国策に疑問をもって、具体的な行動に出てしまうというのは。。意外な感じもするし、科学者的純粋な精神からの行動で
あったのかもしれないな。

あ、ヴォーバンのページあります(海外)。
ご参考に。
http://www.geocities.com/Pentagon/6750/vauban_frame.htm
432名無し三等兵:02/12/31 11:03 ID:???
むろんコピペだから、その後の展開はない。
433某研究者:03/01/04 13:55 ID:HYLTJy4d
まあ17世紀の兵士の服がだぶ付いているのは
レイピアの突きを引っ掛ける為だろうし
これに対して突きを不用意に掛けるのはどうかだろうが
(完全鎧が廃れたからでは無くこれの登場でレイピアが廃れ
 サーベルが主体と成った訳だろうか)
434名無し三等兵:03/01/09 23:11 ID:???
  
435山崎渉:03/01/13 04:28 ID:???
(^^)
436名無し三等兵:03/01/14 21:37 ID:???
世界史板
http://academy.2ch.net/test/read.cgi/whis/1042542633/
■イギリス軍事・外交史■
437某研究者:03/01/20 16:26 ID:j1cL8bNG
まあしかしプレートアーマーと言うのは片手剣の打撃を
完全に止められる物なのだろうか
(片手用メイスや斧の一撃でも大概は骨折や戦闘不能とは
 成らぬ訳なのだろうか)
438名無し三等兵:03/01/20 22:01 ID:???
>>430-432
遅くなりましたが回答ありがとうございます。
その後の展開はないと言われてこういう事を書くのはなんですが、
ヨーロッパでこういうヴォーバン式をはじめとする
稜堡式の城郭が最後に築城されていたのは、いつ頃なのでしょうか?
(少なくともナポレオンの時代には時代遅れだったそうですが。)

それとこの後の世代の城郭がどのように発達したかを教えていただけないでしょうか?
(某研様もご存知なら回答して頂けると…)
439山崎渉:03/01/22 11:16 ID:???
(^^;
440名無し三等兵:03/01/22 12:54 ID:???
誘導あげ
441某研究者:03/01/22 13:17 ID:A0FoNtnT
まあロングボウ兵もポワティエでプレートの完全鎧の前に
威力は減殺されており
薔薇戦争では余り役には立たなかった訳だろうか
(強力なクロスボウの至近射撃の方が
 完全鎧に対しては未だ役に立った可能性も有るだろうか)
ロングボウ兵にはパビスは無いが
クロスボウ兵には有るなら矢に対する防御力は
クロスボウ兵の方が上だった可能性も有るだろうが
重装歩兵のパビスの背後にロングボウ兵が
隠れていた可能性も有るだろうか
442劣等従者:03/01/22 13:19 ID:???
クロスボウの弱点は、連射速度が遅いことですからね。
攻城戦では、敵側の攻撃通路が限られているので、多大な役割を果たしましたけど、野戦では駄目だ、とフランス人が言っていたような…
443某研究者:03/01/22 13:40 ID:A0FoNtnT
>野戦では駄目だ、とフランス人が言っていたような…

まあしかし彼等の携帯しているパビスでロングボウを防げないのかと言う事だが
ロングボウ兵はパビスが無いので矢の攻撃には
脆いのかも知れぬが
ポワティエでロングボウ兵が下馬騎兵の突入を許したのも
仏騎兵のプレートアーマーが完全鎧と成った所為だろうか
444名無し三等兵:03/01/22 13:53 ID:???
http://www1.kcn.ne.jp/~typezero/rd/cavalry.html
誰のサイトか知らないけど、騎兵についての記述。
ちょっとおもしろかったんで。
445某研究者:03/01/22 14:04 ID:A0FoNtnT
まあストックの無い火縄銃と
ストックの有る火打石式のライフル銃の命中精度は
何れが高いのだろうか
(ストック装備の火縄或いはホイールロック式の
 銃の精度は火打石式のライフル銃よりも
 高い可能性は有る訳だろうか)
446劣等従者:03/01/22 14:06 ID:???
日本の火縄銃がストック無しだったのは、たまたま最初に手にした火縄銃が、狩猟用(?)の変り種だったからだそうですけど。
447某研究者:03/01/22 14:11 ID:A0FoNtnT
上まあ只ストック装備の物は全く無いのかと言う事だろうが
西欧も狙撃兵は精度の高いホイールロックか
火縄式の銃を用いるべきだったのではないのか
448劣等従者:03/01/22 14:13 ID:???
現存する古式銃は、精度の高いものと、低いものがあるそうな。
戦列用と、狙撃用かもしれません。
449某研究者:03/01/22 14:18 ID:A0FoNtnT
明・朝鮮軍のクロスボウや弓も鉄砲でアウトレンジされていたのか
大砲の葡萄弾で銃をアウトンレンジすると言う事は
明・朝鮮側はしなかったのだろうか
450劣等従者:03/01/22 14:19 ID:???
当時の大砲の、リアクション能力は低いので、狙撃的対処は困難でしょう。
451某研究者:03/01/22 14:23 ID:A0FoNtnT
まあしかし一分間に3発程度は撃てたかも知れないが
明や朝鮮の大砲の連射能力はどの程度だったのだろうか
(まあ只何回も撃つと砲身が保たず破裂していたと言うが
 どうなのだろうか)
452某研究者:03/01/22 14:30 ID:A0FoNtnT
まあ明・朝鮮軍も攻城戦時以外は銃でやられたと言うより
日本の長槍兵にやられたと言う側面が多い訳だろうか
(明・朝鮮軍には両手持ちの長槍等は
 無い訳だろうし片手槍+盾だろうから
 盾で全身をカバー不能なら長槍で一方的にやられていた可能性も
 有るだろうか
 下手をするとこの種の短槍兵では槍騎兵を阻止出来なかった可能性も
 有るだろうか)
453某研究者:03/01/22 14:47 ID:A0FoNtnT
まあ明・朝鮮の歩兵は盾は有るが
装甲は薄いので日本刀で斬り込まれても
脆い部分は有る訳だろうか
454某研究者:03/01/23 11:49 ID:ETC12COk
まあ補給用の馬車の馬を弓や銃で狙えば
少数の兵でも敵の補給をかなり遅滞させる事は出来る訳だろうか
(まあ只騎兵の軍馬を輸送用に使われる可能性も
 有るだろうか)
火矢や銃で弾薬を狙うのは兎も角
火矢で荷物を狙うと言うのは荷物は余り燃えず
余り効果は無い訳だろうか
455某研究者:03/01/23 13:20 ID:ETC12COk
日本の最初の槍と言うのはモンゴルの槍の模倣であるなら
これは片手で扱う短い物だったのか
しかし日本の足軽はこれを両手で用いていたのだろうか
(投げ槍等はモンゴルから入らなかったのだろうか
 或いはモンゴルは通常の槍を投げていたのだろうか)
槍が後で長く成ったのは西欧のパイク兵や矛槍兵の影響なのか
或いは独自に集団戦での教訓等を元に進化したのだろうか
456名無し三等兵:03/01/24 04:30 ID:Sm6EAUZA
ナポレオン戦争当時のマスケット銃の、
実用(?)最大発射速度はどれくらいだったんでしょうね?
457某研究者:03/01/24 06:46 ID:Qljo7xnx
http://www.sanynet.ne.jp/~imperial/Napoleon/weapon/weapon1.html
まあ5発程度撃てるとの意見も有るが
どうなのだろうか
(マスケット銃も連続で12発以上撃つと銃身が加熱すると言うが
 小便や水の量にも限度は有るだろうし(苦笑
 まあ歩兵や騎兵は連続で12発等撃つ前に恐らく突撃するだろうが
 銃撃主体の狙撃兵等が連続して銃撃を行いたければ
 複数の銃或いは複数の銃身を持つ銃でも持つしか無いだろうか)
458某研究者:03/01/24 06:56 ID:Qljo7xnx
まあ上を見る限りマスケット銃は不発や兵のやる気の無さ等も有り余り役に立たないが
大砲は役に立ったと言う認識で良いのだろうか
(大砲はマスケット銃に比べてかなり連射が可能なのだろうか)
459某研究者:03/01/24 07:00 ID:Qljo7xnx
まあ確かに歩兵はやる気は無いのかも知れぬが
砲兵や騎兵の戦意は低かったのかと言う事だろうし
彼等は全員砲や銃を発射していた訳だろうか
(少数の騎兵や砲兵に歩兵が殲滅される事も
 多く騎兵を阻止したのは歩兵では無く騎兵・下馬騎兵や
 砲兵だったと言う事なのだろうか)
460某研究者:03/01/24 11:42 ID:lTTei59W
http://www.sanspo.com/geino/top/gt200301/gt2003012411.html
まあしかしこのバスタードソードの長い鍔と言うのは
剣を振り回す時に邪魔には成らないのだろうか(苦笑
461456:03/01/24 18:54 ID:LjUDRTvk
おお、某研閣下のレスが。光栄であります。
>>457
マスケット銃でも大砲同様、
一発発射するごとに銃身内の清掃をせねばならなかったのでしょうか?
これを怠るとまだ銃身の奥でくすぶっている火薬に、
次に注いだ装薬が引火してしまうとか、あったんでしょうかね。

時間がないのでここまで。また書き込ませていただきます
462456:03/01/25 00:39 ID:1LAQNVbL
大砲は砲員(砲手?)が専門教育を受けていたこと、
複数の人員で一つの砲を操作するために、
発砲準備が砲の大きさの割りに簡便だったということでしょうかね?

歩兵の士気が、必ずしも騎兵より格段に劣っていたということは
ないかと思われますが……(根拠のない考えですが)。
463某研究者:03/01/26 15:58 ID:h/oGttN1
まあ戦国時代の日本刀の質が悪いと言っても
騎馬武者の其れの質は高いだろうし
徒歩武者の其れも高かった可能性は有る訳だろうか
464某研究者:03/01/26 16:14 ID:h/oGttN1
>歩兵の士気が、必ずしも騎兵より格段に劣っていたということは
>ないかと思われますが

まあ現実に半数以上の歩兵のマスケット銃が発射されない侭で有った事も有るだろうし
彼等も包囲されてさえ強行突破はせず
圧死と言う様な事も起こり得るだろうか
465名無し三等兵:03/01/26 18:41 ID:???
>>460
鍔持って振り回すって方法もあったと思うんですが…
違う剣でしたっけ?
466某研究者:03/01/31 14:45 ID:KB6zvINh
>鍔持って振り回すって方法もあったと思うんですが

まあこれはツヴァイハンダーかと思ったが
あれの鍔も正直邪魔に成る場合も有るだろうし
鍔を持って振り回すと言うのも果たして実戦でどれだけ
使えたのかと言う事だろうが
467某研究者:03/01/31 14:47 ID:KB6zvINh
まあしかし日本にモンゴル侵攻迄槍が入っていなかったのかと言う事だが
古代に中国から槍は入っていなかったのだろうか
(一応は入って来たが鎌倉時代には弓や長巻が主力と成ったので
 廃れたと言う事なのだろうか)
468某研究者:03/02/01 15:18 ID:4a5FEMP7
まあ良く小説等で敵の軍の使者を交渉決裂等で斬るシーンが有るが
これは捕らえた後では無くその場で斬る場合も有るだろうが
その場で斬る場合は矢張り一撃で仕留めないと格好が悪いが
敵の反撃も有るだろうし現実には多数の兵で仕留めていた訳だろうか(苦笑
469名無し三等兵:03/02/01 17:09 ID:???
>>468
微妙に異議あり。
対陣中でなければ、やっぱり縄で絞殺じゃないのか?
後始末大変だし。
470某研究者:03/02/01 21:33 ID:4a5FEMP7
まあ無理に捕らえようとすると味方が傷付き得るだろうし
矢張り其の場で人数を掛けて槍等で始末だろうか(苦笑
471某研究者:03/02/05 06:34 ID:onGtqJOT
割に軽く攻撃パターンの多い十文字槍を短くして両手剣並とすれば
自分より高速の相手に踏み込まれても副武装を使わず共
対応は可能だろうか
(十文字槍も鍵型の刃の付いた物なら装甲の有る相手を引っ掛けて倒してから
 装甲の隙間を突く様な事も可能だろうし
 装甲の有る者と無い者双方に対応可能だろうか)
472某研究者:03/02/05 07:24 ID:onGtqJOT
まあ長巻程度の長さの十文字槍を作れば
踏み込まれても対応は可能だろうか
473某研究者:03/02/05 07:28 ID:onGtqJOT
まあ後はソケット部分を長くして
敵の両手剣の攻撃でも柄の部分を切断されぬ様に
すべきだろうか
474某研究者:03/02/05 14:20 ID:3eymEnL+
まあ孔明も10000のクロスボウ兵を持っていたと言うが
これは100000の蜀軍の中の数だろうか
(弓兵もクロスボウ兵だけでは無いだろうし
 通常の弓兵も恐らくクロスボウ兵より数は少ないが
 有っただろうか)
弓兵が全軍の1/5程度と言うのは日本と同程度だろうが
モンゴル軍に関してはどうだったのだろうか
(まあ恐らくイングランドの様に弓兵が全体の2/3等には
 成らないだろうか(苦笑)
475某研究者:03/02/05 14:23 ID:3eymEnL+
>弓兵が全軍の1/5程度と言うのは日本と同程度だろうが

まあこれは戦国時代の比率で有って
騎馬弓兵が主体である鎌倉時代は
恐らく弓兵の比率はイングランド並みとは言わぬ迄も
かなり高かった(恐らく1/3以上は有るだろうか)訳だろうか
(まあ只モンゴル軍より多いかどうかだが)
476某研究者:03/02/05 16:16 ID:3eymEnL+
まあしかしクレッシーの仏のクロスボウ兵の数と言うのは
英弓兵より確か多かったかも知れぬが
仏の弓兵の比率はどの程度なのだろうか
477某研究者:03/02/05 16:31 ID:3eymEnL+
まあ中国のクロスボウがモンゴル騎兵や歩兵の
弓をアウトレンジする可能性も有ったのかも知れぬが
数が無いので踏み込まれれば弓でやられていたのだろうか
478某研究者:03/02/05 16:33 ID:3eymEnL+
十字軍歩兵のクロスボウがアラブ軍の弓を
アウトレンジしたとの記述も有るが
現実には踏み込まれてやられており
アルスーフ勝利は馬車やパビスを盾にした故なのだろうか
(弓が通用せず突入を焦ったアラブ槍騎兵は
 十字軍のクロスボウの攻撃後の騎兵突撃で粉砕されたのだろうか)
479某研究者:03/02/11 16:24 ID:mI0na/jL
まあ同じ国の傭兵同士が戦った事と言うのは
有るのかと言う事だが
例えばランツクネヒト対ランツクネヒト・
スイスのパイク兵同士が敵味方に分かれて戦った事は有ったのだろうか

まあしかしパイク兵の密度と言うのはどの程度の物なのか
マケドニアファランクスの様な物なら全く隊列を崩さずに
ランツクネヒトや円盾兵が乱入する事は困難であるかも知れぬが
480某研究者:03/02/13 18:20 ID:AeJtR7WY
まあ騎士の扱う重い武器(特にメイスやハンマー・斧・両手剣等)は防御面で問題が有り
敵の打撃を武器で防ぎ難く成る訳だろうか
(重い甲冑を装備する或いは馬上での戦闘では
 敵の攻撃を武器を使わずに回避するのは
 困難だろうし武器を使って防御が出来ねば
 致命的な事に成る訳だろうが)
重いハルバードや長刀もスピア・槍に比べ防御面では
問題は有る訳だろうか


まあ只槍で完全鎧を来た騎士を倒せるのかと言う事は
有るだろうし鉤等を装備した槍で転倒させてから
鎧の無い部分を貫くと言う方向も有るだろうが
これは困難であり矢張り鎧の上から
斧より軽量な長刀の様な物で殴るのが
一番良い訳だろうか
(まあ只長刀を持つ自分に鎧が無く相手に鎧が有れば
 自分が踏み込まれて攻撃される可能性は低いから
 重い武器でも良い訳だろうが)
481某研究者:03/02/13 18:21 ID:AeJtR7WY
>(まあ只長刀を持つ自分に鎧が無く相手に鎧が有れば
> 自分が踏み込まれて攻撃される可能性は低いから
> 重い武器でも良い訳だろうが)

まあ只早目に攻撃を当てて始末したいなら
矛槍より軽量な長刀の方が良いだろうか

まあ軽装な相手には長柄武器は槍兵に突入する円盾兵やローマ兵の様に
踏み込まれるリスクが有るから
使うべきでは無いとの意見も有るだろうが
(まあ只騎兵を阻止するのには槍は有効だろうが
 パイクの様に長い槍で無く共良いだろうか)
482某研究者:03/02/13 18:21 ID:AeJtR7WY
まあ矢張りサップの様にパワートスピードの有る人間に
軽い武器を振り回されれば完全鎧を着てさえ
スピードの無い人間がこれを防ぐ事が
出来るのかと言う事だが(苦笑
483某研究者:03/02/13 18:43 ID:AeJtR7WY
両手で振るう軽い(0.7−0.8kg)日本刀の斬撃のスピードに
片手剣(1.5kg)やバスタードソード(2.5kg)を振るう
西欧の騎士が付いてこれるのかと言う事だが
馬上では剣は片手で振るわれる場合も多い訳だろうし
0.8kgの剣の片手での斬撃ではプレートを来た騎士相手に
ダメージは与えられぬかも知れぬし
有功打を与えるには両手で振るう必要も有るだろうか
(まあ馬上では剣を両手で扱い難いなら片手で斬撃して敵の体制を崩した後で
 両手で打撃を与えると言う方向も有るだろうが)
484名無し三等兵:03/02/13 19:01 ID:???
>某研究者さん
ご存知かもしれないですけど、学研から、
中世の戦略戦術兵器シリーズがでてますよ。
参考になるかと。
485某研究者:03/02/13 19:05 ID:AeJtR7WY
まあ長刀を持った女が竹刀を持った男に勝ったと言う事も有る様だが
競技用の長刀は650g以上だが
現実の長刀の重量は2.5〜5kgであるし
(実物の刀は竹刀510g以上に対し700−800gだろうから
 余り変わらぬ訳だろうか)
女が競技と同じ様に素早く振るえる物では無いだろうから
余り参考には成らないだろうか
(長刀では無く槍なら1kg程度の物も
 実際に有るだろうが
 相手の前進する速度は自分が後退する速度より大概は速いだろうし
 槍の間合いより内側に踏み込まれれば厳しい場合も有るだろうか

矢張り現実の刀の重量の半分は有る竹刀と
現実の長刀の1/4程度しか無い重量の競技用の長刀を用いて
どちらが強いかを論じてもどうかだろうが(苦笑
競技では余り重い武器は防具が有ろうと危なくて使えないだろうが
軽い競技用の槍と竹刀で勝負したら大体実物の強弱と同じに成るだろうか
486某研究者:03/02/13 19:07 ID:AeJtR7WY
兎も角競技用の槍を持つ女と竹刀を持つ男が戦ったら
どちらが勝つのかと言う事だが
槍も攻撃パターンの多い十文字槍を使えば
女が勝つ可能性も高いのだろうか
(仮に踏み込まれても十文字槍で首を後ろから掛ける等すれば
 勝てるかも知れぬが万一外した場合は
 恐らくアウトだろうか(苦笑)
487某研究者:03/02/13 19:10 ID:AeJtR7WY
>ご存知かもしれないですけど、学研から、
>中世の戦略戦術兵器シリーズがでてますよ。

まあこれは最近出た物なのだろうか
そうであれば過去のこの手の物に比べて新たな内容は増えたのだろうか
488名無し三等兵:03/02/13 19:12 ID:???
十字でなく普通の槍でも、槍を持っている方が圧倒的に有利では?
競技として行うとなると、有効打突部位が限られるから微妙かもしれんが。
489某研究者:03/02/13 19:15 ID:AeJtR7WY
>十字でなく普通の槍でも、槍を持っている方が圧倒的に有利では?

まあ只パイク兵や矛槍兵を崩した剣と盾を持つ円盾兵の存在も有るだろうし
人間が後退する速度と言うのは前進する速度よりは遅い訳で
槍を持つ人間は刀を持つ人間に踏み込まれると簡単にやられる危険も有るだろうが
490某研究者:03/02/13 19:20 ID:AeJtR7WY
全身に鎖帷子でも着ていれば十文字槍の引いて斬る攻撃等は
問題では無いだろうが移動速度が鈍り
槍を持つ相手に踏み込めない可能性も有る訳だろうか
491某研究者:03/02/13 19:23 ID:AeJtR7WY
>まあ只パイク兵や矛槍兵を崩した剣と盾を持つ円盾兵の存在も有るだろうし

他にも剣と盾・投げ槍を持つレギオンが槍を持つギリシャファランクスを
倒したと言うのも有るだろうし
単純に槍が剣より強いとは言えないだろうが
騎兵は矢張り槍で無いと阻止し難い部分は有る訳だろうか
492名無し三等兵:03/02/13 19:31 ID:???
強力な射撃兵装を使わずに騎兵を相手にする場合、
突っ込まれて蹂躙されないよう槍衾で接近を防ぐしかないのではないかな。
そうでなければ、障害物を作るしかないだろうね。
493名無し三等兵:03/02/13 19:36 ID:???
太平記の記述には、楠正成が連結可能な盾を作って騎馬武者を撃破した
なんて記述もないわけじゃないが、眉唾ものだね。
背後にまわりこまれたら、瞬殺だし。
大体、日本の騎馬武者を本格的な騎兵と同列に並べるわけにもいかんだろうし。
494某研究者:03/02/13 19:44 ID:AeJtR7WY
>太平記の記述には、楠正成が連結可能な盾を作って騎馬武者を撃破した

まあこれは柵の様な物であり騎兵を足止めして
其の間に射撃を加える為の物だろうか

>背後にまわりこまれたら、瞬殺だし。

まあ騎兵に背後や側面に回り込まれない様な地形に布陣するか
騎兵に対する方陣や八陣の様な陣形を組んでおれば良いだろうか
(テルシオの様な方陣はこの時代には無かっただろうが
 同様の陣形である諸葛の八陣の様な物が輸入或いは
 独自に考案されていた可能性は有っただろうか)
495某研究者:03/02/13 19:47 ID:AeJtR7WY
>強力な射撃兵装を使わずに騎兵を相手にする場合、
>突っ込まれて蹂躙されないよう槍衾で接近を防ぐしかないのではないかな。

まあ騎兵ではこれを崩せないなら射撃
(まあパビスでも装備されていれば砲撃以外では崩せないだろうが)
或いは円盾兵や下馬騎兵・両手剣を持ったランツクネヒトの突入で
崩せるだろうか
(パイク兵等は矛槍兵や短槍兵に踏み込まれても
 厳しいだろうが矛槍兵が短槍と盾を持つ短槍兵に踏み込まれた場合
 これは撃破され得る訳だろうか)
496某研究者:03/02/13 19:49 ID:AeJtR7WY
>或いは円盾兵や下馬騎兵・両手剣を持ったランツクネヒトの突入で
>崩せるだろうか

まあしかしスイスのパイク兵と両手剣を持った独のランツクネヒト
何れが強かったのかと言う事だが
以外にランツクネヒトの方が騎兵は兎も角パイク兵相手には
強かったと言う事は無いのだろうか
497某研究者:03/02/13 19:50 ID:AeJtR7WY
>厳しいだろうが矛槍兵が短槍と盾を持つ短槍兵に踏み込まれた場合
>これは撃破され得る訳だろうか)

まあ只短槍兵では槍騎兵を阻止できるのかと言う問題は有るだろうし
矛槍程度の長さの槍で無いと騎兵を阻止し難いと言う事は
無いのだろうか
498某研究者:03/02/13 20:05 ID:AeJtR7WY
まあしかし日本の騎馬鉄砲隊の銃は役に立ったのに
西欧のカラビニエや竜騎兵の銃が役立たずであると言うのは
良く解らない事だが
矢張りピストル或いは騎兵銃を確実に当てられる近距離で馬にでも撃てばかなりダメージを与えられる様にも
思える訳だが
499イプスィロン:03/02/13 20:10 ID:7BdFqH1n
短い槍で騎兵に突撃されたら劣勢でしょう。(勢いが違う
まあ、槍は何故、騎兵に対して効果があるのは、
駱駝と同様馬が(槍の穂を見て)突進しなくするためです。
勢いがなければ騎兵も歩兵と同類ですからね。
海戦で(大航海時代から蒸気船誕生まで)
大砲の威力が最も強力な船はなんだったのでしょうかね?
やっぱり戦列艦っていう船だったのかな?
500某研究者:03/02/13 20:12 ID:AeJtR7WY
>まあ明・朝鮮軍も攻城戦時以外は銃でやられたと言うより
>日本の長槍兵にやられたと言う側面が多い訳だろうか

まあ彼等も軽装であるなら移動速度を利用して
長槍の間合いの内側に踏み込めば良かったのかも知れぬが
踏み込んだとしても日本刀の抜刀速度は早く
踏み込んだとしてもこれで防護され攻撃される可能性が有る訳だろうが
軽装である彼等は日本刀の切れ味を恐れて踏み込めず
其の間に槍でやられていたと言う可能性は有る訳だろうか
501某研究者:03/02/13 20:22 ID:AeJtR7WY
>まあ、槍は何故、騎兵に対して効果があるのは、
>駱駝と同様馬が(槍の穂を見て)突進しなくするためです。

まあ歩兵の長槍であれば騎兵の槍が当たるより先に
馬に槍を当てられると言う事も有るだろうが
短槍では騎兵の槍が先に歩兵に当たる可能性も有る訳だろうし
馬に片手持ちの槍を当てたとしても其の侭突入され
密集隊形では馬を回避する訳にも行かず押し潰される危険も有る訳だろうが
(まあ盾を捨てて槍を両手で構えれば阻止は可能かも知れぬが
 矢張り槍が短か過ぎるだろうか)
剣では槍を掻い潜って馬を切り伏せても密集隊形では同様に押し潰される危険が
高いと言う事だろうか
502某研究者:03/02/13 20:23 ID:AeJtR7WY
まあ密集隊形でも1列だけ前に出せば
後退するスペースが有るから倒した馬に潰される可能性は
無いだろうか
503イプスィロン:03/02/13 20:42 ID:7BdFqH1n
密集隊列ってファランクスっていうことか。なら、なんとなく
わかったと思う。(間違ってたらスマソ魚雷発射ok*ただし4発までね。
ちょっと右甲板がさびかけててね・・。)
504少佐:03/02/13 20:57 ID:FLhg5ik/
まあこの問題に関してはすでに私が論じ尽くしているといていいだろう。
私は現代戦のみならず、古代中世の戦いにおいても造詣が深いのだ。
中世の陸戦においては馬の数と質が勝敗を決めたのだ。
いい馬といい草が生える草原を持つ国が強かった訳だな。
それはモンゴル帝国だ。
50543:03/02/13 21:00 ID:WkTEWq+U
■■出会い系サイト運営システムレンタル■■

儲かる出会い系ビジネス

初心者でも簡単運営

写メール、画像対応

http://kgy999.net/open/






506少佐:03/02/13 21:04 ID:FLhg5ik/
海戦に関してはちと判断が難しい。
造船技術がまだ進んでなかった古代から中世まででは局地的な海戦しか
ないからである。
だが私は語る。
古代から中世までにかけて最高の航海技術を持っていたのはバイキング
なのである。
バイキングはボートでアメリカ大陸まで貿易に出かけているからだ。
それから古代から中世にかけて一番水泳が上手かったのが日本人であった
ことも付け加えたい。
日本人は鎧と刀を付けて泳いだと記録にある。

507某研究者:03/02/13 21:36 ID:+eloBD2P
>中世の陸戦においては馬の数と質が勝敗を決めたのだ。

まあ騎兵を阻止出来るパイク兵や矛槍兵の出現前は
弓兵は雨天では無力であるからそうかも知れぬが


>日本人は鎧と刀を付けて泳いだと記録にある。

まあ西欧の騎士も一応水泳の訓練は受けていたと言うが
鎧を着けては泳げないのだろうか
508某研究者:03/02/13 21:49 ID:+eloBD2P
>踏み込んだとしても日本刀の抜刀速度は早く

まあ西欧の小剣も抜き易いだろうが
中国の歩兵の副武装と言うのは
通常の長剣なのか片刃の環刀なのだろうか
509名無し三等兵:03/02/13 21:54 ID:???
古式泳法ね。扇子持って甲冑着込んだおじいちゃんが立ち泳ぎするの。
510名無し三等兵:03/02/13 21:55 ID:???
プロ野球巨人軍の桑田投手が学んだ
古武術研究家によると、
間合いの距離であらかじめ刀を抜いて似る人間に対しても、
現代社会の人間がしない動きをすれば、
鞘に刀を入れている人間が抜刀して勝つことが可能だそうです。
511名無し三等兵:03/02/13 21:56 ID:vqas2wiL
>古代から中世までにかけて最高の航海技術を持っていたのは
インド洋のムスリムと思われ
512名無し三等兵:03/02/13 21:58 ID:???
>>510
コマネチとか?
513AIRSHIP:03/02/13 22:57 ID:uOxNwOyW
西洋甲冑と盾の使い方、ツバイハンダーと片手剣の戦い。 槍と剣、盾の使い方。
ハルバードと剣の闘いなどが実際に見たかったら、5月3〜4日のブラックホール
(ミリタリ〜イベント)に行ったら見られます。(浅草都立産業貿易センター)
アヴァロンさんが、実際に甲冑を着て格闘戦のデモンストレーションを見せてくれ
ますから。 これを見るだけでも実際の動きが、良くわかると思います。
また、足捌きや剣の使い方等のパンフレット(ただし英文)も販売しています。
プレートアーマーのオーダーや各種剣(レプリカ)の販売も有ります。
514通りすがり:03/02/13 22:59 ID:???
>>511
それは明の鄭和も含んでいるのでしょうか?
515某研究者:03/02/13 23:22 ID:+eloBD2P
>西洋甲冑と盾の使い方、ツバイハンダーと片手剣の戦い。 槍と剣、盾の使い方。
>ハルバードと剣の闘いなどが実際に見たかったら

まあ実際の重量比を保って武器を軽量化した場合
強いのはバスタードソードである訳だろうか
(最も軽量なタウンソードやショートソード・短剣の類は
 使えない訳だろうか)
バスタードソードやハルバードは競技用の片手剣を500gとしたら
同じ比率で軽量化したら750g位だろうが
ツヴァイハンダーは軽く共1.3kg程度だろうから
これを受けた場合防具が有っても厳しい訳だろうか
516某研究者:03/02/13 23:24 ID:+eloBD2P
>バスタードソードやハルバードは競技用の片手剣を500gとしたら

この比率では日本刀やショートソード・タウンソードは250g程度であるから
かなり軽く成る訳だろうか
517某研究者:03/02/13 23:27 ID:+eloBD2P
>、5月3〜4日のブラックホール
>(ミリタリ〜イベント)に行ったら見られます。(

これの武器の重量制限と言うのは
矢張り1kg程度である訳なのだろうか
518少佐:03/02/14 00:26 ID:buSmoQMV
某研究者よ。
君は古代や中世においては馬が輸送手段であるということが解っているか?
また馬の皮から弓の弦が作られたということを理解しているか?
519某研究者:03/02/14 17:58 ID:riu/zUTs
まあ剣は斬る・或いは突く事で敵に引っかかる危険が有り
其の間に敵にやられる危険も有る訳だろうから
棒や中国の鞭(これは剣の刃を無くした様な物だが)の様な物を用いれば
敵を殴っても引っ掛かる可能性は無い訳だから
安全に敵を倒せる訳だろうか
(或いは日本刀の様に引いて斬る刀でも叩き付けたり突いたりするのでは無く
 引いて斬る分には敵に引っ掛かり難いだろうが
 棒よりはそれで動きを拘束される危険も有り峰打ち主体で隙有らば引いて斬ると言う事を
 しないと駄目だsろうか)
520某研究者:03/02/14 21:44 ID:dcB+WJn9
>棒よりはそれで動きを拘束される危険も有り峰打ち主体で隙有らば引いて斬ると言う事を
>しないと駄目だsろうか)

まあ西欧の剣は峰打ちは無理だろうから
どうしても剣の刃が敵に引っ掛かる懸念は
敵が完全鎧でも着ていない限り有る訳だろうか
槍や長刀も殴ってから隙あらば刺すか
引いて斬る・或いは叩き斬ると言う事で良い訳だろうか
521某研究者:03/02/14 21:53 ID:dcB+WJn9
>槍や長刀も殴ってから隙あらば刺すか
>引いて斬る・或いは叩き斬ると言う事で良い訳だろうか

まあ十文字槍の引いて斬る攻撃も
敵に引っかかる可能性は有るだろうし
首を確実に捉えねば危険だろうか
(或いは足を捕らえて敵を転倒させれば
 1対1の場合は槍を引く時間が有るなら問題は無いだろうが
 刀で足を狙った場合は間合いの問題から引いている間にやられる危険も
 有るだろうか
 (まあ槍対槍であれば槍を用いても足を斬る・突く等した場合同様に
  やられる危険も有るだろうか)
522某研究者:03/02/15 00:59 ID:hsY9LJ8V
まあ矢張り殴って敵を倒すか
殴ってから首等を引いて斬って
止めを刺すと言うのが1対多・多対多での戦いでは
有効である訳だろうか
(故に鎧の無い敵に対しては中国の鞭(これは両手用・或いは両片手用の物が良いだろうが
 両手用の鞭と言うのは有っただろうか)か木刀の様な物
 日本刀が一番有効である訳だろうか
 鎧の有る敵に対しては両手用・或いは両片手用の鞭・或いは木刀が
 有効だろうか)
523某研究者:03/02/15 01:01 ID:hsY9LJ8V
まあ矢張り敵に刺さって自分の動きを拘束しない
打撃での突きと言う選択肢も使えるなら
木刀か鞭が一番良いだろうか
524某研究者:03/02/15 01:04 ID:hsY9LJ8V
まあしかし木刀を鎧の上から何度も打った場合
砕ける危険と言うのは無いのだろうか
525某研究者:03/02/15 01:07 ID:hsY9LJ8V
まあ某スレッドで

>ようやくイタリア14世紀プレートアーマー完装。重さは25kg。
>私は体重54kgで163cm。重さは程良く分散され思ったより動きやすい。

まあ1.2mmの全身鎧が20kgと言う事は
2mmにすれば40kgだろうし
これでは少なく共手足の防弾効果は皆無であると見て良いだろうか
(胴体の厚みを5mm等とすれば
 手足は1mm程度に成らざる負えぬ訳だろうか)
手足の丸みで矢が滑ると言う事は有るだろうが
銃弾は滑らすのは困難である可能性も有るだろうか


882 名前:某研究者 :03/02/14 23:56
>2mmにすれば40kgだろうし

トーナメントアーマーの重量は80kgと言うのも有る様だが
これの厚みと言うのは平均すれば4mm程度なのだろうか
(まあこれはトーナメント用であるなら恐らく正面のみを厚くしてあり
 正面の特に胴体や頭部はかなりの厚みが有る訳だろうか) 


プレートアーマーの厚みと言うのはこの程度であるなら
木刀や鞭を両手で振るえば確実にダメージは
トーナメントアーマー相手でさえ手足には与えられる訳だろうか
526某研究者:03/02/15 01:11 ID:hsY9LJ8V
上失礼鎧の厚みが抜けていたが

>厚みは1,2mm。ステンレス。実際の物は焼きを入れた鉄で1mm。
527某研究者:03/02/15 10:07 ID:hsY9LJ8V
馬に乗る際は剣で馬を守る訳にも行かず
槍で馬を防護するしか無い訳だろうが
刃の有る槍は敵に引っ掛かる恐れも有るだろうし
突槍や棒を用いるべきであると言う事だろうか
528某研究者:03/02/15 19:59 ID:tVdU8O3L
まあそうすると馬上での使用及び騎兵を阻止する際は突槍
それ以外では棒(これは踏み込まれても対応可能な短い物だが)
か木刀・中国の鞭を使っての物だろうか
(まあ恐らく棒術の攻撃パターンの方が木刀や鞭より多い訳だろうが
 実戦ではどちらが強いだろうか)
529名無し三等兵:03/02/15 20:18 ID:???
>>528
棒術は棒の両端を切先として使用することが要諦であり、
騎馬の突進力とは両立し難い技術と考えます。
敵に留められなければ突き抜け、槍により再度突撃した方が有利であり、
それが適わなければ刀を使った方が合理的かと。
530某研究者:03/02/15 20:53 ID:tVdU8O3L
まあ槍を馬に刺しては敵の騎兵の波状攻撃を
一列の槍兵では阻止出来ず
騎兵が騎兵・歩兵を倒す場合も同様である訳だろうが

馬上で棒術は扱い難く木刀の方が良いかも知れぬが
短い棒なら馬上での近接戦でも扱えるだろうか
まあしかし馬上或いは地上での木刀と棒術
何れが強いだろうか
531430:03/02/15 21:00 ID:a37s/0JB
■■出会い系サイト運営システムレンタル■■

儲かる出会い系ビジネス

初心者でも簡単運営

写メール、画像対応

http://www.geocities.jp/kgy919/





532某研究者:03/02/15 23:47 ID:tVdU8O3L
非力な人間も木刀や棒・突槍を用いて何度も殴る・或いは突けば
敵の動きが鈍り更に連打すれば倒せる可能性は有る訳だろうか
533某研究者:03/02/15 23:54 ID:tVdU8O3L
まあしかし西欧にクオータースタッフと言う棒は有る様だが
突槍や木刀・或いは中国の鞭の様な物は有るのだろうか
534ななしさん:03/02/16 00:26 ID:TdCgwQp4
>533
西欧には牛のちんちんで作った鞭があるよ。
中国の鉄製とかの奴みたいにひと殺せる奴ですよ。
535某研究者:03/02/16 07:15 ID:rVQj9B9M
http://plaza4.mbn.or.jp/~tactic/match.html
まあ火縄銃も火蓋の中に火薬が入れられれば
火縄に火を付ければ即座に発射出来るのだろうか
536某研究者:03/02/16 07:26 ID:rVQj9B9M
http://www.hondaplus.co.jp/sinsiro/hinawaju.htm
上に良く見るとストック装備の火縄銃が有るが
これは戦国時代の物なのだろうか
537某研究者:03/02/16 08:38 ID:rVQj9B9M
http://www.sanada.or.jp/contents/column/hinawa/teppo_tai.html
欧州の鉄砲保有数が関が原で使われた5万丁程度であるなら
日本の鉄砲保有数15万丁と言うのは欧州の3倍に及ぶ訳だろうか
538某研究者:03/02/16 08:49 ID:rVQj9B9M
http://www.sanada.or.jp/contents/column/hinawa/teppo_tai.html
火縄銃で500m先の敵を狙撃したと言う記述も有るが
これは長銃である可能性も有るだろうし
500m先からでは恐らく鎧は貫けない可能性も
有るだろうか
(まあしかし足軽の1.4mmの鎧等は300m付近で貫通だろうし
 3mmや5mmの鎧は貫けず共装甲の薄い部分や鎧の無い部分に当たればアウトだろうが)

まあ井伊直政を狙撃したと言うのも長銃かも知れないが
敵の指揮官の狙撃と言うのは他にも行われたのだろうか
(まあ李舜臣やネルソンも狙撃され命を落とした様だが
 他の将軍に関してはどうなのだろうか)
539某研究者:03/02/16 08:56 ID:rVQj9B9M
最大射程500mでは柵の無い状態では騎兵に対しては
一回の射撃が限度だろうから槍兵と入れ替われる距離で
ぎりぎり迄引き付けて放った方が良いだろうか
突入して来る敵歩兵に対しては4回程度は射撃可能な訳だろうか
540燃料投下:03/02/17 07:37 ID:lFhdDh6a
日本の火縄銃は頬にあてる構造上の特質から、命中時の威力は小さいため、
競技射撃の命中精度を、単純に軍用有効射程とみるわけには行かないと思われ。
李舜臣は流れ弾という説も有力。
ストック付は、「洋風〜」という銘から、江戸時代に洋式銃をもして作ったものと思われる。
「雑兵物語」などの記述によると、火縄銃で騎馬武者を撃つときは、馬を狙うのが常識だったようだ。
541某研究者:03/02/17 07:45 ID:0R0zqM1x
>ストック付は、「洋風〜」という銘から、江戸時代に洋式銃をもして作ったものと思われる。

まあ只ストックのみではなく着火方式も
真似しなかったのだろうか
542某研究者:03/02/17 07:47 ID:0R0zqM1x
>日本の火縄銃は頬にあてる構造上の特質から、命中時の威力は小さいため、
>競技射撃の命中精度を、単純に軍用有効射程とみるわけには行かないと思われ。

まあ鎧は貫けず共鎧の無い場所に当たれば
戦闘は不能だろうしその確率は特に騎兵では
かなり高い訳だろうか
543某研究者:03/02/17 08:16 ID:0R0zqM1x
http://www.hondaplus.co.jp/sinsiro/hinawaju.htm
10匁弾程度でも土を詰めない竹束なら貫けたのか
土を詰めた竹束であっても30匁弾以上では貫通する可能性も
有る訳だろうか
大阪夏の陣で使われた鉄の盾も通常の火縄銃の近距離からの銃撃
或いは10匁弾での射撃では貫かれていたのだろうか
(まあ現存している大阪夏の陣で使われたと言う鉄の盾には弾が貫通した跡が有る様だが
 鉄の盾等所詮士気を維持する為の物に過ぎなかったのだろうか)

武田信玄の考案したと言う竹束の盾と言うのは
竹束の中には土が詰まっていないと言うなら
近距離では通常の銃でも貫かれていた訳だろうか
(まあ車竹束には恐らく土が詰められていた訳だろうが
 これを用いても大筒の攻撃には限度は有る訳だろうか)
544某研究者:03/02/17 08:20 ID:0R0zqM1x
ストック部分のコスト増を考えれば
これが無く共精度や威力が余り変わらないのであれば
ストックを装備しないと言うのは銃剣でも付けない限り
ストックに掛かる金で鉄砲や兵員の数を増やすと言う意味は有った訳だろうか
545名無し三等兵:03/02/17 09:52 ID:lFhdDh6a
>>541 着火方式
江戸期に燧石式・雷管式も試作されてはいたが、作動時の振動で命中精度に劣り、
また火縄銃の各流派により抵抗されたこともあり、実用化はされなかったようで。
ストックに関しては、単純に発想が生まれなかったと見るほうが自然ではないか?
546名無し三等兵:03/02/17 12:08 ID:???
>>536
「洋風〜」は思いっきり管打ち(パーカショッン)式じゃないですか。
日本で管打ちが登場するのは1840年代以降、これはおそらく1850-60年代に
海外のパーカッション長銃をモデルにして製造したものと思われ。
547某研究者:03/02/17 22:17 ID:afl22Lkm
日本の騎兵の馬は大きくは無いと言うが
当時の絵を見る限り確かに大きくは無いが異様に小さい共
言えないだろうか(無論格好が悪いので大きくしている可能性は
有るかも知れぬが)

まあ元寇を描いた絵には確か馬上から弓を射撃している絵しか無く
徒歩では放っていないと思うが
当時の歩兵は弓は使えたのだろうか
(まあしかし戦国時代の絵では歩兵も弓を放っていただろうか)
548某研究者:03/02/17 22:33 ID:afl22Lkm
http://www.lib.kyushu-u.ac.jp/gallery/moukoshurai/e04.htm
まあモンゴル騎兵の馬と日本の馬のサイズは変わらないが
これは矢張り誤りだろうが恐らくモンゴルの馬が日本の馬と同サイズに
小さく描かれていると見て良いのだろうか
549某研究者:03/02/17 22:47 ID:afl22Lkm
http://www.lib.kyushu-u.ac.jp/gallery/moukoshurai/e09.htm
まあこれを見ると大袖の付いていない大鎧を着ている騎兵も存在する訳だが
これは位の問題なのだろうか
(徒歩の兵も草履の様な物を履いているが
 何も履いていない場合も多かった訳だろうか)

先頭の騎兵は靴では無く草履の様な物を履いているが
彼の位はどうだったのだろうか
550某研究者:03/02/17 23:01 ID:afl22Lkm
http://www.lib.kyushu-u.ac.jp/gallery/moukoshurai/e11.htm
まあ矢張り城よりも狭い船上では
上に描かれている様な大型の弓は扱い難いと言う事だろうか(苦笑

上の絵の長巻の形状はいびつな形状をしているが
実際に使われたのは長刀の様な刃はしてないと言う事なのだろうか

http://www.lib.kyushu-u.ac.jp/gallery/moukoshurai/e12.htm
まあしかしモンゴル側が日章旗の様な旗を用いているのは
どう言う事なのだろうか
551名無し三等兵:03/02/17 23:46 ID:???
>>547
やはり、日本馬は小さいです。骨や馬具から明らかです。
確かに騎乗する人間のほうも今よりは小さいですが、それでもバランスとしては格好が悪い。
描かれているものは、格好よくデフォルメしたとみて間違いない模様。

>>548
モンゴル馬は小さいですよ。

>>549
>草履
先頭の騎乗兵は旗をもった先触れのようなものでしょう。
大鎧も着ていません。季長の郎党では?
>大袖のない大鎧
前から4人目の兜をかぶった騎乗兵のことだろうか?
彼の鎧なら、大鎧ではなく単なる胴丸だと思うが。
552某研究者:03/02/18 00:21 ID:vWbMLdu/
>>大袖のない大鎧
>前から4人目の兜をかぶった騎乗兵のことだろうか?
>彼の鎧なら、大鎧ではなく単なる胴丸だと思うが。

まあ確かに胴体正面の形状と草摺の形状を考えれば
その様だが大鎧を着れない騎馬武者と言うのも
居たと言う事なのだろうか
553名無し三等兵:03/02/18 00:27 ID:mU/xmjHn
葉武者。またはただの乗馬歩兵。
554某研究者:03/02/18 00:30 ID:vWbMLdu/
>葉武者。またはただの乗馬歩兵。

まあしかし乗馬歩兵と言うのは鎌倉時代・或いは日本に
存在したのだろうか
555某研究者:03/02/18 02:21 ID:vWbMLdu/
円盾兵・槍を持つギリシャやマケドニアファランクスを撃退した
近接戦兵器は剣と盾しか持たないローマレギオン
或いは両手剣を持つランツクネヒトの存在も有る訳だろうし
歩兵対騎兵・騎兵対騎兵は兎も角歩兵対歩兵で
槍と剣の戦いに決着は付いたとは言い難い部分も有る訳だろうか
(騎兵対歩兵では剣では短過ぎて騎兵の馬が防護出来ぬ訳だろうし
 槍を使わざる負えず騎兵対騎兵でも同様である訳だろうか)

まあしかし歩兵対歩兵の戦いでも槍の間合いの内側に
踏み込めぬ様な
訓練度や士気の低い雑兵には槍を持たせた方が良いとの
意見も有る訳だろうが

まあ雑兵である足軽や徒士は兎も角
円盾兵・レギオンやランツクネヒトは訓練を受けた兵であるだろうし
剣を有効に使えるだろうが彼等とて数が無ければ
槍兵に包囲されやられる訳だろうか

まあ故に少数の円盾兵を敵の槍兵から守り
戦線を支える大量の雑兵である槍兵と
戦線を崩す小数の円盾兵の様な物を
上手く使えれば良かっただろうか

まあ兵の数や戦術が拮抗していても
少数の精鋭部隊が存在すればそれを用いて
突破口等を開いて騎兵等を突入させ
勝利する事が出来る場合も有るだろうか
556えすえすびーえぬ:03/02/18 02:47 ID:???
>>555
ローマがマケドニアふぁランクすを撃破したのは騎兵などが側面から攻撃したからでは無かったですか?

マケドニアファランクスは槍長くなったりでギリシアのそれより機動力無かったみたいですが、
ギリシャファランクスとぶつかったときローマはどう戦ったんでしょう。

いきなり第三列兵投入して前二列で側面攻撃?
557某研究者:03/02/18 07:15 ID:2ATpPbsJ
雑兵に剣を持たせても槍の間合いの内側に
飛び込む技量は無い訳だろうし
矢張り槍を持たせなければやられるだけだろうか
無論剣で槍兵を崩すのではなく
パイク兵の様な精鋭部隊に槍を持たせて
槍で槍を崩すと言う方向性も有る訳だろうが
精鋭槍兵も剣を持った精鋭歩兵に踏み込まれる危険は
有る訳だろうか
558某研究者:03/02/18 07:22 ID:2ATpPbsJ
まあ雑兵である短槍兵では矛槍兵の間合いの内側にも入れず
一方的にやられる危険も有る訳だろうし
短槍兵は騎兵も阻止出来ぬなら雨天以外では弓兵以下の
雑魚である訳なのだろうか(苦笑
559某研究者:03/02/18 07:30 ID:2ATpPbsJ
>ローマがマケドニアふぁランクすを撃破したのは騎兵などが側面から攻撃したからでは無かったですか?

まあローマ騎兵もマケドニアの長槍騎兵に勝てたのかと言う事だろうし
仮に勝てても地形等の問題で側面を突けねば
ローマレギオンや騎兵のピルムが正面からでは多数の槍の壁で弾かれた
可能性も有る訳だろうが近距離から足等に打ち込む事は
可能だっただろうか
(其れで隊列を崩して剣を持った歩兵や騎兵を乱入させれば側面以外からでも
 壊滅可能かも知れぬがファランクス側も長槍以外に剣と盾を持つ訳であり
 槍の間合いの内側に入ろうと其れを読まれれば剣で反撃される可能性も有るだろうが
 矢張り彼等の剣の練度はローマレギオンの比では無いだろうか(苦笑)


>ギリシャファランクスとぶつかったときローマはどう戦ったんでしょう。

正面からでもピルムで隊列を崩した後剣での戦いに持ち込めば
恐らく剣の錬度の差で勝てるだろうか
560某研究者:03/02/18 07:53 ID:2ATpPbsJ
狭い場所・或いは敵に包囲されぬ様な状況で
100人の精鋭兵が一人辺り10人の敵を倒せば
1000人の敵を倒せる訳だろうし
一人当たり10人程度倒したら撤退して疲労回復後に
再度攻撃する事を繰り返せば
補給も余り要らずに敵を倒せる訳だろうし
これを10回やれば100人で10000人を倒せるだろうが
矢張り雨天で無いと弓兵にやられる危険は有るだろうか
561某研究者:03/02/18 07:57 ID:2ATpPbsJ
>矢張り雨天で無いと弓兵にやられる危険は有るだろうか

まあ雨天であれ投げ槍の攻撃をどうするのかと言う事だが

まあ飛び道具に対処出来且つ
上手く休めば一日で一人100人・下手をすると其れ以上倒せるだろうし
一日で100人の精鋭兵に10000人以上がやられると言う可能性も
有る訳だろうか(苦笑
562某研究者:03/02/18 08:00 ID:2ATpPbsJ
まあ後は100人が密集している所に
カタパルトやバリスタでも打ち込まれれば
簡単に壊滅する可能性も有るだろうしこれを防ぐ手も
考えないと無理だろうか
563某研究者:03/02/20 06:01 ID:qE5l/wWw
まあ500m飛ぶクロスボウは
装填に1分程度掛かるかも知れぬし
同様に500m飛ぶ銃は
一分に3発は撃てる訳だからこれは有利である訳だろうか
(他にもクロスボウより銃のコストの方が
 低いと言う事は有る訳だろうが)

日本側の騎兵銃なら
明の騎兵のクロスボウをアウトレンジ可能であり
明歩兵に重クロスボウが少なければ
銃・騎兵銃でアウトレンジして壊滅的打撃を与える事も可能だっただろうか
564某研究者:03/02/21 07:45 ID:E6diMxmf
まあラストオブサムライでは無いが
日本刀が銃に勝てるかは兎も角銃剣に勝てるのかと言う事だが
4−5kgと重いマスケット銃(日本では1870年代は未だこれが主流だろうか)
に銃剣を付けても軽い両手で振るわれる日本刀(0.7−0.8kg)に
対抗可能なのかと言う事だろうし
達人であれば銃剣の間合いの内側に入り込む事は
容易かも知れぬが
素人では銃剣の間合いの内側に踏み込む勇気も無いだろうし
銃剣にやられる可能性は高いだろうか
片手で振るわれる重いサーベル(1.7kg)であれば
0.7−0.8kgの日本刀の方がスピードも速く相手に
攻撃を当てられる可能性も高く防御も素早く行える訳だろうか
ハンガーの様な軽い剣も有るがこれは短く乱戦時以外では日本刀には
対抗し難いだろうか

まあ雨が降っている時はマスケット銃は撃てないだろうし
其の間に斬り込むと言う方向も有るだろうが
当時の大砲は雨天でも使用出来る可能性は有るだろうか

http://member.nifty.ne.jp/bintaro/touken/cutlass.htm
カトラスも日本刀よりかなり重いから
矢張り乱戦以外では勝ち目は無いと見て良いだろうか
565某研究者:03/02/21 07:45 ID:E6diMxmf
http://member.nifty.ne.jp/bintaro/touken/katzbalger.htm
西欧の両手剣で一番軽いのはこの類だろうか

http://member.nifty.ne.jp/bintaro/touken/dress.htm
http://member.nifty.ne.jp/bintaro/touken/smallsword.htm
http://member.nifty.ne.jp/bintaro/touken/pallasch.htm
http://member.nifty.ne.jp/bintaro/touken/backsword.htm
片手剣では上が軽いだろうが
矢張り日本刀とを両手で扱われたら其のスピードには
対応困難だろうか

http://member.nifty.ne.jp/bintaro/touken/fleuret.htm
エペでさえ0.3kgは有るなら両手持ちの日本刀には対応され
折られる危険も有るのではないのか 

まあ只エペ2本と日本刀何れが強いかは不明だが
どうなのだろうか

まあ引いて斬れる武器と言うのは鍔競り合い等の接近戦や
相手が短剣等の場合は使える訳だろうか
566某研究者:03/02/21 07:51 ID:E6diMxmf
http://member.nifty.ne.jp/bintaro/touken/katzbalger.htm
>西欧の両手剣で一番軽いのはこの類だろうか

http://www.fabri-armorum.cz/medieval_swords.htm
まあ1.5kg以下の両手剣と言うのは
西欧には無い訳だろうか
(カッツバルゲルの1.4kgの物と言うのは
 片手用の可能性も有るだろうか)
567某研究者:03/02/21 23:58 ID:1t1zh1dg
>まあ500m飛ぶクロスボウは
>装填に1分程度掛かるかも知れぬし

http://plaza4.mbn.or.jp/~tactic/longbow.html
まあしかし350m飛ぶクロスボウの装填速度も
1分程度は掛かるのだろうか
568某研究者:03/02/22 00:10 ID:XtWZsSQ4
まあしかし同時代に作られた大鎧が角張っており
胴丸が丸いのはどう言う事なのかと言う事だが
569某研究者:03/02/22 01:57 ID:XtWZsSQ4
まあ恐らくは大鎧が角張っているのは矢の貫撤を防ぐ為に札の隙間を無くす為の工夫だろうが
室町時代後期には札では無く槍や鉄砲に対抗して板を使う様に成ったので
防弾効果を保った侭丸く作れていた訳だろうか
570某研究者:03/02/22 02:00 ID:XtWZsSQ4
>まあしかし350m飛ぶクロスボウの装填速度も
>1分程度は掛かるのだろうか

まあクロスボウは兎も角バリスタの発射速度は
複数人で装填するから余り遅くは成らない可能性も有るが
矢張り数は揃えられない訳だろうか
571某研究者:03/02/22 02:25 ID:XtWZsSQ4
>日本側の騎兵銃なら
>明の騎兵のクロスボウをアウトレンジ可能であり

まあしかし馬上での騎兵銃の装填は可能なのかと言う事だが
クロスボウは巻き上げ式で無いと困難である可能性も有るだろうか
明の騎兵が射撃戦では敵わぬと見て突撃を掛けて来た場合は
騎馬鉄砲隊は銃を撃った後に短銃も放てるだろうか
(まあ只弓を持つ騎兵が踏み込んで矢を連射して来る危険も有るだろうが
 其の前に銃でダメージを受け得るだろうし
 日本側も騎馬弓兵で彼等の牽制も可能かも知れぬが
 日本・明側の騎馬弓兵の数はどの程度なのだろうか)
572名無し三等兵:03/02/22 02:39 ID:???
           ゙ミ;;;;;,_
            ミ;;;;;;;;、;:..,,.,,,,,
            i;i;i;i; '',',;^′..ヽ
            ゙ゞy、、;:..、)  }
             .¨.、,_,,、_,,r_,ノ′
           /;:;":;.:;";i; '',',;;;_~;;;′.ヽ
          ゙{y、、;:...:,:.:.、;:..:,:.:. ._  、}
          ".¨ー=v ''‐ .:v、,,、_,r_,ノ′
         /;i;i; '',',;;;_~⌒¨;;;;;;;;ヾ.ミ゙´゙^′..ヽ
         ゙{y、、;:...:,:.:.、;:..:,:.:. ._  .、)  、}
         ".¨ー=v ''‐ .:v、冫_._ .、,_,,、_,,r_,ノ′
        /i;i; '',',;;;_~υ⌒¨;;;;;;;;ヾ.ミ゙´゙^′.ソ.ヽ
        ゙{y、、;:..ゞ.:,:.:.、;:.ミ.:,:.:. ._υ゚o,,'.、)  、}
        ヾ,,..;::;;;::,;,::;):;:;:; .:v、冫_._ .、,_,,、_,,r_,ノ′
       /:::::::::: ,,,,;;:::::::::::::::             ヽ
      /:::::::::  "__  ::::    ,,::::::::::::,,,,  |  はぁ・・・・・・。
     /:::::::::  - '"-ゞ,●> :::::.      __ ''  |
    /::::::::           ::::::    <'●,  |
    |::::::           ( ○ ,:○)       |
    |:::::::                           |
    |::::::         __,-'ニニニヽ       |
     \::::::         ヾニ二ン"       ,/
      \:::::                   /ヽ ミ 3
       \                  /(  |
       /:::::::                l    ヽ
       /;:::::::                 \   \
      /::::::   ,,_/\                |
      |:::::::  ~      |          ゝ___ノ
      ヽ、____ノヽ' うんこスレ>>1-571
573名無し三等兵:03/02/23 11:40 ID:???
このスレ、キモイそれはそうと、武防板の、現代で有効な武器ってスレで某研究者叩きしてるよ、皆注目だ
574名無し三等兵:03/02/23 11:56 ID:???
戦場での死因のトップは不衛生な環境による病死(破傷風や赤痢など)
戦闘による臓器損傷や失血による死亡は総死者数の1割程度

って話はキシュシでしょうか?
575死ね:03/02/23 13:51 ID:???
死ね死ね死ね死ね死ね死ね死ね死ね死ね死ね死ね死ね死ね死ね死ね死ね死ね死ね死ね死ね死ね死ね死ね死ね死ね死ね死ね死ね死ぬ死ね死ね死ね死ね死ね死ね死ね死ね死ね死ね死ね死ね死ね死ね死ね死ね死ね死ね死ね死ね死ね死ね死ね死ね死ね死ね死ね死ね死ね死ね死ね死ぬ死ね死ね死ね
576名無し三等兵:03/02/23 13:52 ID:???
死ね死ね死ね死ね死ね死ね死ね死ね死ね死ね死ね死ね死ね死ね死ね死ね死ね死ね死ね死ね死ね死ね死ね死ね死ね死ね死ね死ね死ぬ死ね死ね死ね死ね死ね死ね死ね死ね死ね死ね死ね死ね死ね死ね死ね死ね死ね死ね死ね死ね死ね死ね死ね死ね死ね死ね死ね死ね死ね死ね死ね死ぬ死ね死ね死ね
577名無し三等兵:03/02/23 13:52 ID:???
死ね死ね死ね死ね死ね死ね死ね死ね死ね死ね死ね死ね死ね死ね死ね死ね死ね死ね死ね死ね死ね死ね死ね死ね死ね死ね死ね死ね死ぬ死ね死ね死ね死ね死ね死ね死ね死ね死ね死ね死ね死ね死ね死ね死ね死ね死ね死ね死ね死ね死ね死ね死ね死ね死ね死ね死ね死ね死ね死ね死ね死ぬ死ね死ね死ね
578名無し三等兵:03/02/23 13:52 ID:???
死ね死ね死ね死ね死ね死ね死ね死ね死ね死ね死ね死ね死ね死ね死ね死ね死ね死ね死ね死ね死ね死ね死ね死ね死ね死ね死ね死ね死ぬ死ね死ね死ね死ね死ね死ね死ね死ね死ね死ね死ね死ね死ね死ね死ね死ね死ね死ね死ね死ね死ね死ね死ね死ね死ね死ね死ね死ね死ね死ね死ね死ぬ死ね死ね死ね
579名無し三等兵:03/02/23 13:53 ID:???
死ね死ね死ね死ね死ね死ね死ね死ね死ね死ね死ね死ね死ね死ね死ね死ね死ね死ね死ね死ね死ね死ね死ね死ね死ね死ね死ね死ね死ぬ死ね死ね死ね死ね死ね死ね死ね死ね死ね死ね死ね死ね死ね死ね死ね死ね死ね死ね死ね死ね死ね死ね死ね死ね死ね死ね死ね死ね死ね死ね死ね死ぬ死ね死ね死ね
580名無し三等兵:03/02/23 13:53 ID:???
死ね死ね死ね死ね死ね死ね死ね死ね死ね死ね死ね死ね死ね死ね死ね死ね死ね死ね死ね死ね死ね死ね死ね死ね死ね死ね死ね死ね死ぬ死ね死ね死ね死ね死ね死ね死ね死ね死ね死ね死ね死ね死ね死ね死ね死ね死ね死ね死ね死ね死ね死ね死ね死ね死ね死ね死ね死ね死ね死ね死ね死ぬ死ね死ね死ね
581名無し三等兵:03/02/23 15:46 ID:???
一人でなにやってんの?
582某研究者:03/02/23 16:19 ID:R++5d3BI
>戦場での死因のトップは不衛生な環境による病死(破傷風や赤痢など)

まあ故に敵の陣地や水源等に死体を投げ込む等の行為が
有効だった訳だろうか
583名無し三等兵:03/02/23 17:39 ID:???
>某研究者
まあしかしずいぶんと君は「まあ」「〜だが」を多様する様だが。
584名無し三等兵:03/02/23 19:46 ID:???
某研、実戦では食料不足が一番恐いのではないのか?
585名無し三等兵:03/02/24 11:41 ID:???
>>582
まあ、いずれにしても、兵氏の死傷要因が、直接の戦闘によるものばかりではなく、
間接的にも大きかった…
かつ、士気を維持するための兵站の確立が困難であった…

というようなことが、ナショナリズムを発見するまで、大規模な会戦を困難にしたわけでしょうが…。
586某研究者:03/02/25 07:13 ID:mdjbPdJ3
まあ兵のみではなく武田信玄やアレクサンダー等も
目的達成迄後一歩の所で病死している訳だろうが


>某研、実戦では食料不足が一番恐いのではないのか?

まあ中世では現地調達も多いのであれば
後方からの補給の無い場合は
一箇所に留まる訳には行かずそこら中略奪して回らねば
成らぬと言う事も有った訳だろうか(苦笑
(敵地に篭城している戦力が食料を集める為に
 外部の村や都市・軍を襲撃すると言う事も有った訳だろうか(苦笑
 まあ敵だけではなく味方の村や都市・或いは軍を襲撃すると言う事も
 有り得た訳だろうが)
587某研究者:03/02/25 07:19 ID:mdjbPdJ3
まあ後方の兵を前線に迅速に移動させる為に
武器や鎧・食料は前線の城に備蓄しておき
兵は前線迄丸腰で移動すると言う方法も有る様だが
(まあ武器や物資は車や船で別途輸送すると言う方向も
 有る様だが)
丸腰の兵が万一行軍中に敵に襲撃される・或いは
反乱でも起こされれば壊滅的な事に成ると言う事は
無いのだろうか
588某研究者:03/02/25 07:27 ID:mdjbPdJ3
中国では政治に関わらない純粋な武人と言うのは
戦争が終わった後も粛清されないと聞いたが
純粋な武人の振りをして政敵が粛清されるのを待ち
それから事を起こした者と言うのも居た訳だろうか
(其の様な事をすれば以後は武人も粛清されると言う事に
 成り得ただろうか)
589某研究者:03/02/25 14:46 ID:HqETohfM
まあ胴丸も布に鉄を縫い付けた物を
2枚重ねれば似た様な物だったろうし
防御上問題と成る糸を通す穴が余り増えなくて良かったのではないのか(笑
590某研究者:03/02/25 14:54 ID:HqETohfM
まあローマ時代に既に鉄製のプレートアーマー
(まあ只これは日本の桶側胴の様に胴体が2枚の鉄板で出来ている物では無いが)
が有った事を考えると
(ギリシャの其れは中世の其れの様に一体構造だが
 あれは青銅製だろうか)
矢張り鱗鎧や鎖帷子しか無い暗黒時代と言うのは
技術的に停滞していた訳だろうか
(同時期のパルティアにプレートアーマーの様な物が有り
 トルコにも有った様だがこれは隙間が有りどちらかと言えば
 畳具足に近い物だろうか)
591某研究者:03/02/25 15:00 ID:HqETohfM
>まあ胴丸も布に鉄を縫い付けた物を
>2枚重ねれば似た様な物だったろうし

日本にもこのタイプの鎧は入って来た訳だが
廃れたのは何故なのだろうか
生産性を考えれば平安時代や鎌倉時代の徒士は胴丸ではなく
この様な畳具足の様な物を使うべきだっただろうが
(防御力を増したいならこれを2枚重ねれば
 胴丸と同じかそれ以上ではないのか)
592某研究者:03/02/25 15:03 ID:HqETohfM
まあしかし中国等で鉄を縫い付けた綿鎧を2枚重ねて着ると言う事は
有ったのだろうか
(まあ上下の札が交互に組み合わさり隙間を無くす様な工夫迄されたかは
 不明だが)
593某研究者:03/02/26 17:21 ID:s8hKZPjj
http://www.siue.edu/COSTUMES/PLATE3BX.HTML
まあ上はアッシリアの軍装だが
パビスよりも巨大な盾を持っているがこの段階で弓が
大量に使われていたのか・或いはこれは投げ槍に対しての
物なのだろうか

http://www.siue.edu/COSTUMES/PLATE3CX.HTML
これはアルスラーン戦記が参考としたササン朝ペルシャよりは前なのか
まあしかし彼等の格好は中世の欧州に随分似ているが

http://www.siue.edu/COSTUMES/PLATE3DX.HTML
まあギリシャの文化程度がペルシャよりそう高い共
思えないだろうし彼等はサンダルでは無く靴を履いているから
中世ヨーロッパに妙に似ているのではないのか

http://www.siue.edu/COSTUMES/PLATE4AX.HTML
まあユダヤの文化程度が
http://www.siue.edu/COSTUMES/PLATE2AX.HTML
彼等の敵であるアッシリアやエジプトより
特段高い共思えぬしこの周辺は皆同じ様な物だろうか
594某研究者:03/02/26 17:39 ID:s8hKZPjj
http://www.siue.edu/COSTUMES/PLATE7CX.HTML
まあ矢張り同時期のゲルマン民族の格好を考えれば
どうかだろうが(苦笑

http://www.siue.edu/COSTUMES/PLATE7DX.HTML
女がサクスの様な剣を持っている様に見えるが
この時期のゲルマン人種は女は皆剣を持っていたのだろうか
(しかしこの時期のゲルマン人に鎖帷子等有ったのかと言う事だが
 矢張りローマの影響で手に入れた物なのだろうか)


595某研究者:03/02/26 17:53 ID:s8hKZPjj
http://www.siue.edu/COSTUMES/PLATE11AX.HTML
ブーツとサンダルの中間の様な物を履いているのが
彼等の特徴である訳だろうか

http://www.siue.edu/COSTUMES/PLATE12DX.HTML
後ろの人間がプレートの様な物を着ているが
この時期には有ったのだろうか
(まあブーツとサンダルの中間の様な物は
 この時期東ローマ以外にも有ったと言う事なのだろうか)
596某研究者:03/02/26 18:16 ID:s8hKZPjj
http://www.siue.edu/COSTUMES/PLATE14DX.HTML
まあ十字軍の装備と言うのは
剣も兜も鎧も斧も
皆基本的にはバイキング=ノルマン人の考案した物のコピーであるし
文化的には兎も角軍事的にはバイキングの方が
大陸の人間より上だったと言う事なのだろうか
597某研究者:03/02/26 18:19 ID:s8hKZPjj
まあ矢張り彼等に取り略奪者である筈の
バイキングの格好をして
聖戦をすると言うのも何であるし
これは矢張り彼等の本質を現しているのかも知れぬが(苦笑
この時点ではヘルメット・盾・剣・斧のデザインも殆ど
ノルマン人の其れと変わらぬ物である訳だろうが
598某研究者:03/02/26 18:24 ID:s8hKZPjj
まあ矢張りバイキングの長剣の原型と言うのは
何処から来たのかと言う事であり
これは中国或いは中東の剣が原型と言う事は
無いのだろうか
(ゴート人やケルト人・フランク人も長剣をかなり前から
 使っていた筈だがこれは何処から手に入れたのだろうか)
599某研究者:03/02/26 18:27 ID:s8hKZPjj
十字軍のロングソードは兜を割れたと言うが
ロングソードでプレートアーマーの胴部を両断可能なのだろうか
600某研究者:03/02/26 18:41 ID:s8hKZPjj
http://www.siue.edu/COSTUMES/PLATE18BX.HTML
まあこの様に武器を鎖で繋いでおれば
アジャンクールで武器を泥の中に落とす様な事も
無かっただろうが矢張り疲労でやられていただろうか
601某研究者:03/02/27 08:57 ID:Y3gu6NLW
http://www.adachi-hanga.com/sensyu/adachi_kokuritsu/adachi_image_eishosaichoki_yukinakashushokume.htm
まあしかしこう言う絵は余り西欧では見ないが
駕籠を担いでいる人間の地位が低いと言うのも
有る訳だろうか(苦笑

http://www.geocities.co.jp/AnimeComic-Cell/8793/kiyomizu.htm
まあこう言う構図の絵も西欧では余り見ないが
日本にも無く彼独特の物なのだろうか
602某研究者:03/02/27 09:24 ID:Y3gu6NLW
http://www.adachi-hanga.com/bijin/b10.html
http://www.adachi-hanga.com/sensyu/adachi_koumei/adachi_image_utamaro_naniwayaokita_2.htm
この二枚は矢張り同一人物であるのだろうか

http://www.adachi-hanga.com/bijin/b15.html
下の二人の職業が同じと言うのが興味深いが
当時は茶を汲む女と言う物に人気が有ったのだろうか
603某研究者:03/02/27 10:33 ID:Y3gu6NLW
http://www.city.chiba.jp/art/exhibition/harunobu.html
http://www.saitoan.com/museum/harunobu/harunobu.html
http://www.tvz.com/nishiki-e/nishiki2/ni13.html
まあこの時代にこれだけ現代的な構図・描写を描き得たと言うのは
驚きであるが外国の美術館に有る物も多いし
西欧が彼の絵を参考にした可能性と言うのはどうなのだろうか
(当時の西欧にこの様な現代的な構図を作り得た者と言うのは
 存在したのだろうか)
604某研究者:03/02/27 10:38 ID:Y3gu6NLW
http://www.ne.jp/asahi/kaki/hp/uk_doc05.htm
印象派の画家が浮世絵を参考としたのは有名だろうが
もし浮世絵が無かったら印象派も無かったのだろうか
矢張り人物の描写や構図に上の画家の影響が
かなり有る様に感じるが
605某研究者:03/02/27 10:43 ID:Y3gu6NLW
矢張り同時期の西欧の絵画には自由な構図等は無いだろうし
民主主義の無い国の絵画の方が自由が有ったと言うのも
皮肉な事では有るだろうが(苦笑
606某研究者:03/02/27 10:45 ID:Y3gu6NLW
つまり浮世絵が無ければ西欧の文化に今程の自由は無く
ビートルズも無かったと言う可能性さえも有るのだろうか(苦笑
(まあ別の自由を手に入れた可能性も有るだろうが
 手本と成る物が無いとすればかなり自由化が遅れた可能性も
 有る訳だろうが)
607某研究者:03/02/27 10:49 ID:Y3gu6NLW
まあ矢張り文化の自由が無ければ世界の民主化も遅れていただろうし
科学の進歩も恐らく遅れていたのではないのか
608某研究者:03/02/27 11:41 ID:Y3gu6NLW
http://www.aurora.dti.ne.jp/~ssaton/meisyo/daienji.html
中央の右の絵は女が剣を持っている様にも見えるが
どうなのだろうか
609某研究者:03/02/27 12:00 ID:Y3gu6NLW
http://www.jti.co.jp/Culture/museum/ukiyoe/864-j.html
上の双六はかなり細かい様な気もするが
当時の物は残っているのだろうか
(まあしかし当時の炬燵は妙に小さい様だが)
610某研究者:03/02/27 12:04 ID:Y3gu6NLW
http://www.tt.rim.or.jp/~haruto/ukiyoe/eshi/harunobu.html
まあ彼も5年しか錦絵を描けなかった訳だし
彼がもう少し生きていたら一体どんな物が出ていたか想像も付かず
世界の美術の歴史・或いは日本の歴史も変わったと言う可能性さえ有るだろうか
611某研究者:03/02/27 12:17 ID:Y3gu6NLW
http://www.meiyo.co.jp/kanban.html
まあ上の笠森お仙は赤い前垂れ等は着ていない様にも見えるが
この絵は実物を参考としたのだろうか
612某研究者:03/02/27 12:36 ID:Y3gu6NLW
http://www.ne.jp/asahi/hon/bando-1000/tama/tjo/j014/j014t.htm
まあしかしお仙人形・お仙双六・お仙手拭い等と言うのが
当時本当に出ていたのだろうか(苦笑
(まあ依然魔女狩りが続いている当時の西欧では
 この様な事は有り得なかった訳なのだろうか)
彼女のライバルとの戦いと言うのも妙に現代的であるが
これは事実なのだろうか
(今更宮本武蔵等の男中心の時代劇等を描くよりは
 女が中心であるこの話でも作った方が面白いのではないのか(苦笑)
613某研究者:03/02/27 12:43 ID:Y3gu6NLW
まあしかし鈴木春信の錦絵には平賀源内の工夫が入っている
というが具体的にはどう言う物なのだろうか
614某研究者:03/02/27 12:48 ID:Y3gu6NLW
まあお仙は突然姿を消したから忍者の嫁に成ったのだと言う
噂が出たのか・或いは本当に忍者の嫁に成ったと言う事なのだろうか
615某研究者:03/02/27 12:54 ID:Y3gu6NLW
http://www.kiseido.com/printss/p10-j.htm
おきたとおひさが囲碁で対戦したと言うのは
恐らく作り話だろうが
女同士の対戦と言うのは当時有ったのだろうか
(まあ矢張りこの様な物を時代劇としたら面白いのではないのか(苦笑)
616某研究者:03/02/27 13:02 ID:Y3gu6NLW
>女同士の対戦と言うのは当時有ったのだろうか

囲碁はこれだけ絵が残っているのだから恐らく有るだろうが
将棋或いは武術等に関してはどうなのだろうか

特定の美人を宣伝してはならないと言う幕府の禁令が有った様だが
これは何故になのだろうか
617某研究者:03/02/27 13:59 ID:Y3gu6NLW
http://www.lian.com/TANAKA/comhosei/print/photo/pp00041.jpg
まあ春信の手に成ると達磨も女に成るのだろうか(笑

http://www.jmdb.ne.jp/1934/bj001860.htm
おせんの映画は有った様だが
これは随分前の物だろうか

http://www.lian.com/TANAKA/comhosei/textile/photott/tt00020.jpg
中国やインドの布で出来た帯をしている女も
存在した様だが矢張りこれは高価だろうし
日本に入ったのは少数である訳だろうか
618某研究者:03/02/27 14:12 ID:Y3gu6NLW
http://www.lian.com/TANAKA/comhosei/textile/photott/tt00013.jpg
インドの布で出来た陣羽織
http://www.lian.com/TANAKA/comhosei/textile/photott/tt00015.jpg
http://www.lian.com/TANAKA/comhosei/textile/photott/tt00016.jpg
インドの布で出来た茶入れ・筆入れ等も有る様だが

http://www.lian.com/TANAKA/comhosei/textile/photott/tt00017.jpg
和製の更紗等も有る様だが
数はどうだったのだろうか

http://www.lian.com/TANAKA/comhosei/textile/photott/tt00019.jpg
http://www.lian.com/TANAKA/comhosei/textile/photott/tt00021.jpg
江戸時代の服や陣羽織にもインド的な紋章が有る様だが
どの程度使われたのだろうか

http://www.lian.com/TANAKA/comhosei/textile/photott/tt00022.jpg
火消し・赤穂浪士・新撰組の陣羽織の袖の部分のデザインは
インド起源の物(トゥンパル)であると言う事だろうか
619某研究者:03/02/27 14:24 ID:Y3gu6NLW
http://www.lian.com/TANAKA/comhosei/textile/photott/tt00023.jpg
着物の縞模様と言うのも日本独自の物では無く
インドの影響との事だが

http://www.lian.com/TANAKA/comhosei/textile/photott/tt00026.jpg
これもインドからの輸入品或いは国産のインドの影響を受けた
服である様だが

http://www.lian.com/TANAKA/comhosei/textile/photott/tt00033.jpg
まあこれは男装の女の様だが
服が随分派手だがこれは日本の物なのだろうか

矢張りオランダから輸入されたインドの布の文様が着物の柄に
かなり影響を与えた様だが幕府も西欧の文化の流入は悪としていたが
インドや中国に関しては規制はしなかったと言う事なのだろうか
620某研究者:03/02/27 14:54 ID:Y3gu6NLW
http://www.lian.com/TANAKA/comhosei/textile/photott/tt00036.jpg
伊達正宗の陣羽織だが大河ドラマもリアリティを追求するなら
この様な物を着せるべきだろうか
(まあ秀吉等は
 http://www.lian.com/TANAKA/comhosei/textile/photott/tt00039.jpg
 上の様にペルシャ絨毯を陣羽織にしていた訳だろうし
 其れも再現されないのだろうか)

http://www.lian.com/TANAKA/comhosei/textile/photott/tt00037.jpg
上杉謙信の其れも派手であるが
水玉模様は当時の流行でありこれは日本のオリジナルで有ったのだろうか

http://www.lian.com/TANAKA/comhosei/textile/photott/tt00041.jpg
http://www.lian.com/TANAKA/comhosei/textile/photott/tt00042.jpg
http://www.lian.com/TANAKA/comhosei/textile/photott/tt00043.jpg
http://www.lian.com/TANAKA/comhosei/textile/photott/tt00044.jpg
この柄の服は流石に日本のオリジナルだろうが
621某研究者:03/02/27 15:11 ID:Y3gu6NLW
http://www.lian.com/TANAKA/comhosei/print/photo/pp00054.jpg
メガネを掛けると映像が見えると言うのは
西欧のSF等からアイデアは得たのだろうか

http://www.lian.com/TANAKA/comhosei/print/photo/pp00045.jpg
まあ鈴木春信の偽者(司馬江漢)も銅版画を亜細亜で
最初に作ったと言うから只者では無い訳だろうか(苦笑
622某研究者:03/02/27 15:17 ID:Y3gu6NLW
http://www.tokyo-np.co.jp/tokyo-art/top.html
矢張り鈴木春信の弟子も只者では無かった訳だろうが
鈴木春信がもう少し生きていても面白かっただろうか(笑
623まさ:03/02/27 21:02 ID:???
ところで、某研究者さん、キシュツかもしれないが。剣術の研究をするなら、アメリカ・インディアンとヒィリピンの武器格闘術も研究スルべきと、思われ。
これらは、短い武器を使用するためか、現代の軍隊でも、近接格闘でも応用されているので、研究する必要があるのでは?
まあ、余計なおせは、かもしれないが?
624某研究者:03/02/28 17:16 ID:32cBq+Va
まあ矢張り鈴木春信や写楽が後10年絵を描いていたら
日本或いは世界の歴史も変わっていたかも知れぬし
WW2も無かった可能性も有る訳だろうか
625某研究者:03/02/28 17:33 ID:32cBq+Va
>まあ矢張り鈴木春信や写楽が後10年絵を描いていたら

写楽等は1年も絵を描いていないし
彼が後数年描いただけでも浮世絵の印象と言うのは
かなり違っていたかのも知れぬが(苦笑
626名無し三等兵:03/03/01 00:39 ID:???
げ、元気ですなぁ>閣下
627某研究者:03/03/01 09:00 ID:JLrtWjz8
彼等だけでは無く他の浮世絵画家に関しても
時々奇抜な構図・描写が見られる事は有る訳だろうし
海外の博物館等に有る未公開の物も多いので今後の此れ等の公開と言うのが
楽しみである訳だが
628某研究者:03/03/01 09:42 ID:JLrtWjz8
>これらは、短い武器を使用するためか、現代の軍隊でも、近接格闘でも応用されているので、研究する必要があるのでは?

まあ敵も短い武器なら良いだろうが
間合いの長い銃剣等を持っていた場合は厳しいかも知れぬが
銃の重量は重いので短剣でも対抗は可能だろうし
短剣の方が有利である可能性も有るだろうか
629名無し三等兵:03/03/01 23:22 ID:n4X8iG+m
まず、単発レスです。
近代軍の銃剣の戦い方についてですが、二次大戦の近接戦闘で全体的に強かったのは
やはり日本軍が筆頭に上がりますが、フランスがアジアに派遣していた外人部隊の
銃剣の戦い方に面白いものがあります。
日本軍は勢いを利用する突撃型の攻撃法(つまり突進)なのですが。
フランス外人部隊の白兵戦法は銃剣でフェイントを掛けて(つまり刃に敵の注意を向けて)
重い銃床で顎を砕くと言うユニークな戦法を使用しておりました。

既出だったかもしれませんでしたが、参考になれば幸いです。
630某研究者:03/03/02 11:04 ID:dklJXefx
兎も角ヒトラーの言う所の退廃芸術である印象派等が参考とした物が
同盟国である日本に有ったと言うのも皮肉な話だろうが
631創価学会員:03/03/02 11:09 ID:mm/OG0ey
ここにいる皆さんは一体なんですか?
戦争について語るなど平和を侵害するものです。

ここで私たちと一緒に学び、真の平和とは何か考えていきましょう。]
http://ime.nu/jbbs.shitaraba.com/study/168/
632某研究者:03/03/02 12:43 ID:dklJXefx
>戦争について語るなど平和を侵害するものです。

まあ戦争は自衛の為の手段で有って目的では無いと考える者が
平和をどれだけ侵害しているのか
平和主義を唱えてイラクで盾に成りみすみす殺される事等が
果たして正義等を捨てて生き延びる事より正しい事なのかと言う事だが
633某研究者:03/03/02 14:50 ID:dklJXefx
http://133.9.157.108/enpakunishik/FMPro?-db=nishikie.fmj&-lay=layout2&all%5ffields=%8f%74%90%4d&-max=30&-skip=4&-Format=results-big.htm&-max=1&-Find
まあこれ等を見ると現代の漫画等の構図・描写と
殆ど見分けの付かぬ物・或いは其れ以上の物であるし
矢張り鈴木春信は18世紀の中では(或いは19世紀でさえも)最高の画家と言っても
過言では無いかも知れぬが
(無論ルネサンス期の遠近法を開発した画家にも
 意味は有る訳だろうが)
634某研究者:03/03/02 14:51 ID:dklJXefx
>まあこれ等を見ると現代の漫画等の構図・描写と
>殆ど見分けの付かぬ物・或いは其れ以上の物であるし

まあ矢張り自由と言う意味では現代の下手な画家よりも
上である部分も有るのではないのか
635某研究者:03/03/02 15:00 ID:dklJXefx
http://133.9.157.108/enpakunishik/FMPro?-db=nishikie.fmj&-lay=layout2&all%5ffields=%8f%74%90%4d&-max=30&-skip=3&-Format=results-big.htm&-max=1&-Find
これ程動きの有る・或いは他に全く例の無い様な構図を
18世紀に描いていたとは驚きである訳だろうが

http://133.9.157.108/enpakunishik/FMPro?-db=nishikie.fmj&-lay=layout2&all%5ffields=%8f%74%90%4d&-max=30&-skip=1&-Format=results-big.htm&-max=1&-Find
こう言う絵を見ると江戸時代は女性上位であると言うのも
少なく共一部では本当であるのか共思えるが
(江戸末期の浮世絵に出て来る勝気な女性と言うのも
 西欧の影響で出て来た物では無い訳だろうか)
636某研究者:03/03/02 15:02 ID:dklJXefx
>(江戸末期の浮世絵に出て来る勝気な女性と言うのも
> 西欧の影響で出て来た物では無い訳だろうか)

まあこれは本当に沢山出て来るし
下手をすると文明開化等せぬ方が日本の女は
強くなっていた可能性も有るのかも知れぬが(苦笑
637名無し三等兵:03/03/02 15:04 ID:???
>>631
そこ覗いたけど・・・皆本気?ってオモタヨ

宗教についてはどう考えているんだ?>某研究者

これらが戦術や装具に影響を及ぼした点とかないのかね?
638某研究者:03/03/02 15:08 ID:dklJXefx
まあ矢張り日本人には創造性が無い等と言う人間は
春信の絵を見てから先ず物を言うべきだろうか(苦笑
639某研究者:03/03/02 15:22 ID:dklJXefx
>まあ矢張り日本人には創造性が無い等と言う人間は

まあこう言う人間は自分に創造性が無いのを日本人の遺伝子や
文化に創造性が無いと言う事の所為に
したいと言う部分も有る訳だろうが(苦笑
640某研究者:03/03/02 15:30 ID:dklJXefx
江戸時代の女性上位の文化が人類に取り一番新しい
最先端の文化であり
20世紀や現在の西欧中心の依然男性原理が支配している文化は
其れより古い物であり
世界の文化は亜細亜人より脳細胞の少ない白人に支配され退化した
可能性と言うのも正直有るかと思うがどなのだろうか
641某研究者:03/03/02 15:35 ID:dklJXefx
矢張りこんな退化を自然が何時迄も許しておくのかと言う事は
有る訳だろうし近々動きが有る可能性も有る訳だろうか
642某研究者:03/03/02 15:46 ID:dklJXefx
まあ欧米の文化の行き詰まりと言う物を別のスレッドに描いたが
後から見れば欧米文化=男性原理からでは結局はデニス・ロドマン以上の物は
出て来なかったと成る可能性も有るのかも知れないが
643ひみつの検疫さん:2024/08/10(土) 07:28:30 ID:MarkedRes
汚染を除去しました。
644某研究者:03/03/02 15:51 ID:dklJXefx
まあそうすると女性原理から出て来た最上の物が
鈴木春信の絵で有ると言う可能性は無いのだろうか
(無論これは男が描いた物であると言う意見も有るだろうが
 彼の描く男というのは女と見分けが付かぬ場合も有るし
 矢張り本質的には女性原理・女性的な感覚に支配されていると
 言えないだろうか)
無論少女漫画と言うのが完全に女性原理に支配された物であると言うなら
現在迄に女性原理から出た最上の物である可能性も有る訳だろうが
645某研究者:03/03/02 15:56 ID:dklJXefx
まあ矢張り米英や其れを信仰する物の限界=男性原理の限界と見て
良い訳だろうか
646某研究者:03/03/02 16:04 ID:dklJXefx
まあ矢張り何故か男性を女性化するのではなく
女性を男性化せねば成らぬと言う使命感を持った
欧米文化=男性原理の頂点である米英や
其れを信仰する物の限界=男性原理の限界と見て
良い訳だろうか
647某研究者:03/03/02 16:12 ID:dklJXefx
まあしかし女は完全な男性化・
男は完全な女性化では無く中性化を望んでいると言う可能性も有るだろうし
矢張りどちからに偏ると思考のバランスが損なわれると言う部分も
有るが故に中性化を望んでいると言う可能性も有る訳だろうから
今後完全に女性原理が勝利するのかも疑問である部分は有る訳だろうから
中性的な言語である東京弁を話す日本の人間と言うのは
ある種世界の先端である可能性も有るのかも知れないが
648某研究者:03/03/02 16:14 ID:dklJXefx
まあ京都の言語は女性化しており
東京の言語も京都程では無いが女性化している訳だろうし
女性原理が男性原理より本質的に劣るならこの様な現象は
有り得るのかと言う事であり
京都弁や東京弁を話す事で男性もある程度は女性化され
女性的な原理にも支配されていると言える訳だろうが

まあしかし女は完全な男性化・
男は完全な女性化では無く中性化を望んでいると言う可能性も有るだろうし
矢張りどちからに偏ると思考のバランスが損なわれると言う部分も
有るが故に世界は中性化を望んでいると言う可能性も有る訳だろうから
今後完全に女性原理が勝利するのかも疑問である部分は有る訳だろうから
中性的な言語である東京弁を話す日本の人間と言うのは
ある種世界の先端である可能性も有るのかも知れないが
649某研究者:03/03/02 16:26 ID:dklJXefx
>欧米文化=男性原理の頂点である米英や
>其れを信仰する物の限界=男性原理の限界と見て
>良い訳だろうか

日本人や世界の人間がこれを認めても
自然がこれを古い物として認めず
彼等は文化・人類の進歩を望んでいる自然に
ネアンデルタール人の様に排除される可能性も有る訳だろうか
650某研究者:03/03/02 16:30 ID:dklJXefx
まあ男性原理の勝利=米英の対イラク戦争勝利後
米英の覇権主義=男性原理に拠る世界の支配は限界に達し
米英の文化的主導権・覇権は失われる可能性も有る訳だろうか
651某研究者:03/03/02 17:00 ID:dklJXefx
http://133.9.157.108/enpakunishik/FMPro?-db=nishikie.fmj&-lay=layout2&-error=search_error.htm&-format=results.htm&eshi=文調&-max=10&-Find
まあ春信の影響か彼の絵にもかなり動きが有るが
春信との合作の絵も彼には確か有っただろうか
652某研究者:03/03/02 17:05 ID:dklJXefx
http://133.9.157.108/enpakunishik/FMPro?-db=nishikie.fmj&-lay=layout2&eshi=%95%b6%92%b2&-max=10&-skip=8&-Format=results-big.htm&-max=1&-Find
彼の描く女も春信に似ているが
何となく堅苦しく型に嵌った様な感じがするが
653某研究者:03/03/02 17:15 ID:dklJXefx
まあしかし上に有る人物の衣装と言うのが
余りにも多種多様で驚くが
これは彼のオリジナルではなく実物を写した物なのだろうか
(まあ実物ではなくオリジナルであるにしても
 これだけの種類の物を考えられる彼の創造力と言うのは大した物だろうし
 実物であるなら江戸の文化程度と言うのは
 かなりの物だろうか
 矢張り時代劇等で出て来る衣装の種類はこれ等の衣装が実物であれば
 史実より余りにも少ないと言う事だろうか)
654名無し三等兵:03/03/02 17:18 ID:c4ZNDbqX
↓↓↓↓↓★ピンクエンジェル★↓↓↓↓↓
http://www.pink-angel.jp/betu/linkvp/linkvp.html
655某研究者:03/03/02 17:23 ID:dklJXefx
http://133.9.157.108/enpakunishik/FMPro?-db=nishikie.fmj&-lay=layout2&eshi=%95%b6%92%b2&-max=10&-skip=20&-Format=results-big.htm&-max=1&-Find

まあこれは女の様にも見えるが
矢張り男装の女なのだろか
(まあ女が舞台に上がるのは確か禁止されていた筈だが
 江戸の3美人の中には確か役者が居なかっただろうか
 矢張りあれは女装の男では無い訳だろうか(苦笑)
656某研究者:03/03/02 17:33 ID:dklJXefx
http://133.9.157.108/enpakunishik/FMPro?-db=nishikie.fmj&-lay=layout2&eshi=%95%b6%92%b2&-max=10&-skip=26&-Format=results-big.htm&-max=1&-Find
まあ上の絵も男装の女だろうか

http://133.9.157.108/enpakunishik/FMPro?-db=nishikie.fmj&-lay=layout2&eshi=%95%b6%92%b2&-max=10&-skip=29&-Format=results-big.htm&-max=1&-Find
この絵は中々動きが有るし漫画の表紙の様だが
(彼等の着物も皆時代劇に出て来る物より
 何かセンスが有り妙に現代的である様に感じるが
 気の所為だろうか)
657某研究者:03/03/02 17:36 ID:dklJXefx
まあ矢張り浮世絵の構図も単に奇抜なだけは無く現代のレベルに迄
完成された物も有る訳だろうか
(正直現代の物と比べてさえ練られており
 完成度の高い物が有るかと思うが
 兎も角上の様な構図を見て日本人に創造性が無い等と
 誰が言えるのだろうか)
658某研究者:03/03/02 17:48 ID:dklJXefx
http://133.9.157.108/enpakunishik/FMPro?-db=nishikie.fmj&-lay=layout2&eshi=%95%b6%92%b2&-max=10&-skip=30&-Format=results-big.htm&-max=1&-Find
こう言う飾りも余り見ないし未だ江戸時代には
未知の物が沢山有ると言う事だろうか
(この種の道具も時代劇に出て来る物より
 皆繊細で緻密である様に感じるがどうだろうか)

http://133.9.157.108:591/image/enpaku/nishikie/030-0/030-0032.jpg
これは背中の箱の文字や人物の格好が
妙に現代的では有る訳だろうし中々箱や中身の品質も高そうだが

http://133.9.157.108:591/image/enpaku/nishikie/030-0/030-0033.jpg
演技で使われるのは矢張り木刀なのだろうか

http://133.9.157.108:591/image/enpaku/nishikie/030-0/030-0036.jpg
まあこれは長刀2本を挿している様にも見えるが
気の所為だろうが
この服の模様も余り見ないが演劇で使われる物は
同じ物は2つは無いと見て良いのだろうか
659某研究者:03/03/02 18:07 ID:dklJXefx
http://133.9.157.108:591/image/enpaku/nishikie/030-0/030-0038.jpg
彼も打刀二本を挿している様に見えるが
2刀流で長刀を2本使う剣術と言うのも
有る訳なのだろうか

http://133.9.157.108:591/image/enpaku/nishikie/030-0/030-0039.jpg
まあこれ等は矢張り下手な現代アート等より服の模様は
面白く無いだろうか
(まあ結局世界や存在には価値が無い・人間に生きる意味等無いと言う事しか
 最早言う事の出来ない現代アート等より私に取ってはこちらの方が余程価値は有るが(苦笑)

http://133.9.157.108/enpakunishik/FMPro?-db=nishikie.fmj&-lay=layout2&eshi=%95%b6%92%b2&-max=10&-skip=39&-Format=results-big.htm&-max=1&-Find
看板等の字は皆綺麗で整っており
只書いているのではなくデザイン的な意図が感じられるが
これも実物の写しであるなら職人の腕であると言う事なのか
(乱れた字は乱れた精神を現すと言うが
 この字を見る限り秩序は維持され乍らも創造性も同時に
 相当程度存在していたと見るべきだろうか)

660某研究者:03/03/02 18:21 ID:dklJXefx
http://133.9.157.108/enpakunishik/FMPro?-db=nishikie.fmj&-lay=layout2&eshi=%95%b6%92%b2&-max=10&-skip=43&-Format=results-big.htm&-max=1&-Find
障子の向こうに人物の影の様な物が見えるが
まあ何か2つと同じ様な絵は描かないと言う様な
作者の精神の様な物も見える訳だが
(矢張りこう言う影を利用した演出と言うのが
 この時点で存在していたと言う事だろうか)

http://133.9.157.108/enpakunishik/FMPro?-db=nishikie.fmj&-lay=layout2&eshi=%95%b6%92%b2&-max=10&-skip=45&-Format=results-big.htm&-max=1&-Find
http://133.9.157.108/enpakunishik/FMPro?-db=nishikie.fmj&-lay=layout2&eshi=%95%b6%92%b2&-max=10&-skip=47&-Format=results-big.htm&-max=1&-Find
http://133.9.157.108/enpakunishik/FMPro?-db=nishikie.fmj&-lay=layout2&eshi=%95%b6%92%b2&-max=10&-skip=49&-Format=results-big.htm&-max=1&-Find
まあしかしこの服の模様は現代のデザインに
利用出来ぬだろうか(笑
(まあ寧ろ参考に成らぬ物の方が
 少ないかも知れぬが)

http://133.9.157.108:591/image/enpaku/nishikie/030-0/030-0049.jpg
この甲冑の足甲の部分等は
余り見ない型だが
(良く見ると刀の鞘の部分にも滑り止めの紐が巻かれており
 日本刀の装飾も未だ見ぬ様な物も多い訳だろうか)
661某研究者:03/03/02 18:42 ID:dklJXefx
http://133.9.157.108:591/image/enpaku/nishikie/030-0/030-0054.jpg
この手に持っているのは弓矢なのだろうか

http://133.9.157.108:591/image/enpaku/nishikie/030-0/030-0055.jpg
短刀を逆手に構える事も実際有ったのだろうか
(まあしかし彼の服のデザインも良く見ると中々の物だが)

http://133.9.157.108:591/image/enpaku/nishikie/030-0/030-0058.jpg
伊達や上杉の陣羽織に有る水玉模様は
この時代にも残っていたのだろうか(苦笑
(西欧風の衣装を浮世絵では見ないが
 これは禁止されたから消えたのか或いは単に廃れたのだろうか
 まあ信長が米国に進出していたら浮世絵等有ったのか
 江戸時代の様な独自文化が果たして作り得たのかと言う問題は
 有る訳だろうし現実には世界の進歩に取り日本が鎖国をして
 独自文化を創っていた方が良かったと言う可能性も有るのではないのか)

http://133.9.157.108/enpakunishik/FMPro?-db=nishikie.fmj&-lay=layout2&eshi=%95%b6%92%b2&-max=10&-skip=58&-Format=results-big.htm&-max=1&-Find
まあ剥製では無く本物の鹿が舞台に上がっていたのか
(まあしかしこの鹿は妙にリアルだが)
役者が手に持っている枝に付いている物は一体何なのだろうか

http://133.9.157.108/enpakunishik/FMPro?-db=nishikie.fmj&-lay=layout2&eshi=%95%b6%92%b2&-max=10&-skip=59&-Format=results-big.htm&-max=1&-Find
http://133.9.157.108:591/image/enpaku/nishikie/030-0/030-0062.jpg
この女は割りに表情が出ているかと思うが
矢張り上の物の服の柄は構成が複雑であり余り見た事の無い物だろうか
662某研究者:03/03/02 19:14 ID:dklJXefx
http://133.9.157.108:591/image/enpaku/nishikie/030-0/030-0063.jpg
この女の髪型も余り見ないし
服の柄も同様だろうか

http://133.9.157.108:591/image/enpaku/nishikie/030-0/030-0067.jpg
http://133.9.157.108/enpakunishik/FMPro?-db=nishikie.fmj&-lay=layout2&eshi=%95%b6%92%b2&-max=10&-skip=66&-Format=results-big.htm&-max=1&-Find
まあこれも男か女か良く判らないが
上の二人の服は腕の部分に同じ様なマークが2つ有るが
これは何なのだろうか

http://133.9.157.108:591/image/enpaku/nishikie/030-0/030-0069.jpg
忍者刀でも無いのに四角い刀の鍔が
存在したと言う事なのだろうか

http://133.9.157.108:591/image/enpaku/nishikie/030-0/030-0071.jpg
これも中々表情が出ているが
舞台に猫も上がっていたのだろうか(笑
(爪を研いでいる様にも見えなくも無いが
 どうだろうか)

http://133.9.157.108/enpakunishik/FMPro?-db=nishikie.fmj&-lay=layout2&eshi=%95%b6%92%b2&-max=10&-skip=74&-Format=results-big.htm&-max=1&-Find
この刀は道中指しの様に見えるが
矢張り短刀では無くそれより長い
道中指しを持っていた女も存在していた訳だろうか
(まあ服の柄も矢が掛かれており
 彼女は男性的な女性であると言う事なのだろうか(笑)
663某研究者:03/03/02 19:40 ID:dklJXefx
http://133.9.157.108/enpakunishik/FMPro?-db=nishikie.fmj&-lay=layout2&eshi=%95%b6%92%b2&-max=10&-skip=80&-Format=results-big.htm&-max=1&-Find
まあしかしこんな格好で猫を連れているのは
何か変だが(苦笑

http://133.9.157.108/enpakunishik/FMPro?-db=nishikie.fmj&-lay=layout2&eshi=%95%b6%92%b2&-max=10&-skip=83&-Format=results-big.htm&-max=1&-Find
まあこれは外に居る図でも無いだろうし
雪やつらら迄当時の舞台は妙にリアルに
再現されていたのだろうか

http://133.9.157.108/enpakunishik/FMPro?-db=nishikie.fmj&-lay=layout2&eshi=%95%b6%92%b2&-max=10&-skip=84&-Format=results-big.htm&-max=1&-Find
まあ絵の中で絵を見ていると言う事で
これを描いたのだろうか(笑

http://133.9.157.108:591/image/enpaku/nishikie/030-0/030-0088.jpg
http://133.9.157.108/enpakunishik/FMPro?-db=nishikie.fmj&-lay=layout2&eshi=%95%b6%92%b2&-max=10&-skip=88&-Format=results-big.htm&-max=1&-Find
http://133.9.157.108:591/image/enpaku/nishikie/030-0/030-0091.jpg
これも中々構図が現代的で面白いが
(上と下の絵は何か女性上位的な雰囲気も有るが)


http://133.9.157.108/enpakunishik/FMPro?-db=nishikie.fmj&-lay=layout2&eshi=%95%b6%92%b2&-max=10&-skip=90&-Format=results-big.htm&-max=1&-Find
これも何か女性上位的な雰囲気が有るが
一体どう言う場面なのだろうか

http://133.9.157.108/enpakunishik/FMPro?-db=nishikie.fmj&-lay=layout2&eshi=%95%b6%92%b2&-max=10&-skip=91&-Format=results-big.htm&-max=1&-Find
まあこの二人も表向きには男が演じている事に
成っているのだろうが(苦笑
実際はそうでは無い事も多かったのだろうか
664某研究者:03/03/02 19:55 ID:dklJXefx
http://133.9.157.108/enpakunishik/FMPro?-db=nishikie.fmj&-lay=layout2&eshi=%95%b6%92%b2&-max=10&-skip=93&-Format=results-big.htm&-max=1&-Find
まあ剣道の構えと剣の持ち方が違う様な気もするが
役者は侍では無いなら剣術には無知であった
或いはこう言う構え方も存在した可能性も
有るだろうか

http://133.9.157.108/enpakunishik/FMPro?-db=nishikie.fmj&-lay=layout2&eshi=%95%b6%92%b2&-max=10&-skip=94&-Format=results-big.htm&-max=1&-Find
まあしかし配役を見ると剣を持っているのは一応男の様だが

http://133.9.157.108/enpakunishik/FMPro?-db=nishikie.fmj&-lay=layout2&eshi=%95%b6%92%b2&-max=10&-skip=95&-Format=results-big.htm&-max=1&-Find
まあしかしこれは一体どう言う場面なのか判らないが
複雑な事が表現されているのだろうか

http://133.9.157.108:591/image/enpaku/nishikie/030-0/030-0099.jpg
上の人間は女だろうが
(まあしかし侍以外の女が道中指し以外持てたのだろうか)
下の二人は男なのだろうか

http://133.9.157.108:591/image/enpaku/nishikie/030-0/030-0100.jpg
上の人間は多分男だろうが
舞台には牛迄上がっていたのだろうか(笑
665某研究者:03/03/02 19:59 ID:dklJXefx
http://133.9.157.108/enpakunishik/FMPro?-db=nishikie.fmj&-lay=layout2&eshi=%95%b6%92%b2&-max=10&-skip=99&-Format=results-big.htm&-max=1&-Find
まあ鏡の中の男も描かれているが
この男の服もかなり凄い訳だが(笑

まあ今日は疲れたので此所迄としたいが
矢張り想像以上に構図や服の柄の種類も本質的な差異が多く
切りが無い訳だろうし1枚1枚に殆ど何か
新しい発見が有る訳だろうか
666某研究者:03/03/03 15:41 ID:L1cuMojQ
http://133.9.157.108/enpakunishik/FMPro?-db=nishikie.fmj&-lay=layout2&eshi=%95%b6%92%b2&-max=10&-skip=115&-Format=results-big.htm&-max=1&-Find
袖が長い服を着ている者も
髪飾りの付いているのは女で付いていないのは男である訳だろうか(笑
故に後ろの刀を持っているのは女(無論表向きには男が演じている事に成っているだろうが)
だろうか

http://133.9.157.108/enpakunishik/FMPro?-db=nishikie.fmj&-lay=layout2&eshi=%95%b6%92%b2&-max=10&-skip=112&-Format=results-big.htm&-max=1&-Find
まあしかしこの人形と言うのは動くのだろうか

http://133.9.157.108/enpakunishik/FMPro?-db=nishikie.fmj&-lay=layout2&eshi=%95%b6%92%b2&-max=10&-skip=118&-Format=results-big.htm&-max=1&-Find
しかし桶の中の物は空であると言う事は無いのだろうか(笑

http://133.9.157.108/enpakunishik/FMPro?-db=nishikie.fmj&-lay=layout2&eshi=%95%b6%92%b2&-max=10&-skip=119&-Format=results-big.htm&-max=1&-Find
貴族ばかりでなく七福神の布袋まで劇に出ていたと言う事なのだろうか(笑

http://133.9.157.108/enpakunishik/FMPro?-db=nishikie.fmj&-lay=layout2&eshi=%95%b6%92%b2&-max=10&-skip=122&-Format=results-big.htm&-max=1&-Find
子供の服も中々手が込んでいるが

http://133.9.157.108/enpakunishik/FMPro?-db=nishikie.fmj&-lay=layout2&eshi=%95%b6%92%b2&-max=10&-skip=125&-Format=results-big.htm&-max=1&-Find
こんなある種行儀の悪い格好も劇中・或いは実際に有り得たと言う事なのだろうか(苦笑
667名無し三等兵:03/03/03 15:43 ID:djpRSvWi
↓↓↓↓↓★ピンクエンジェル★↓↓↓↓↓
http://www.pink-angel.jp/betu/linkvp/linkvp.html
668某研究者:03/03/03 15:48 ID:L1cuMojQ
http://133.9.157.108/enpakunishik/FMPro?-db=nishikie.fmj&-lay=layout2&eshi=%95%b6%92%b2&-max=10&-skip=125&-Format=results-big.htm&-max=1&-Find
>こんなある種行儀の悪い格好も劇中・或いは実際に有り得たと言う事なのだろうか(苦笑

こう言う型に嵌っていない女性上位を匂わせる絵と言うのは
今見ても面白い部分は有る訳だろうし
無論魔女狩りを依然続けている当時の西欧でこんな絵は描けなかった訳だろうか
(絵だけでは無く其処に描かれている元々の劇のレベル・其処で表現されている物のレベルも高く
 題材はかなり自由であり多種多様である訳だろうか)
669某研究者:03/03/03 16:18 ID:L1cuMojQ
670某研究者:03/03/03 16:45 ID:L1cuMojQ
http://133.9.157.108/enpakunishik/FMPro?-db=nishikie.fmj&-lay=layout2&eshi=%95%b6%92%b2&-max=10&-skip=145&-Format=results-big.htm&-max=1&-Find
こんな事迄本当にやっていたのかと言う事だが
この石が落ちたら下の女はアウトだろうか(苦笑
(まあこの絵は珍しく女性上位の雰囲気には
 見えないが(苦笑)

http://133.9.157.108/enpakunishik/FMPro?-db=nishikie.fmj&-lay=layout2&eshi=%95%b6%92%b2&-max=10&-skip=158&-Format=results-big.htm&-max=1&-Find
矢張り女がこう言う顔をしている絵は珍しいだろうが
これは単なる創作では無くこう言う表情を劇中や実際にしていたと言う事だろうか(苦笑

http://133.9.157.108/enpakunishik/FMPro?-db=nishikie.fmj&-lay=layout2&eshi=%95%b6%92%b2&-max=10&-skip=161&-Format=results-big.htm&-max=1&-Find
右の様な人物の顔は珍しいが
左の人物の持っている面と言うのは
似た様な物と言うのは現存していないのだろうか
671名無し三等兵:03/03/03 17:28 ID:1cITBO4V

★☆★☆★☆★☆★☆★☆★☆★☆★☆★☆★☆★☆★☆★☆★☆★☆

     手っ取りばやくお金もうけしましょう
     http://www.h4.dion.ne.jp/~sohotop/

      日給2万円も無理じゃない!!!

★☆★☆★☆★☆★☆★☆★☆★☆★☆★☆★☆★☆★☆★☆★☆★☆
672某研究者:03/03/03 17:39 ID:L1cuMojQ
欧州は兎も角米の映画・スポーツ中心の・スポーツから派生して来る文化は
男性原理に支配されている訳だろうか
673某研究者:03/03/04 13:20 ID:BSWm+c+2
http://133.9.157.108/enpakunishik/FMPro?-db=nishikie.fmj&-lay=layout2&eshi=%96%4c%8d%91&-max=30&-skip=5&-Format=results-big.htm&-max=1&-Find
道中指しも両手で扱える物は有るのかも知れないが
脇差しには有るのだろうか
(まあ豊国も割に面白い構図・描写が有るのではないのか)
674某研究者:03/03/04 13:22 ID:BSWm+c+2
>道中指しも両手で扱える物は有るのかも知れないが

まあ低く構えれば短い刀でも
足を守れると言う事は有るだろうが
果たしてこの構えをしても長刀を使うより強いだろうか
675名無し三等兵:03/03/04 13:28 ID:???
最新の音楽情報満載。音楽ファン必見のサイト
音楽ギフト券の全プレあり
http://camellia16.fc2web.com/ongakudb.html
676某研究者:03/03/04 13:54 ID:BSWm+c+2
677某研究者:03/03/04 17:07 ID:BSWm+c+2
>まあ低く構えれば短い刀でも
>足を守れると言う事は有るだろうが

まあ長刀を持って低く構えれば
更に防御力は増すと言う事は
無いのだろうか
678某研究者:03/03/04 17:52 ID:BSWm+c+2
まあ矢張り短い剣では囲まれぬ様走り乍ら
足を防護すると言う事は困難である訳だろうか
679某研究者:03/03/04 17:54 ID:BSWm+c+2
まあしかし木刀では無く竹刀の様な物は
西欧に有ったのだろうか
680某研究者:03/03/04 18:25 ID:BSWm+c+2
http://www.geocities.co.jp/Athlete-Rodos/2981/page004.html
小型・中型の盾の弱点も足だろうが
低く構えられたら足も狙えないだろうか
(視野が限定されるのが盾の欠点(まあ現代の盾は透明な物も有るが)であり
 頭部を狙った後に足等を狙われれば防護不能である場合も
 多いだろうか
 (読まれれば頭部を狙った後又頭部を狙うと言う方向も
  有るだろうが)
 後は盾を持った状態で格闘戦に持ち込まれると
 ロードス島戦記のアシュラム対パーンの様な状況にも
 成り得るだろうか
 しかし盾に刺でも付けておけば
 相手に鎧の無い場合は格闘戦には持ち込めないだろうか(苦笑)
盾も盾の右から攻撃されると剣が出せなく成ると言う欠点が
有る様だが右からの攻撃は剣で防いで盾で敵を攻撃すると言う方向も
有るだろうか
681某研究者:03/03/04 18:40 ID:BSWm+c+2
盾での攻撃等相手が常に見えないなら限度も有るだろうし
盾の右側から剣で連続攻撃を繰り返せば脆いかも知れないが
盾で下半身をカバーし剣で上体をカバーされれば
どうだろうか
まあ2刀で上体を攻撃すれば1本の剣でこれを防げるのかと言う事だし
これを防ぐ為に盾も上に上げれば視界も無くなる訳だろうし足を剣で狙う事も
可能だろうか
682某研究者:03/03/04 19:06 ID:BSWm+c+2
http://thehaca.com/Videos/TPVideos.htm
2刀流や片手剣では上のグレートソードは
受け流せるとさえ思えないが
(この様な高速な太刀筋も達人なら見切れるのか
 見切れても受け流せるのかと言う事だろうが)
これだけ重い剣では打刀では防げず
野太刀で無いと無理だろうか
683某研究者:03/03/04 19:25 ID:BSWm+c+2
まあ上の映像を見ると突きと言うのが切りに比べて
如何に防ぎ易く読み易い攻撃であるのかと言う事が判るだろうが
達人の太刀筋と言うのは全く予測し難く高速である訳だろうか
684某研究者:03/03/04 19:27 ID:BSWm+c+2
矢張り次にどう言うコースで剣を振るのか全く読めず
且つ動作が高速であるから素人は訳の判らぬ内に
下手をすると最初の一撃でやられている訳だろうか
685某研究者:03/03/04 19:31 ID:BSWm+c+2
まあ只西欧の剣術は擦り足では無いからステップが遅く
これは読み易い様な気もするが
(まあ只平らな場所で無いとこれは出来ぬだろうが
 城内や室内・道路等の平らな場所と言うのは有るには有る訳だろうが)
読み難い突きのパターンも幾つかは有る様だが
686某研究者:03/03/04 19:41 ID:BSWm+c+2
>2刀流や片手剣では上のグレートソードは
>受け流せるとさえ思えないが

グレートソードでも上の攻撃が防げるとは思えないが
両手剣対両手剣ではトーナメントアーマーでも着ないと
二人共相打ちに成るのが精々だろうか
(巨大な盾で攻撃コースを限定しても
 これを止められる共思えぬし
 この様な打撃を受ければトーナメントアーマーを着てさえ
 一撃でアウトではないのか(苦笑)
687某研究者:03/03/04 19:49 ID:BSWm+c+2
まあビデオを見る限り両手剣に比べ片手剣の攻撃はかなり遅いし素人でも
次の動きはある程度読め
矢張り2刀で1刀に対すれば最初の一撃でやられかないと言う
印象を受けたが
(矢張りランツクネヒトも上のビデオを見ると
 パイク兵より本当に弱いのか疑問も出て来るが
 パイクの攻撃のスピードと言うのはどの程度高速なのだろうか)
688某研究者:03/03/04 19:57 ID:BSWm+c+2
片手剣の攻撃の大半は何とか目で追えるが
両手剣の攻撃は殆ど目で追えない様なスピードの物が
大半である訳だろうか
(これだけスピードの差が有れば
 女の両手剣に男の片手剣が負ける状況も確かに
 かなり出て来るだろうか
 両手剣は武器で受けるのは困難であり盾と鎧で
 何とか受け流すしか無い類の物であり
 西欧の鎧と盾の装備と言うのは合理的なのかも知れぬが
 現実には鎧等装備出来る兵は少ないだろうが
 盾+手甲・足甲・冑を装備すれば両手剣の攻撃を受け流せる可能性も
 有るだろうか)
689某研究者:03/03/04 19:58 ID:BSWm+c+2
雑魚では盾と完全鎧が有ってもこれを受け流すのは無理だろうし
正規の騎士とて非常に困難だろうか
690某研究者:03/03/04 20:18 ID:BSWm+c+2
上のビデオを見る限りフルプレートを着ると両手剣もスピードが鈍って
太刀筋が見える様に成るから
これでは鎧等装備しない方がましだろうか(苦笑
691某研究者:03/03/04 20:23 ID:BSWm+c+2
まあそうすると現実にはフルプレート等を着たら
鎧を着ていない相手の両手剣の太刀筋は見えず自分に当たり
自分の鎧を着て動きの鈍った両手剣の太刀筋は相手に見られ
防御或いは回避される・つまり一方的にやられると言う事だろうか(苦笑
692某研究者:03/03/04 21:58 ID:bEnpFE5E
まあ相手が見切れない範囲の速度を出せる範囲の重量の刀で
最も長い物を使って相手をアウトレンジするしか
敵を一方的に倒す術は相手が太刀筋の読めぬ達人であれば
無い訳だろうか
(同じ長さの刀で同じレベルの人間がやれば
 太刀筋が読めぬのであれば殆ど最初の一撃で相打ちと成るしか
 無いのではないのか
 矢張り決闘では其れで運良く生き残った方が勝者と成るだけだろうか(苦笑)
読めない高速の太刀筋を用いて一撃で敵を倒せるなら
余り体力を消耗せずにかなりの数の敵を
囲まれねれば倒せるのではないのか
(フルプレートを着ても振りが鈍らぬ体力が有ればこれを着れば
 槍の攻撃等は無力だろうか
 或いはパビスを構えて槍兵に乱入し剣の間合いで戦うと言う方向も
 有るだろうが)
693某研究者:03/03/04 22:05 ID:bEnpFE5E
>まあ相手が見切れない範囲の速度を出せる範囲の重量の刀で
>最も長い物を使って相手をアウトレンジするしか

まあこれを狙って野太刀を使用したのが新撰組だろうし
鎖もまともに野太刀等を受ければ無力だろうか
(鎖も剣速が鈍らぬなら装備しても問題は無いだろうが
 相手は雑魚ばかりでは無いなら鎧は装備せずに敵に太刀筋を見られぬ範囲の重さで
 出来るだけ長い刀を用いた方が良いだろうか)

雑兵相手には体力消耗での振りの鈍りも考えて
やや短い刀を用いても良いだろうが
雑兵と同じ長さの刀では危険かも知れぬだろうから
雑兵の使うよりはやや長い物を用いるべきだろうか
694某研究者:03/03/04 22:12 ID:bEnpFE5E
まあ上のビデオを見て仕舞うと
剣術の力と言うのは殆ど体力であり
より長い剣を太刀筋の見られぬ速さで振える方が勝つだけだろうか(苦笑
695某研究者:03/03/05 09:12 ID:UaGLuzaA
上のビデオを見続けたら太刀筋もある程度見える様に成ったが
日本刀の様な軽い両手剣を高速に振るわれた場合はどうなのだろうか
696某研究者:03/03/05 09:33 ID:UaGLuzaA
まあ上のビデオ程度の攻撃なら達人なら
完全に防護可能であり
無駄に動いて体力が消耗した方が負ける訳だろうか
(まあクリスタニアでレードンが熱い砂漠で防御に徹して相手が疲れるのを待ち
 其処から反撃に転じたと言う戦法は正しい訳なのだろうか)
697某研究者:03/03/05 09:37 ID:UaGLuzaA
まあしかし剣道程度の剣速でさえ
全く体力を消耗していない段階で
達人が敵の攻撃を防御出来ない事も有るから
矢張りスピード以外の要素も重要である・
スピードの無い攻撃でもやられる場合が有ると言う事だろうか
(コンパクトな動きで多彩に変化して来る様な物では
 スピードが遅く共これを読めずやられる場合も有る訳だろうか
 或いは大きな振りを手元で変化させる等の方向ではどうなのだろうか)
698名無し三等兵:03/03/05 09:38 ID:zgsE5sea
↑白痴
699某研究者:03/03/05 09:45 ID:UaGLuzaA
>(コンパクトな動きで多彩に変化して来る様な物では
> スピードが遅く共これを読めずやられる場合も有る訳だろうか

しかしコンパクトな動きでは打撃力は落ち
相手が鎧を着ている場合は無効である場合も有るだろうか


>或いは大きな振りを手元で変化させる等の方向ではどうなのだろうか)

これであれば鎧を着た相手も倒せるのかも知れぬが
この手の技で相手に攻撃を読ませず当てられるだろうか
700名無し三等兵:03/03/05 09:49 ID:wFQlKbaZ
どーでもいいけど、あんたなんで語尾が疑問形なのよ
701某研究者:03/03/05 09:51 ID:UaGLuzaA
>しかしコンパクトな動きでは打撃力は落ち
>相手が鎧を着ている場合は無効である場合も有るだろうか

重い剣をコンパクト且つ高速に多方向に振るわれれば
これには対応困難であり打撃力も有るなら
鎧を着ていても厳しい訳だろうか
702某研究者:03/03/05 09:57 ID:UaGLuzaA
フェンシングの剣の振りはコンパクトで高速であり
これには慣れていないと対応困難だろうが
鎧を着た相手には重い剣であれを出来ないと
駄目だろうか
703bloom:03/03/05 09:59 ID:D1DGidtD
704某研究者:03/03/05 12:41 ID:UaGLuzaA
まあ素人では上のビデオの振りでも見る事は出来ず
一撃でやられる危険は有るだろうし
良くて数回の攻撃を防護するのが限度だろうが
横や下からの攻撃と言うのは素人に
防護する術と言うのは有るのだろうか
燕返しと言うのは仮に一撃目を防御出来ても
2撃目の下からの攻撃を素人では防げないだろうか

矢張り変則な技(これは無論敵に見て対応されぬ為に
突然に・高速に行う必要は有るだろうし
決闘等を行う場合は決戦の日迄隠しておく必要も有るだろうか)
も達人同士の戦いでは重要だろうが
剣の峰や平の部分・柄での打撃や
両刃の刀の逆の刃を利用して切る様な戦術も
有り得る訳だろうか
705某研究者:03/03/05 13:00 ID:UaGLuzaA
棒の先端にスパイクを付けてこれを多方向に振り回せば
敵を傷付ける事は刃の向きが限定されている剣よりも
容易である場合も有るだろうか
(単なる棒と異なりコンパクトな振りでも
 敵を傷付けられるだろうか)
706某研究者:03/03/05 15:37 ID:UaGLuzaA
江戸時代の文化は本質的には女性上位の文化だろうが
武道も同時に存在しており男性原理も軽視されていた訳では
無い訳だろうか
(市民は女性上位の文化に支配されていたが
 侍・或いは侍の女はそうでは無いと言う事だろうか)


http://www.thearma.org/Videos/NTCvids/testingbladesandmaterials.htm
上の剣で兜を割るビデオを見ると
かなりコンパクトなモーションからの剣戟でも
兜は大きく凹んでいる訳だが
矢張りこれは西欧の剣の重さと使い手の筋力の強さに
拠る物だろうか
(よりコンパクトなモーションからの一撃でも
 或いは軽い武器でも彼の様なパワーが有れば
 鎧を着た人間を一撃で戦闘不能とする事は可能だろうし
 一撃で倒せず共動きを鈍らせる事なら彼より力が無く共
 可能だろうか)
しかし西欧の兜の厚みより日本の其れの方が
上と言う事は無いのだろうか
(まあ日本の筋が入っている兜・或いはマクシミリアン鎧の
 筋の入っている兜でも片手剣でモーションの大きい攻撃を受ければ
 雑兵の力でさえ兜を被った人間は昏倒・或いは即死だろうか
 日本刀の打撃でも両手で振るえばモーションが小さく共
 ほぼ同様の事に成るかも知れぬが)
707某研究者:03/03/05 15:44 ID:UaGLuzaA
まあ片手剣のモーションの小さい打撃でもこれだけ凹むのでは
パワーの有る者の両手剣なら鎧を着た人間の腕位なら
モーションを大きくすれば両断も十分可能だろうか
(ベルセルク等で両手剣や斧で鎧が切断されていたのも
 別段非現実的では無いと言う事だろうが
 グインサーガの様に両手剣で鎧を着た人間を頭から
 一刀両断する等と言う事は流石に可能なのかだが
 鎧を着ていない人間の胴体を一撃で両断する位なら
 グインの力なら可能だろうか)
708某研究者:03/03/05 15:53 ID:UaGLuzaA
まあそうすると鎧等モーションのかなり小さい攻撃
或いは槍等の軽い武器の攻撃
或いはモーションの大きい攻撃が掠めたのを
防げる程度であると言う事だろうか
(鎖帷子等モーションのかなり小さい攻撃も防げず
 攻撃が掠ったのを止められる程度であるなら
 30kg等の重量の物を着て振りを鈍くする意味は
 有るだろうか)
709某研究者:03/03/05 16:07 ID:UaGLuzaA
まあビデオは全部は見ていないが両手剣であれば突きで恐らく鎧を
貫通する事は可能だろうし
より軽量な剣・或いは棒等でも鎧を貫通せぬ迄もかなりのダメージを与える事が
可能ではないのか
(鎧の隙間等態々狙わなく共
 余程軽量な剣で無ければ鎧の上から突きで
 敵を仕留める事も可能だろうか)
710某研究者:03/03/05 16:29 ID:UaGLuzaA
剣でこれだけ鎧を凹ませる事が出来るなら
斧やメイス等無用の様にも見えるが
より小さなモーションで敵を倒せると言うメリットも
有る訳だろうか
(まあ短いメイスで敵の剣の間合いの内側に踏み込んだ後
 連打と言う様な事も可能だろうか)
711まあまあまあまあまあまあ:03/03/05 16:39 ID:???
まあまあまあまあまあまあまあまあまあまあまあまあまあまあまあまあまあまあ
まあまあまあまあまあまあまあまあまあまあまあまあまあまあまあまあまあまあ
まあまあまあまあまあまあまあまあまあまあまあまあまあまあまあまあまあまあ
まあまあまあまあまあまあまあまあまあまあまあまあまあまあまあまあまあまあ
まあまあまあまあまあまあまあまあまあまあまあまあまあまあまあまあまあまあ
まあまあまあまあまあまあまあまあまあまあまあまあまあまあまあまあまあまあ
まあまあまあまあまあまあまあまあまあまあまあまあまあまあまあまあまあまあ
まあまあまあまあまあまあまあまあまあまあまあまあまあまあまあまあまあまあ
まあまあまあまあまあまあまあまあまあまあまあまあまあまあまあまあまあまあ
まあまあまあまあまあまあまあまあまあまあまあまあまあまあまあまあまあまあ
まあまあまあまあまあまあまあまあまあまあまあまあまあまあまあまあまあまあ
まあまあまあまあまあまあまあまあまあまあまあまあまあまあまあまあまあまあ
まあまあまあまあまあまあまあまあまあまあまあまあまあまあまあまあまあまあ
まあまあまあまあまあまあまあまあまあまあまあまあまあまあまあまあまあまあ
まあまあまあまあまあまあまあまあまあまあまあまあまあまあまあまあまあまあ
まあまあまあまあまあまあまあまあまあまあまあまあまあまあまあまあまあまあ
まあまあまあまあまあまあまあまあまあまあまあまあまあまあまあまあまあまあ
まあまあまあまあまあまあまあまあまあまあまあまあまあまあまあまあまあまあ
まあまあまあまあまあまあまあまあまあまあまあまあまあまあまあまあまあまあ
まあまあまあまあまあまあまあまあまあまあまあまあまあまあまあまあまあまあ
まあまあまあまあまあまあまあまあまあまあまあまあまあまあまあまあまあまあ
まあまあまあまあまあまあまあまあまあまあまあまあまあまあまあまあまあまあ
まあまあまあまあまあまあまあまあまあまあまあまあまあまあまあまあまあまあ
まあまあまあまあまあまあまあまあまあまあまあまあまあまあまあまあまあまあ
まあまあまあまあまあまあまあまあまあまあまあまあまあまあまあまあまあまあ
712某研究者:03/03/05 16:39 ID:UaGLuzaA
まあこれでは剣と剣が当たったら
まともな状態では済まないだろうが
盾も両手剣の攻撃を受ければ
かなり凹むだろうし貫通する危険も
有る訳だろうか
(片手剣でも腕を通している場所を打撃されれば
 厳しいだろうしこの部分にはスペースが必要だろうか)
ランスが騎兵・歩兵或いはパイクが走行中の騎兵を正面から刺した場合は
盾や鎧等何の問題も無く貫通する訳だろうか
713まあまあまあまあまあまあ:03/03/05 16:42 ID:???
まあまあまあまあまあまあまあまあまあまあまあまあまあまあまあまあまあまあ
まあまあまあまあまあまあまあまあまあまあまあまあまあまあまあまあまあまあ
まあまあまあまあまあまあまあまあまあまあまあまあまあまあまあまあまあまあ
まあまあまあまあまあまあまあまあまあまあまあまあまあまあまあまあまあまあ
まあまあまあまあまあまあまあまあまあまあまあまあまあまあまあまあまあまあ
まあまあまあまあまあまあまあまあまあまあまあまあまあまあまあまあまあまあ
まあまあまあまあまあまあまあまあまあまあまあまあまあまあまあまあまあまあ
まあまあまあまあまあまあまあまあまあまあまあまあまあまあまあまあまあまあ
まあまあまあまあまあまあまあまあまあまあまあまあまあまあまあまあまあまあ
まあまあまあまあまあまあまあまあまあまあまあまあまあまあまあまあまあまあ
まあまあまあまあまあまあまあまあまあまあまあまあまあまあまあまあまあまあ
まあまあまあまあまあまあまあまあまあまあまあまあまあまあまあまあまあまあ
まあまあまあまあまあまあまあまあまあまあまあまあまあまあまあまあまあまあ
まあまあまあまあまあまあまあまあまあまあまあまあまあまあまあまあまあまあ
まあまあまあまあまあまあまあまあまあまあまあまあまあまあまあまあまあまあ
まあまあまあまあまあまあまあまあまあまあまあまあまあまあまあまあまあまあ
まあまあまあまあまあまあまあまあまあまあまあまあまあまあまあまあまあまあ
まあまあまあまあまあまあまあまあまあまあまあまあまあまあまあまあまあまあ
まあまあまあまあまあまあまあまあまあまあまあまあまあまあまあまあまあまあ
まあまあまあまあまあまあまあまあまあまあまあまあまあまあまあまあまあまあ
まあまあまあまあまあまあまあまあまあまあまあまあまあまあまあまあまあまあ
まあまあまあまあまあまあまあまあまあまあまあまあまあまあまあまあまあまあ
まあまあまあまあまあまあまあまあまあまあまあまあまあまあまあまあまあまあ
まあまあまあまあまあまあまあまあまあまあまあまあまあまあまあまあまあまあ
まあまあまあまあまあまあまあまあまあまあまあまあまあまあまあまあまあまあ
714まあまあまあまあまあまあ:03/03/05 16:42 ID:???
まあまあまあまあまあまあまあまあまあまあまあまあまあまあまあまあまあまあ
まあまあまあまあまあまあまあまあまあまあまあまあまあまあまあまあまあまあ
まあまあまあまあまあまあまあまあまあまあまあまあまあまあまあまあまあまあ
まあまあまあまあまあまあまあまあまあまあまあまあまあまあまあまあまあまあ
まあまあまあまあまあまあまあまあまあまあまあまあまあまあまあまあまあまあ
まあまあまあまあまあまあまあまあまあまあまあまあまあまあまあまあまあまあ
まあまあまあまあまあまあまあまあまあまあまあまあまあまあまあまあまあまあ
まあまあまあまあまあまあまあまあまあまあまあまあまあまあまあまあまあまあ
まあまあまあまあまあまあまあまあまあまあまあまあまあまあまあまあまあまあ
まあまあまあまあまあまあまあまあまあまあまあまあまあまあまあまあまあまあ
まあまあまあまあまあまあまあまあまあまあまあまあまあまあまあまあまあまあ
まあまあまあまあまあまあまあまあまあまあまあまあまあまあまあまあまあまあ
まあまあまあまあまあまあまあまあまあまあまあまあまあまあまあまあまあまあ
まあまあまあまあまあまあまあまあまあまあまあまあまあまあまあまあまあまあ
まあまあまあまあまあまあまあまあまあまあまあまあまあまあまあまあまあまあ
まあまあまあまあまあまあまあまあまあまあまあまあまあまあまあまあまあまあ
まあまあまあまあまあまあまあまあまあまあまあまあまあまあまあまあまあまあ
まあまあまあまあまあまあまあまあまあまあまあまあまあまあまあまあまあまあ
まあまあまあまあまあまあまあまあまあまあまあまあまあまあまあまあまあまあ
まあまあまあまあまあまあまあまあまあまあまあまあまあまあまあまあまあまあ
まあまあまあまあまあまあまあまあまあまあまあまあまあまあまあまあまあまあ
まあまあまあまあまあまあまあまあまあまあまあまあまあまあまあまあまあまあ
まあまあまあまあまあまあまあまあまあまあまあまあまあまあまあまあまあまあ
まあまあまあまあまあまあまあまあまあまあまあまあまあまあまあまあまあまあ
まあまあまあまあまあまあまあまあまあまあまあまあまあまあまあまあまあまあ 
715まあまあまあまあまあまあ:03/03/05 16:42 ID:???
まあまあまあまあまあまあまあまあまあまあまあまあまあまあまあまあまあまあ
まあまあまあまあまあまあまあまあまあまあまあまあまあまあまあまあまあまあ
まあまあまあまあまあまあまあまあまあまあまあまあまあまあまあまあまあまあ
まあまあまあまあまあまあまあまあまあまあまあまあまあまあまあまあまあまあ
まあまあまあまあまあまあまあまあまあまあまあまあまあまあまあまあまあまあ
まあまあまあまあまあまあまあまあまあまあまあまあまあまあまあまあまあまあ
まあまあまあまあまあまあまあまあまあまあまあまあまあまあまあまあまあまあ
まあまあまあまあまあまあまあまあまあまあまあまあまあまあまあまあまあまあ
まあまあまあまあまあまあまあまあまあまあまあまあまあまあまあまあまあまあ
まあまあまあまあまあまあまあまあまあまあまあまあまあまあまあまあまあまあ
まあまあまあまあまあまあまあまあまあまあまあまあまあまあまあまあまあまあ
まあまあまあまあまあまあまあまあまあまあまあまあまあまあまあまあまあまあ
まあまあまあまあまあまあまあまあまあまあまあまあまあまあまあまあまあまあ
まあまあまあまあまあまあまあまあまあまあまあまあまあまあまあまあまあまあ
まあまあまあまあまあまあまあまあまあまあまあまあまあまあまあまあまあまあ
まあまあまあまあまあまあまあまあまあまあまあまあまあまあまあまあまあまあ
まあまあまあまあまあまあまあまあまあまあまあまあまあまあまあまあまあまあ
まあまあまあまあまあまあまあまあまあまあまあまあまあまあまあまあまあまあ
まあまあまあまあまあまあまあまあまあまあまあまあまあまあまあまあまあまあ
まあまあまあまあまあまあまあまあまあまあまあまあまあまあまあまあまあまあ
まあまあまあまあまあまあまあまあまあまあまあまあまあまあまあまあまあまあ
まあまあまあまあまあまあまあまあまあまあまあまあまあまあまあまあまあまあ
まあまあまあまあまあまあまあまあまあまあまあまあまあまあまあまあまあまあ
まあまあまあまあまあまあまあまあまあまあまあまあまあまあまあまあまあまあ
まあまあまあまあまあまあまあまあまあまあまあまあまあまあまあまあまあまあ  
716某研究者:03/03/05 16:42 ID:UaGLuzaA
ロードス島戦記等で剣でフルプレートを着た騎士を
一撃で倒すシーンが良く有るが
当時は魔剣でも無ければ無理だと思っていたが
上を見れば実際片手剣でさえ一撃でやられ兼ねぬ訳だろうか(苦笑
(グインサーガで盾を割って後ろの人間を倒すと言う様な物が
 有ったが其れも別段非現実的では無い訳だろうか)
717まあまあまあまあまあまあ:03/03/05 16:43 ID:???
まあまあまあまあまあまあまあまあまあまあまあまあまあまあまあまあまあまあ
まあまあまあまあまあまあまあまあまあまあまあまあまあまあまあまあまあまあ
まあまあまあまあまあまあまあまあまあまあまあまあまあまあまあまあまあまあ
まあまあまあまあまあまあまあまあまあまあまあまあまあまあまあまあまあまあ
まあまあまあまあまあまあまあまあまあまあまあまあまあまあまあまあまあまあ
まあまあまあまあまあまあまあまあまあまあまあまあまあまあまあまあまあまあ
まあまあまあまあまあまあまあまあまあまあまあまあまあまあまあまあまあまあ
まあまあまあまあまあまあまあまあまあまあまあまあまあまあまあまあまあまあ
まあまあまあまあまあまあまあまあまあまあまあまあまあまあまあまあまあまあ
まあまあまあまあまあまあまあまあまあまあまあまあまあまあまあまあまあまあ
まあまあまあまあまあまあまあまあまあまあまあまあまあまあまあまあまあまあ
まあまあまあまあまあまあまあまあまあまあまあまあまあまあまあまあまあまあ
まあまあまあまあまあまあまあまあまあまあまあまあまあまあまあまあまあまあ
まあまあまあまあまあまあまあまあまあまあまあまあまあまあまあまあまあまあ
まあまあまあまあまあまあまあまあまあまあまあまあまあまあまあまあまあまあ
まあまあまあまあまあまあまあまあまあまあまあまあまあまあまあまあまあまあ
まあまあまあまあまあまあまあまあまあまあまあまあまあまあまあまあまあまあ
まあまあまあまあまあまあまあまあまあまあまあまあまあまあまあまあまあまあ
まあまあまあまあまあまあまあまあまあまあまあまあまあまあまあまあまあまあ
まあまあまあまあまあまあまあまあまあまあまあまあまあまあまあまあまあまあ
まあまあまあまあまあまあまあまあまあまあまあまあまあまあまあまあまあまあ
まあまあまあまあまあまあまあまあまあまあまあまあまあまあまあまあまあまあ
まあまあまあまあまあまあまあまあまあまあまあまあまあまあまあまあまあまあ
まあまあまあまあまあまあまあまあまあまあまあまあまあまあまあまあまあまあ
まあまあまあまあまあまあまあまあまあまあしかしあまあまあまあまあまあまあ     
718某研究者:03/03/05 16:49 ID:UaGLuzaA
>ロードス島戦記等で剣でフルプレートを着た騎士を
>一撃で倒すシーンが良く有るが
>当時は魔剣でも無ければ無理だと思っていたが
>上を見れば実際片手剣でさえ一撃でやられ兼ねぬ訳だろうか(苦笑

まあ現実は逆に魔法の鎧でも無いと
片手剣の攻撃さえ完全に防御する事は
無理である訳だろうか
(矢張りショートソードでもモーションの大きい攻撃を受ければ
 普通の鎧では厳しいだろうか)
719名無し三等兵:03/03/05 16:55 ID:???
このスレって、アルバレストやクインクレーンの話題もOK?
720某研究者:03/03/05 17:01 ID:UaGLuzaA
>このスレって、アルバレストやクインクレーンの話題もOK?

アルバレストと言うのはクロスボウの一種だろうが
プレートはどの程度の距離で貫けるのだろうか
(銃より発射速度(毎分1発・銃は2−3発)が遅く
 射程も短い(クロスボウ350m・銃500m)ので
 銃が出た後は廃れた訳だろうか
 銃は盾を貫いて後ろの人間を仕留められるが弓では無理と言う事も
 有るだろうか)
721某研究者:03/03/05 17:10 ID:UaGLuzaA
まあ只西欧の鏃と言うのは矢に固定されていない訳だろうし
矢に単に被せてあるだけであるなら
盾を貫通した後矢の軸が引っ掛かっても
鏃だけが飛んで後ろの兵が傷付くと言う可能性も有る訳だろうか
(まあプレートアーマーを着ていれば飛んで来た鏃で
 やられる可能性は恐らく無い訳だろうが
 バリスタの鏃ではどうだろうか(苦笑)
722名無し三等兵:03/03/05 17:12 ID:???
>某研

うぜーなこいつ…
723某研究者:03/03/05 17:17 ID:UaGLuzaA
>盾を貫通した後矢の軸が引っ掛かっても

日本の矢の鏃は固定されていたのかと言う事だろうが
八人張りの弓等は矢毎人間や盾を貫通して後ろの人間に
ダメージを与え得た訳だろうか
724某研究者:03/03/05 17:28 ID:UaGLuzaA
まあビデオ程の力を掛けなく共鎧を着た敵を
戦闘不能・或いは倒す事は出来るだろうし
女が両手で剣を振るえば十分鎧を着た相手を倒せるだろうし
片手剣でも何とか成るだろうか
725某研究者:03/03/05 18:06 ID:UaGLuzaA
まあ片手用のウオーハンマーの威力にも驚いたが
尖っていない部分の先端が完全に兜の内部に入り込んで仕舞い
兜に大穴が空いた訳だが(苦笑
盾がこれをまともに受ければどうなるのだろうか
(モーションも余り大きく無いし
 この程度のモーションでこの威力ではかなりコンパクトな
 モーションでも鎧を着た相手を一撃で倒せるだろうか)
726名無し三等兵:03/03/05 18:10 ID:5ff2aTY3
馬鹿だね。
鏃を固定しないのはインディアンの弓だろ。

西欧はするぞw
しないってならソースを聞いてみたいw
727某研究者:03/03/05 18:15 ID:UaGLuzaA
>尖っていない部分の先端が完全に兜の内部に入り込んで仕舞い
>兜に大穴が空いた訳だが(苦笑

まあこれを見て仕舞うと両手用ウオーハンマーの威力はどうなのか
想像する事すら恐ろしいが
フルプレートを着て盾を構えても盾毎粉砕される様な威力だろうか
これだとかなり小さいモーションでも敵を倒せそうであり
軽い物なら敵の剣の攻撃も有効に防御可能だろうか

まあ漫画やアニメ以上に現実の白兵戦用武器の威力と言うのは
大きい様だが
鎖帷子等もまるで紙を切る様に
剣で切り裂かれている訳だが(苦笑
728某研究者:03/03/05 18:18 ID:UaGLuzaA
>鎖帷子等もまるで紙を切る様に
>剣で切り裂かれている訳だが(苦笑

ロードス島戦記でアシュラムが鎖帷子を着た兵士を簡単に切り裂いていたが
別に魔剣で無く共あの様な事は容易である訳だろうか(苦笑
729某研究者:03/03/05 18:33 ID:UaGLuzaA
>西欧はするぞw
>しないってならソースを聞いてみたいw

イングランドのロングボウ兵の鏃は矢に固定されていない
(これは矢を抜いても体内に鏃を残す為だが)と聞いたが
他国の兵はどうなのだろうか
(しかし鏃を矢に固定しない場合矢が盾を貫いた後鏃のみが飛ぶ等と言う現象が
 起こり得るのかと言う事だが)
730某研究者:03/03/05 20:59 ID:GPqlSasQ
まあ上の実物の剣を一人で振っているビデオと剣道の剣速の差を見れば
何処かのドラマや漫画では無いが
剣道等所詮遊びに過ぎないと言うのも
一面真実だろうか(苦笑
731AIRSHIP:03/03/05 22:18 ID:fgNdi2Hb
>>某研究者様
研究熱心なのは、真摯に認める所ですがもう少し日本文化にも造詣を深くしていただ
ければ有難いと思います。
絵の殆どが舞台絵と役者絵であって,実際の市井の風物を映した物ではない事を理解
して下さい。 
658に有る馬は,竹馬で下が輪になっていて,棒の部分に跨って子供が遊ぶもので
す。 弓は,室内用の枕弓,又は李満弓と呼ばれるものでしょう,武家の護身用の物
です。 書き方によっては,廓用の射的用にも見えますが?
以前の質問に、桜の枝に文箱を付けたが何か、の質問が有りましたが,枕草子にも出
てくる有名な手紙のやり取りです。
 浮世絵の多くは,歌舞伎十八版の中から取られたものが多いようです。 私も全部
は見ていませんが,(レス内の絵も,歌舞伎も)日本史にも関心を持って頂きたい所
です。 腕のマークには驚きました。 紋所の意味をご存じ内とは無いとは思いませ
んが? 引両の紋の意味はご存じと思いますし,役者の家に伝わる有名な物です。
江戸時代の商人の風俗には,詳しくはないですが,外郎売と傀儡子ぐらいは有名な所
です。 早口言葉の「武具馬具武具馬具三武具馬具,合わせて武具馬具・・・・」と
有名な台詞を喋る所です。 クグツカラクリも江戸の風物の一つですし、クグツ舞は
難しいので有名です。


732某研究者:03/03/06 12:46 ID:EkhqDVqx
まあ上の詳細に関しては何れ時間を掛けて調べるとしたいが

上のビデオを見ると
時代劇等で刀で鎧武者を鎧の上から斬るシーンと言うのも
別段非現実的では無いと言う事だろうか
まあそうすると鎧は剣では無く遠距離からの流れ弾や槍の攻撃を防ぐ為の物かも知れぬが
(剣に対しては相手のモーションを大きくし
 モーションの極小さな攻撃・掠った程度の攻撃・或いはエペ等の極軽い武器の打撃を止める効果しか無いだろうか)
槍でもモーションを大きくする・或いはチャージからの突きでは鎧を貫通し
槍で殴られた場合は兜や肩の鎧は無力であると言う可能性は無いのだろうか

手裏剣や短剣を投げられた場合鎧はこれを止められるだろうか
(矢張り鎖帷子では厳しい場合も有るだろうか)
ラッパ銃等から放たれる小さな散弾も恐らく止まるだろうが
矢張り投げ槍では完全に貫通し刀を投げられても厳しいだろうか
733某研究者:03/03/06 12:49 ID:EkhqDVqx
>或いはエペ等の極軽い武器の打撃を止める効果しか無いだろうか

まあ打撃は止まってもモーションの大きい突きでは
鎧が貫通する可能性も有るだろうし
モーションの大きいエペの打撃を受ければ
鎧の上からでさえ戦闘不能に陥る可能性も有るだろうか
734某研究者:03/03/06 13:01 ID:EkhqDVqx
>槍でもモーションを大きくする・或いはチャージからの突きでは鎧を貫通し
>槍で殴られた場合は兜や肩の鎧は無力であると言う可能性は無いのだろうか

まあ槍に対してもプレートアーマーでさえ
モーションの小さい突きや叩き・
掠った程度の攻撃を防ぐ事しか出来ぬと言う事だろうか
735某研究者:03/03/06 13:09 ID:EkhqDVqx
まあ槍より重いハルバードや長刀では小さなモーションからの打撃或いは
モーションの小さい突きでもダメージを受けるだろうが
モーションの大きいハルバードの打撃を受ければ
盾の上からでも粉砕される可能性は十分有る訳だろうか
(チャージからの突きであれば盾毎鎧を貫く可能性も有るだろうか
 ランスチャージに対して盾や鎧と言うのは役に立たず
 パイクやハルバードで騎兵は阻止するか
 騎兵対騎兵の戦いではランスは盾では受けられず
 (まあ騎兵より先に馬がやられるだろうが
 馬鎧もランスチャージでは難なく貫通だろうか)
 回避するしか無い訳だろうか)
736某研究者:03/03/06 13:13 ID:EkhqDVqx
馬上で野太刀を振るう様な事も有った様だが
上のビデオを見る限り馬上で
片手用ハンマーや両手剣の攻撃を受ければこれを盾で防げるのだろうか
(矢張り回避するか武器で受ける・受け流すしか無い様にも思えるが(苦笑)
737某研究者:03/03/06 15:55 ID:IwsKpiOc
ベルセルクで刃毀れした斧が兜を割れなかったシーンが有ったが
矢張り現実には歯毀れ等何の意味も無く兜は割れるだろうか(苦笑

まあポールアックスを振るえば中に人間の入っていない兜を
上から両断する事は可能だろうか

剣やハンマーで鎧を叩くビデオは有るが
銃や弓で鎧を撃つビデオと言うのが見当たらない訳だが
何処かに無いだろうか
(まあ100m程度先から銃で鎧を撃った静止画像は有るが
 鎧の後ろの板迄完全に貫かれていたが(苦笑)
 
738某研究者:03/03/07 10:58 ID:Cdb6J1Kd
739某研究者:03/03/07 11:31 ID:Cdb6J1Kd
http://133.9.157.108/enpakunishik/FMPro?-db=nishikie.fmj&-format=results-big.htm&-lay=layout2&haiyaku=%82%a8&-max=1&-skip=53&-find=
障子に文字が書いてあるが
この様な形式の物は多数有るのだろうか

http://133.9.157.108/enpakunishik/FMPro?-db=nishikie.fmj&-format=results-big.htm&-lay=layout2&haiyaku=%82%a8&-max=1&-skip=58&-find=
女同士が戦っている絵も珍しいだろうが
この木刀は道中挿し程度の長さだろうか
(矢張り道中挿しを使う訓練を受けていた女も居るのだろうか)

http://133.9.157.108/enpakunishik/FMPro?-db=nishikie.fmj&-format=results-big.htm&-lay=layout2&haiyaku=%82%a8&-max=1&-skip=60&-find=
これは前にも紹介した物だが
竹製の柄や鞘を持つ刀と言うのも有った訳だろうか
(或いはこれは仕込み杖では無いのだろうか)

http://133.9.157.108/enpakunishik/FMPro?-db=nishikie.fmj&-format=results-big.htm&-lay=layout2&haiyaku=%82%a8&-max=1&-skip=63&-find=
まあしかしこれは一体何の服なのだろうか

http://133.9.157.108/enpakunishik/FMPro?-db=nishikie.fmj&-format=results-big.htm&-lay=layout2&haiyaku=%82%a8&-max=1&-skip=64&-find=
これ等は妙に漫画的な構図では有るが
740某研究者:03/03/07 12:04 ID:Cdb6J1Kd
http://133.9.157.108/enpakunishik/FMPro?-db=nishikie.fmj&-format=results-big.htm&-lay=layout2&haiyaku=%82%a8&-max=1&-skip=67&-find=
上の服の三角形の模様は
例のインドの文様(トゥンパル)が変化した物なのだろうか

http://133.9.157.108/enpakunishik/FMPro?-db=nishikie.fmj&-format=results-big.htm&-lay=layout2&haiyaku=%82%a8&-max=1&-skip=70&-find=
まあしかしこの状況でどうやって口に筆を運んだのだろうか(苦笑

http://133.9.157.108/enpakunishik/FMPro?-db=nishikie.fmj&-format=results-big.htm&-lay=layout2&haiyaku=%82%a8&-max=1&-skip=85&-find=
まあしかしこんな簡単に馬を止められる物なのだろうか(苦笑

http://133.9.157.108/enpakunishik/FMPro?-db=nishikie.fmj&-format=results-big.htm&-lay=layout2&haiyaku=%82%a8&-max=1&-skip=86&-find=
此れは男が女に倒されている様にも見えるがどうなのだろうか

http://133.9.157.108/enpakunishik/FMPro?-db=nishikie.fmj&-format=results-big.htm&-lay=layout2&haiyaku=%82%a8&-max=1&-skip=87&-find=
まあ木刀を逆手に構える事も有ったのか
しかしこれは妙に動きの有る構図だが(笑

http://133.9.157.108/enpakunishik/FMPro?-db=nishikie.fmj&-format=results-big.htm&-lay=layout2&haiyaku=%82%a8&-max=1&-skip=96&-find=
芸者は皆道中指しを扱えたと言う訳でも無いだろうが
しかし女が武器を持っている絵が異様に多くないだろうか(笑
741某研究者:03/03/07 15:54 ID:Cdb6J1Kd
http://133.9.157.108/enpakunishik/FMPro?-db=nishikie.fmj&-format=results-big.htm&-lay=layout2&haiyaku=%82%a8&-max=1&-skip=104&-find=
まあこれは派手な女の服と地味な僧侶の服と言う対比を
考えているのだろうか

http://133.9.157.108/enpakunishik/FMPro?-db=nishikie.fmj&-format=results-big.htm&-lay=layout2&haiyaku=%82%a8&-max=1&-skip=120&-find=
まあしかしこんな戦い方が合理的なのかどうかだが(笑
(鍔に指を掛けている様な構え方が多いが
 これは合理的なのだろうか)
742名無し三等兵:03/03/07 16:30 ID:???
某研閣下!
↓のスレでお呼びですぞ!!

http://academy.2ch.net/test/read.cgi/whis/1046862617/l50
743某研究者:03/03/07 21:47 ID:Ma8owyrU
>(鍔に指を掛けている様な構え方が多いが
> これは合理的なのだろうか)

http://member.nifty.ne.jp/bintaro/touken/walloon.htm
上のワルーンソードは鍔に親指を掛ける部分が有るが
これは意味は有る訳なのだろうか
744某研究者:03/03/07 21:53 ID:Ma8owyrU
http://member.nifty.ne.jp/bintaro/touken/saber.htm
http://member.nifty.ne.jp/bintaro/touken/backsword.htm
http://member.nifty.ne.jp/bintaro/touken/pallasch.htm
http://member.nifty.ne.jp/bintaro/touken/hanger.htm
まあ上の剣にも同様に親指を掛ける部分が有る様だが
雑兵の使うハンガーに迄有るとは思わなかったが(苦笑
745某研究者:03/03/09 19:27 ID:dHr2+TB9
http://www.tnm.go.jp/db/frjf_pub/tmb00047/frjf1024/c0004836.jpg
しかしこんな場面迄有るとは妙に現代的で驚いたが
当時の喫煙率はどの程度だったのだろうか(苦笑
18世紀にこの様な場面が描けるのだから矢張り鈴木春信は只者では無いだろうか
(後ろの人間(これは僧侶だろうか)
 下駄を持っているのは市中では下駄に履き替える為なのだろうか)
746某研究者:03/03/09 19:45 ID:dHr2+TB9
http://www.tnm.go.jp/db/frjf_pub/tmb00047/frjf1024/c0004832.jpg
まあしかし黒い物を被っているのは例の忍者では
無いのだろうか

747某研究者:03/03/09 20:18 ID:dHr2+TB9
http://www.tnm.go.jp/db/frjf_pub/tmb00185/frjf1024/c0014531.jpg
まあしかし左の人物の持っている棒は一体何なのか
中央の人間の服には顔が書いてある(まあ顔と言うより面だろうか)
様にも見えるがこの様な形式の物は余り見ないが

http://www.tnm.go.jp/db/frjf_pub/tmb00141/frjf1024/c0013104.jpg
恋の矢文等と気障な真似が本当に行われていたのかと言う事だが
西欧の天使等を模倣した可能性は無いのだろうか
748某研究者:03/03/09 20:59 ID:dHr2+TB9
http://www.tnm.go.jp/db/frjf_pub/tmb00389/frjf1024/c0027982.jpg
まあこう言う時代劇に良く出て来る場面が
本当に有ったという事だろうか(苦笑

http://www.tnm.go.jp/db/frjf_pub/tmb00389/frjf0924/c0027996.jpg
まあ模様の付いている傘と言うのは余り見ないが

749AIRSHIP:03/03/09 22:32 ID:uz2EIo6M
>>某研究者様
西洋の剣の握り方に就いてですが、時代と剣の種類によって、ロマネスク握、前期ゴチ
ック握、後期ゴチック握、等が在り指掛と指の掛け方にも種種の違いが在ります。
サーベルやナイフの柄によっては、手指の形の握りが着いた物も在りますが、鍔に指を
掛ける事に依って、操作性が向上します。
750某研究者:03/03/10 16:01 ID:m8M1VJG/
http://search.famsf.org/4d.acgi$Record?22166&=list&=1&=harunobu&=And&=8&=0&=keywords&=Yes&=&=&=&=Yes&=&=f
まあこの猫は矢張りお仙の猫なのだろうか(笑

http://www2.library.tohoku.ac.jp/kano/05-001079/05-001079l010.html#gazou
まあこれは一番右の人物と左の情景との対比と言う要素も
有るのだろうか(苦笑

http://www2.library.tohoku.ac.jp/kano/05-001079/05-001079l006.html#gazou
中国に有る様な鳥型の凧も
日本に有った訳だろうか

http://www2.library.tohoku.ac.jp/kano/05-001079/05-001079l016.html#gazou
人間を棒の上に載せている様にも見えるが
これは一体何なのだろうか

http://www2.library.tohoku.ac.jp/kano/05-001079/05-001079l034.html#gazou
まあしかしこんな大きな雪玉を女が作ったのだろうか(苦笑
751某研究者:03/03/11 22:12 ID:fbYDuDjq
まあ矢張りハンニバルとスキピオ・
或いはゲクランとエドワード等では無く
上杉謙信と武田信玄との戦いが
男性原理の頂点であるとの意見も有る訳だろうが
上杉謙信は女であるとの意見も有る訳だろうか(苦笑
752某研究者:03/03/11 22:15 ID:fbYDuDjq
>上杉謙信と武田信玄との戦いが

足利尊氏と楠木正成との戦いもそうかも知れぬが
終盤は足利が手正成を投降させる為に手加減していた部分も
有る訳だろうか
753某研究者:03/03/11 22:16 ID:fbYDuDjq
サラディンとリチャードも同様であると言う意見も有るだろうが
片方は野蛮な虐殺者に過ぎぬと言う意見も
矢張りアラブ側には多い訳だろうか
754某研究者:03/03/13 11:06 ID:XTnuDCCP
http://www2.library.tohoku.ac.jp/kano/05-001079/05-001079l033.html#gazou
まあ侍は兎も角市民の姿勢と言うのは
悪かったのだろうか(苦笑

http://www2.library.tohoku.ac.jp/kano/05-001086/05-001086l002.html#gazou
しかしこの絵は一体何なのだろうか

http://www2.library.tohoku.ac.jp/kano/05-001110/05-001110l005.html#gazou
まあしかしこれだけ馬に乗せたら重く無いのだろうか(苦笑
755山崎渉:03/03/13 14:17 ID:???
(^^)
756某研究者:03/03/13 17:07 ID:DmJC4uuI
まあしかし妙に下がっているから上げておくとしたいが(苦笑
757某研究者:03/03/13 20:52 ID:lHfsbz0k
http://www1.kcn.ne.jp/~areyou/kibyou/index.html
まあ上の本の人間の表情を見ると確かに時代劇で演じられている其れと
非常に似ている訳であるが
仮に黄表紙や洒落本が規制されなかったら
日本には民主主義の様な物が生じた可能性と言うのは有る訳なのだろうか
758某研究者:03/03/13 20:53 ID:lHfsbz0k
まあしかし黄表紙には漫画の様な吹き出し迄有るので驚いたが
西欧に似た様な本は有ったのだろうか
759某研究者:03/03/13 21:00 ID:lHfsbz0k
http://www1.kcn.ne.jp/~areyou/kibyou/una2/2-6.htm
時代劇や浮世絵で髪を剃った部分が
青く成っている物が有るが其れは実際に
青い色が付けられていたと言う事だろうか
760某研究者:03/03/13 23:37 ID:lHfsbz0k
http://www.jti.co.jp/Culture/museum/ukiyoe/jpg/96L.jpg
右端の人間はズボンの様な物を履いているが
女なのだろうか

http://www.jti.co.jp/Culture/museum/ukiyoe/jpg/636L.jpg
まあ時代劇に出て来る物より実物の方が
やや派手である訳だろうか
761某研究者:03/03/13 23:56 ID:lHfsbz0k
http://www.nara-edu.ac.jp/ARCHIVE/UKIYOE/8-3.jpg
http://www.nara-edu.ac.jp/ARCHIVE/UKIYOE/8-4.jpg
http://www.nara-edu.ac.jp/ARCHIVE/UKIYOE/8-2.jpg
http://www.nara-edu.ac.jp/ARCHIVE/UKIYOE/8-1.jpg
上の様な柄の服は時代劇では見ないが
実際存在したのだろうか
(日本と言うか中国に似ているが
 良く見ると柄は日本的な物が多い訳だろうか)

http://www.nara-edu.ac.jp/ARCHIVE/UKIYOE/7-2a.jpg
これ等の時代劇等で見る服より派手であるし
矢張り時代劇や大河ドラマもある意味マンネリ化している合戦シーンに金を掛けるのではなく
偶にはこの様な部分に金を掛けて見るのも良いのではないのか
762某研究者:03/03/14 00:20 ID:QTce7L4K
http://www.nara-edu.ac.jp/ARCHIVE/UKIYOE/toyokuni.htm
上に有る絵の服は役者にしても
随分派手な物が多いが
実際にこの様な服を着ていたのだろうか

http://www.nara-edu.ac.jp/ARCHIVE/UKIYOE/6-1.jpg
この服の柄も(無論他の物も大概は)面白いが
上の人物は女なのだろうか

http://www.nara-edu.ac.jp/ARCHIVE/UKIYOE/9-4.jpg
上の人物は南蛮的な襟の有る服を着ているが
上の様に女が刀を持つ事も有った訳だろうか

http://www.nara-edu.ac.jp/ARCHIVE/UKIYOE/11.jpg
上の絵も女が刀を持っているが
刀を持っている女と言うのは武術が皆使えた可能性は
有るのだろうか

http://www.nara-edu.ac.jp/ARCHIVE/UKIYOE/genjigo.htm
右の人物の服の模様は
幾何学模様の中に柄が入っているが
これは矢張り珍しい物なのだろうか
763某研究者:03/03/14 00:33 ID:QTce7L4K
http://www.nara-edu.ac.jp/ARCHIVE/UKIYOE/15.jpg
確かに派手な服を着た人間は何人も居る様だが
これは殆ど中国の様だが
何時頃からこの様な派手な服が登場したのだろうか
(まあ雅楽等では以前から有った訳だろうが)

http://www.nara-edu.ac.jp/ARCHIVE/UKIYOE/19.jpg
上は名前が書かれている部分に迄色が塗ってあり
何か妙に現代的である訳だが

http://www.nara-edu.ac.jp/ARCHIVE/UKIYOE/22.jpg
左の人間は妙に男性的な格好をしているが
女なのだろうか
764某研究者:03/03/14 00:56 ID:QTce7L4K
http://www.nara-edu.ac.jp/ARCHIVE/UKIYOE/kokoro.htm
この様な物が有ると言う事は
人物の性格付けがかなり細かくされていたと言う事なのだろうか

http://www.nara-edu.ac.jp/ARCHIVE/UKIYOE/28.jpg
http://www.nara-edu.ac.jp/ARCHIVE/UKIYOE/29.jpg
http://www.nara-edu.ac.jp/ARCHIVE/UKIYOE/34.jpg
http://www.nara-edu.ac.jp/ARCHIVE/UKIYOE/35-2.jpg
まあ一体何なのだと言う位複雑な柄の服が
多数有ったのかも知れぬが一体幾ら掛かっていたのだろうか

765某研究者:03/03/14 06:11 ID:QTce7L4K
天明の大飢饉が無ければ田沼意次は失脚せず
ロシアとの貿易や欧米との貿易の拡大が可能と成り
黄表紙や洒落本の表現も規制されず
或いは倹約令等も出なかった訳だろうが
(矢張り鈴木春信以降の浮世絵の構図や描写が
 何か堅苦しいのは個々の作家の創造性の不足や
 西欧絵画の影響のみでは無く
 田沼失脚後の無意味な改革に拠る表現の規制の影響が
 出ている可能性も有るだろうか)
まあ飢饉が無ければ意次が死んだ後も
貿易拡大路線や経済の自由化は維持され
表現の自由も維持されていたとすれば
欧米の干渉が無く共何れは民主主義の様な体制が
日本に生じていた可能性は無いのだろうか
(まあ欧米の干渉が有っても民主化がより容易に進み
 浮世絵等の芸術の表現もより自由で有った筈であり
 世界の芸術により影響を与え世界の文化の自由化も
 進展しこれが民主主義や科学の進歩迄
 加速していたならWW2等の大戦争は
 起きなかった可能性も有る訳だろうか)
766某研究者:03/03/14 06:32 ID:QTce7L4K
まあ天明の大飢饉が無ければ春信や写楽以上の化物が
沢山出て来たと言う事も有るのだろうか
西欧の影響で浮世絵等の構図が形式化しつまらなく成った部分も有るだろうし
欧米の亜細亜進出がもう少し遅れておれば
面白い物が沢山出て来た可能性も高いだろうか
767某研究者:03/03/14 07:08 ID:QTce7L4K
まあ鈴木春信が錦絵を描いたのは僅か5年であるし
寛政の改革が有ったとしても後20・
改革が無かった場合は更に長期間
自由に絵が描けた訳だろうから
其の間に何千・何万種類ものあの様な絵が大量に作られれば
世界の歴史が変わった可能性も有る訳だろうし
意次が失脚しても自由な絵が大量に浸透すれば
これを最早規制する事は困難であり
改革は無くなり意次失脚後も絵は自由に描けたと言う可能性も有るだろうか
768AIRSHIP:03/03/14 21:13 ID:Z6C3ZEyD
某研究者様
上記の絵の多くが廓(遊郭)を描いたもので、花魁の似姿(現代のプロマイドや、ピ
ンナップ)で、派手な衣装が多いのは当然でしょう。 今の世でも、芸能人のステー
ジ衣装が正気の沙汰とは思えないほど凄い物も多いですからね。
江戸時代の大奥や大名屋敷の女性専用区画では、別色女という女性の武芸者が警備や
奥方、女房集のガードマン的役割を果たしていましたので、女性が武器を持っている
のは不思議では有りません。 大奥の女性達は、交代で長刀を持ち夜間の見回りを行
う部署も有りましたので、武芸の稽古も日常的に行われていたと推測されます。
寛政の改革が無ければ、自由に絵をかけたろうという意見には賛成(但し、以前にも
制約が別にあってそれには従っていたのは確かでしょう) 改革が無ければ、日本の
服装に変化が大きくなったのは確かでしょう。 色彩、絵柄等に対する制限と共に、
生地にたいする制限も大きかったのですから。
769某研究者:03/03/14 21:36 ID:KLLfiEqC
>今の世でも、芸能人のステー
>ジ衣装が正気の沙汰とは思えないほど凄い物も多いですからね。

まあ中国でも柄は細かい物は有るだろうがここ迄派手な物は見ないし
こう言う物が何の規制も無く西欧の影響も受けずに発展していたら
一体どんな物が出て来たかと言う事だろうし
其の場合は日本人にオリジナリティが無い等と言う者が
居たかと言う事だろうが(苦笑

>(但し、以前にも
>制約が別にあってそれには従っていたのは確かでしょう)

まあ其の規制も田沼政治が継続されれば制約が緩んだ可能性も有るだろうし
改革や西欧の影響の無い日本がどう成ったのかも見てみたかった所だが(笑
770某研究者:03/03/17 22:03 ID:NYlUGYQX
戦国時代の南蛮胴は恐らくベルトで固定されているだろうが
ベルトの様な物は以後日本では用いられなかったのだろうか
771名無し三等兵:03/03/17 22:05 ID:???
test
772某研究者:03/03/19 16:04 ID:iTEp+7H0
まあ現在の中国人が民族衣装としているのは女真族の物であり
漢民族の物では無く
漢民族は確か女真族・金の侵入で南方に追い遣られた訳だろうし
其の際に南部の人間と混血して別人種と成った可能性も有るだろうし
北部に残った人間は女真族と混血した訳であるなら
金に侵攻される前に中国文明の基礎を築いた純粋な漢民族と言うのは消えて仕舞っており
日本人の遺伝子が意外と元々の漢民族に一番近い形で残っていると言う事は無いのだろうか
773某研究者:03/03/19 16:10 ID:iTEp+7H0
漢民族は南方系では無いが
モンゴル人や女真族の様な新モンゴロイドでは無く
旧モンゴロイドであり日本人に近い人種だった可能性は
無いのだろうか
774某研究者:03/03/19 16:22 ID:iTEp+7H0
まあ黄河文明は西安辺りが中心であるなら
漢民族と言うのは日本人よりはやや南方系の
人間である訳だろうか
(日本人の南方系の人間が
 彼等に似ていると言う可能性も有るだろうが
 沖縄人等に近い可能性はどうなのだろうか)
775某研究者:03/03/19 16:29 ID:iTEp+7H0
>まあ現在の中国人が民族衣装としているのは女真族の物であり
>漢民族の物では無く

女真族も日本人同様漢民族に取っては蛮族だろうし
彼等の衣服を使っている事は最早中国=漢民族では
無いと言う事なのだろうか
776名無し三等兵:03/03/22 00:57 ID:???
巡回保全 
777某研究者:03/03/23 00:04 ID:fDW/wQEE
まあこの状況では
定期的に上げておかないと駄目だろうか(苦笑 
778某研究者:03/03/31 12:25 ID:68HC6CLC
まあ市民や兵士の虐殺も
只殺せば損害も出るだろうから
捕虜にすると見せ掛けて武装を解除させ一箇所に集めた後で
始末すると言う手も使われた訳だろうか
779名無し三等兵:03/04/08 14:25 ID:???
  
780名無し三等兵:03/04/13 14:08 ID:???
  
781名無し三等兵:03/04/13 16:33 ID:Mi2jYkeB
あきらめていた髪を見事に復活させます「ダーバオ育毛剤」・・・6,500円
あの101の最新版!専用シャンプーとの組み合わせで更に強力な発毛効果!「101G」・・・3,500円
中国国家が唯一認定した世界に誇る抗がん漢方薬「天仙液」20箱・・・39,000円
最高級アガリクスが1kg驚きのこの価格・・・10,000円
邱永漢も大絶賛の驚異の糖尿病特効漢方薬「愈消散」・・・9,000円
性格を明るく変える鬱病治療薬「プロザック」・・・7,000円
オランダ製低容量ピル「マーベロン」2ヶ月セット・・・3,500円
1日1錠安全ダイエット欧米でも認可「曲美」・・・8,000円
驚異の黒アリエキスが下半身を直撃!最強精力サプリ「蟻力神」・・・7,900円
中国国家が認可したインポ・早漏治療薬「健陽カプセル」・・・9,800円

その他いろいろな商品がどこよりも安い!どこよりも速い!どこよりも丁寧!
http://www.kanpouya.com/
782山崎渉:03/04/17 11:12 ID:???
(^^)
783某研究者:03/04/18 21:44 ID:Un5hlfAv
まあ中々書く時間も無いが
上げて置かないと駄目だろうか
784名無し三等兵:03/04/18 21:51 ID:???
>>某研究者
おれはこのスレは良いと思うぞ。
がんばれ
785山崎渉:03/04/20 05:23 ID:???
   ∧_∧
  (  ^^ )< ぬるぽ(^^)
786某研究者:03/05/01 09:10 ID:havdB4tO
長篠で使われたと言う塹壕は大砲に対しても
有効だったろうが
朝鮮の明軍の大砲や小火器等(無論矢もある程度は防げるだろうが)
を防ぐ為に塹壕が作られたと言う事は無いのだろうか
(或いは塹壕は信長軍以外は用いなかったのだろうか)

まあしかし西欧の軍隊が塹壕を使用したのは
どの段階でだろうか
787名無し三等兵
1!